94
RECREA Ţ II MATEMATICE REVIST Ă DE MATEMATIC Ă PENTRU ELEVI Ş I PROFESORI Asocia ţ ia “Recrea ţ ii Matematice” IA Ş I - 2013 Iulie Decembrie 2013 Anul XV, Nr. 2 1 = π i e

format .pdf, 3.6 MB

Embed Size (px)

Citation preview

Page 1: format .pdf, 3.6 MB

RECREAŢII MATEMATICE

RE V IS TĂ DE M AT E M AT I CĂ P E NT R U E L E V I Ş I P R O F E S O R I

A s o c i a ţ i a “ R e c r e a ţ i i M a t e m a t i c e ” I AŞ I - 2 0 1 3

Iulie – Decembrie 2013Anul XV, Nr. 2

1−=πie

Page 2: format .pdf, 3.6 MB

Semnificaţia formulei de pe copertă. Într-o formă concisă, formula 1−=πie leagă cele patru ramuri fundamentale ale matematicii: ARITMETICA - reprezentată de 1; GEOMETRIA –reprezentată de π ; ALGEBRA – reprezentată de i ; ANALIZA MATEMATICĂ – reprezentată de e. Membrii fondatori : Temistocle BÎRSAN Alexandru CĂRĂUŞU Adrian CORDUNEANU Cătălin CALISTRU Constantin COCEA Gheorghe IUREA Membri onorifici : Acad. Constantin CORDUNEANU Acad. Radu MIRON Prof.univ. Vasile OPROIU Cercet.pr. Dan TIBA Redactor şef : Temistocle BÎRSAN Redactori principali : Gabriel POPA – coordonator al rubricilor de probleme Gheorghe IUREA, Petru ASAFTEI, Maria RACU Comitetul de redacţie : Alexandru CĂRĂUŞU Paul GEORGESCU Marian TETIVA (Bârlad) Constantin CHIRILĂ Gheorghe ILIE Lucian TUŢESCU (Craiova) Eugenia COHAL Gabriel MÎRŞANU Adrian ZANOSCHI Adrian CORDUNEANU Dan POPESCU (Suceava) Titu ZVONARU (Comăneşti) Mihai CRĂCIUN (Paşcani) Neculai ROMAN (Mirceşti) Paraschiva GALIA Ioan ŞERDEAN (Orăştie) Materialele vor fi trimise la una dintre adresele: [email protected] , [email protected] COPYRIGHT © 2008, ASOCIAŢIA “RECREAŢII MATEMATICE” Toate drepturile aparţin Asociaţiei “Recreaţii Matematice”. Reproducerea integrală sau parţială a textului sau a ilustraţiilor din această revistă este posibilă numai cu acordul prealabil scris al acesteia.

TIPĂRITĂ LA BLUE SIM&Co IAŞI Bd. Carol I, nr. 3-5 Tel. 0332 111021, 0721 571705; e-mail: [email protected]

ISSN 1582 – 1765

Page 3: format .pdf, 3.6 MB

RECREAŢ I I MATEMATICE

RE V IS TĂ DE M AT E M AT I CĂ P E NT R U E L E V I Ş I P R O F E S O R I

Revistă cu apariţie semestrială

EDITURA “RECREAŢII MATEMATICE”

IAŞI - 2013

Anul XV, Nr. 2 Iulie – Decembrie 2013

1−=πie

Page 4: format .pdf, 3.6 MB

.

Page 5: format .pdf, 3.6 MB

Profesorul Constantin Corduneanula aniversarea a 85 de ani

Constantin Corduneanu este bine-cunoscut, apreciat si se bucura de un deosebitprestigiu ın lumea academica matematicagratie prodigioasei sale activitati stiintifice, di-dactice si prezentei active ın manifestarile co-munitatii matematice internationale: confe-rinte, simpozioane si congrese.

S-a nascut ın Iasi la 26 iulie 1928, acopilarit si a parcurs clasele primare ın satulPotangeni, com. Movileni, jud. Iasi, sat ıncare parintii sai erau ınvatatori. Studiile se-cundare le-a ınceput ın 1940 la Liceul Militardin Iasi si le-a terminat dupa Al Doilea RazboiMondial, ın 1947, la Predeal, unde liceul semutase ıntre timp. Ultimele doua clase le-a promovat ıntr-un singur an, regulamentulscolar permitand acest lucru. A fost colabo-rator al Gazetei Matematice si a participat laconcursurile organizate de aceasta; ın ultimul an de liceu a fost premiat la ConcursulGazetei Matematice, clasandu-se pe locul ıntai.

Despre aceasta scoala si dascalii sai, are cuvinte de adanca recunostinta: Am avutsansa de a studia ın acea scoala de elita, unde profesorul de istorie era doctor laOxford, cel de chimie era de asemenea doctor ın chimie si inginer chimist, cel degeografie era simultan si asistent universitar, cel de franceza era doctor ın filologie, custagiu de specializare ın Franta, devenind apoi seful catedrei de limba franceza la Uni-versitatea din Iasi. Profesorul de matematica fusese si cadru universitar (conferentiaruniversitar). Profesorii sai l-au sfatuit ca, ın noile conditii politice din tara, sa renuntela o cariera militara si sa continue cu studii de matematica.

S-a ınscris la Facultatea de Stiinte din Iasi, sectia matematica, la care a promovatprimii doi ani de studii ıntr-un singur an. A absolvit facultatea ın 1950, iar ın ultimulan de studii a fost retinut ca preparator. Fiind un student eminent, scurtand perioadade studii prin comasarea unor ani, sprijinit de profesorii sai ın momente critice alevietii, a reusit sa scape de prigoana contra celor cu origine nu tocmai ,,sanatoasa”.

Cu aceeasi adanca recunostinta se refera si la perioada studiilor universitare: Inanii mei de studentie am avut sansa extraordinara de a nimeri ın scoala de matematicaa lui Alexandru Myller.

A ramas pentru totdeauna atasat de Iasul tineretii sale, de Seminarul Matematicın care s-a format ca specialist, de atmosfera de ınalta tinuta stiintifica a acestuia si deprofesorii sai. In anul 2010, la conferinta dedicata jubileului centenarului SeminaruluiMatematic ,,A. Myller” va expune ın plen un istoric al acestei institutii.

89

Page 6: format .pdf, 3.6 MB

Datorita talentului, pasiunii si daruirii sale, parcurge ın putini ani treptele ierarhieididactice: ın 1955 este lector universitar titular, ın 1962 conferentiar prin concurs, iarın anul 1968 este promovat ca profesor titular. A sustinut ın 1956 la Universitateadin Iasi teza de doctorat Probleme globale pentru ecuatiile diferentiale de ordinul I siII sub conducerea prof.dr. Ilie Popa si ın fata unei comisii formate din academicieniiM. Nicolescu, Gr. Moisil si N. Teodorescu. In aceasta perioada a functionat si laInstitutul Politehnic din Iasi (1950-1954) si a avut norma de cercetare la Institutulde Matematica al Academiei-filiala Iasi.

In perioada 1964-1967 a contribuit la organizarea activitatii ın Institutul Pedagogicdin Suceava (acum Universitatea ,,Stefan cel Mare”), fiind rector ın ultimul an. Inanii academici 1967-68 si 1973-74 a fost profesor vizitator la Universitatea de Stat dinRhode Island (SUA). Intre anii 1968-1972 a fost decan al Facultatii de Matematica,iar ıntre 1972-1973 si 1974-1975 a avut functia de prorector al Universitatii din Iasi.

Printr-un decret din primavara anului 1975, Institutele de Matematica cu sediileın Bucuresti, Iasi si Cluj Napoca treceau de sub controlul Academiei Romane ınsubordinea universitatilor, care aveau obligatia sa distribuie la alte unitati socialistepe toti cercetatorii ıncadrati ın ele. In acel moment, profesorul C. Corduneanu eraprorector cu problemele cercetarii stiintifice si studiilor doctorale si, ın consecinta,i-a revenit ca obligatie de serviciu aplicarea decretului. Mai tarziu, ıntr-un interviu,profesorul ısi aminteste: Majoritatea celor transferati, prin aceasta ingrata sarcina deserviciu, erau fostii mei studenti si erau dintre cei mai buni!

Desfiintarea Institutului de Matematica, cat si climatul politic tensionat din taranoastra, l-au decis sa emigreze. In 1977 este invitat sa expuna o serie de conferinte laCentrul International de Fizica Teoretica (UNESCO), Trieste (Italia). Folosind acestprilej, emigreaza la ınceputul anului 1978 ın SUA. Dupa scurte sederi ca profesorvizitator la Universitatea de Stat din Rhode Island (ian.-iun. 1978) si la Universitateade Stat Tennesse (Knoxville, 1978-1979), ocupa prin concurs o pozitie de profesor laUniversitatea de Stat din Texas, orasul Arlington, pozitie din care se va pensiona ın1996, cu titlul de Profesor Emerit.

Activitatea didactica si de ındrumare a profesorului C. Corduneanu este bogatasi de lunga durata: 47 de ani, dintre care ın Romania 26 de ani si jumatate. Acontribuit la pregatirea a circa 4000 de studenti ın Romania si 3000 ın SUA, iarnumarul tinerilor care si-au trecut doctoratul sub conducerea sa este de peste 20. Afost si este ınconjurat cu recunostinta, respect si iubire de cei carora cu generozitatele-a oferit idei, ındrumari, ıncurajari si sfaturi ın modul cel mai simplu si firesc.

Activitatea stiintifica a prof. C. Corduneanu s-a desfasurat fara ıntreruperesi continua si ın prezent. Rezultatele obtinute i-au adus recunoasterea comunitatiiacademice internationale a matematicienilor. A abordat variate domenii de cercetare:teoria globala a sistemelor diferentiale, teoria admisibilitatii pentru sisteme de ecuatiidiferentiale si integrale, teoria ecuatiilor cu operatori Voltera abstracti (cauzali), teo-ria oscilatiilor si undelor (cazul aproape-periodic), cat si aplicatiile acestor teorii lateoria sistemelor (generale sau de reglare automata). La Iasi a ınfiintat Seminarul deteoria calitativa a ecuatiilor diferentiale si integrale, fiind initiatorul acestei directiide cercetare ın acest centru universitar. Ca o apreciere a cercetarilor sale, a primitmulte distinctii si premii. In Romania a fost distins cu Premiul Ministerului Educatiei

90

Page 7: format .pdf, 3.6 MB

(1963), cu Premiul ,,Gh. Lazar” al Academiei Romane (1965), iar mai tarziu Primariamunicipiului Iasi i-a conferit Premiul ,,Vasile Pogor” (2002). Universitatea de Statdin Texas i-a atribuit Distinguished Research Award (1995), Uniunea Matematicie-nilor Cehi i-a acordat Medalia de Merit ın Matemamatici (2001) etc.

In anul 1974 a fost ales membru corespondent al Academiei Romane (repus ındrepturi ca membru corespondent al Academiei ın 1990).

Activitatea didactica si de cercetare stiintifica este doar o parte din multele directiiın care prof. C. Corduneanu si-a adus aportul; profilul prezentei reviste nu permitedecat o ınsirare pe scurt: autor al unor monografii aparute ın cele mai prestigioaseedituri din lume, membru ın redactiile unor importante reviste din tara si strainatate,membru al unui numar mare de societati si asociatii de matematica, a conferentiatla numeroase universitati si institute de cercetare din toate continentele (cu exceptiaAustraliei), participant activ la conferinte si congrese de pretutindeni s. a. Extragemdintr-un interviu un bilant propriu foarte sumar: Sunt bucuros ca pe parcursul carieriimele stiintifice am reusit sa scriu sase carti, aparute ın 12 editii, toate de circulatieinternationala. Am tinut conferinte ın peste 60 de universitati si academii din diferitetari, am participat la peste o suta de conferinte si congrese de specialitate. Tot atateaprilejuri de satisfactie personala.

In 1981 prof. C. Corduneanu a fondat revista Libertas Mathematica sub egidaAcademiei Romano-Americane de Stiinte si Arte (ARA). Tot el a editat si tiparitrevista timp de 31 de ani, pana ın anul 2012, cand prof. Vasile Staicu (Portugalia)a preluat publicarea acesteia. Revista ısi propune sa capteze energiile matematicie-nilor romani din diaspora si sa introduca rezulatatele lor remarcabile ın mediul aca-demic international. Mai multe volume au fost dedicate (ca volume memoriale sauaniversare) unor distinsi matematicieni: Grigore Moisil, Tiberiu Popoviciu, NicholasGeorgescu Roegen, Radu Rosca s. a. Rubrica Miscellanea include o lista cu numelesi adresele matematicienilor de origine romana din diaspora (ajunsa acum la circa500 nume) menita sa faciliteze contactele dintre acestia. Mentionam si faptul ca ınperioada 1995-1998 prof. C. Corduneanu a fost presedinte al ARA, iar din 1998 estePresedinte de Onoare al ARA.

Dupa evenimentele din 1989, prof. C. Corduneanu revine ın mod constant ınRomania, de cateva ori pe an, la locuinta proprie redobandita ıntre timp. Aici ısicontinua activitatea sau ısi propune noi proiecte: fie ca este vorba de o conferintaın Iasi sau alt centru universitar din tara, fie ca se ıngrijeste de aparitia unui numarde Libertas Mathematica, fie ca pregateste aparitia unei noi carti etc. Universitatea,,Ovidius” din Constanta si Universitatea ,,Al.I. Cuza” din Iasi i-au conferit titlulde Doctor Honoris Causa ın 1994, Universitatea ,,Transilvania” din Brasov ın 1999,iar Universitatea ,,Stefan cel Mare” din Suceava ın 2003. Este Honorary Doctor alUniversitatii din Ecaterinburg (Rusia), 2010.

Cu adanca recunostinta pentru aportul adus ın cresterea prestigiului internationalal matematicii romanesti si cu admiratie fata de exemplul personal de demnitateumana, uram prof. Constantin Corduneanu multi ani de viata fericita, cu sanatate,putere deplina de creatie si multe ımpliniri!

Prof. univ. Temistocle BIRSAN

91

Page 8: format .pdf, 3.6 MB

Grupuri finite cu proprietatea (P)

Marius Tarnauceanu 1

Abstract. In this paper we characterize the finite groups that have the property (P).Keywords: finite abelian groups, automorphisms.

MSC 2010: 20K01, 20K30.

1. Introducere. Problema 2 de la faza judeteana a Olimpiadei de Matematica2013, clasa a XII-a, are urmatorul enunt:

Problema. Un grup (G, ·) are proprietatea (P) daca

(P ) ∀f ∈ Aut(G), ∃g, h ∈ Aut(G) astfel ıncat f(x) = g(x) · h(x), ∀x ∈ G.

Sa se arate ca:

(a) Orice grup cu proprietatea (P) este abelian.

(b) Orice grup abelian finit de ordin impar are proprietatea (P).

(c) Niciun grup finit de ordin 4n+ 2, n ∈ N, nu are proprietatea (P).

Solutia prezentata ın barem se ıncheie cu o remarca interesanta, anume ca existagrupuri de ordin 4n care au proprietatea (P) - e.g., grupul lui Klein Z2 × Z2 - sigrupuri de ordin 4n care nu o au - e.g., grupul aditiv Z4.

Cerintele problemei, ımpreuna cu remarca anterioara, conduc la ıntrebarea fireasca:

Care sunt grupurile ce satisfac proprietatea (P)?

Pentru cazul finit suntem ın masura sa dam un raspuns la aceasta ıntrebare. Maiprecis, vom proba urmatorul rezultat.

Teorema 1.1. Un grup finit G are proprietatea (P) daca si numai daca G ∼=G1 ×G2, unde G1 este un 2-grup abelian de tipul

Z2α1 × Z2α2 × · · · × Z2αk cu |j | αj = αi| ≥ 2, ∀ i = 1, 2, ..., k,

iar G2 este un grup abelian de ordin impar.

In particular, putem decide care din grupurile finite de ordin 4n cu n impar auproprietatea (P).

Corolarul 1.2. Grupurile finite de ordin 4n, n ≡ 1 (mod 2), care au proprietatea(P) sunt de tipul

Z2 × Z2 ×G,

1Lector dr., Universitatea ,,Al.I. Cuza” din Iasi, [email protected]

92

Page 9: format .pdf, 3.6 MB

iar cele care nu au aceasta proprietate sunt de tipul

Z4 ×G,

unde G este un grup abelian de ordin n.

De asemenea, mentionam ca exista si grupuri infinite ce satisfac proprietatea (P)(spre exemplu (Q,+)), o clasificare a acestora fiind mult mai greu de realizat.

2. Preliminarii. Principalul rezultat ca va fi utilizat este teorema de structuraa grupurilor abeliene finite (a se vedea, spre exemplu, [3]).

Teorema 2.1. Fie G un grup abelian finit. Atunci exista si sunt unice numerelenaturale m, d1, d2, ..., dm, astfel ıncat

G ∼= Zd1 × Zd2 × · · · × Zdm ,

unde di > 1,∀ i = 1, 2, ...,m, si d1|d2|...|dm.

Cu notatiile din Teorema 2.1, consideram descompunerile ın produse de factoriprimi ale numerelor d1, d2, ..., dm:

di = pαi11 pαi2

2 · · · pαik

k , i = 1, 2, ...,m.

Tinand cont ca pentru fiecare i are loc izomorfismul Zdi∼= Zp

αi11

× Zpαi22

× · · · ×Zp

αikk

,obtinem

(1) G ∼= G1 ×G2 × · · · ×Gk ,

unde

Gj = Zpα1jj

× Zpα2jj

× · · · × Zpαmjj

si α1j ≤ α2j ≤ ... ≤ αmj , ∀ j = 1, 2, ..., k.

Cu alte cuvinte, orice grup abelian finit este produs direct (sau, echivalent, sumadirecta) de p-grupuri abeliene.

Urmatoarea teorema arata ca studiul grupului automorfismelor unui grup abelianfinit se reduce la p-grupuri abeliene (a se vedea, spre exemplu, Lema 2.1 din [2]).

Teorema 2.2. Fie H si K doua grupuri finite de ordine relativ prime. AtunciAut(H ×K) ∼= Aut(H)×Aut(K).In particular, daca G este un grup abelian finit detipul (1), atunci

(2) Aut(G) ∼= Aut(G1)×Aut(G2)× · · · ×Aut(Gk).

Un rezultat mai puternic decat precedentul ıl constituie Teorema 3.2 din [1].Aceasta indica forma automorfismelor unui produs direct de grupuri finite ce nu aufactori directi comuni.

93

Page 10: format .pdf, 3.6 MB

Teorema 2.3. Fie H si K doua grupuri finite fara factori directi comuni. AtunciAut(H ×K) este izomorf cu grupul multiplicativ§

f uv g

| f∈Aut(H), g∈Aut(K), u∈Hom(K,Z(H)), v∈Hom(H,Z(K))

ª.

In particular, daca grupurile H si K sunt abeliene, atunci Aut(H ×K) este izomorfcu grupul multiplicativ§

f uv g

| f∈Aut(H), g∈Aut(K), u∈Hom(K,H), v∈Hom(H,K)

ª.

Incheiem acest paragraf cu o observatie simpla, dar extrem de utila.

Observatia 2.4. Un grup abelian (G,+) satisface proprietatea (P) daca si numaidaca automorfismul identic 1G poate fi scris sub forma

(3) 1G = g + h cu g, h ∈ Aut(G).

Intr-adevar, daca (3) are loc, atunci pentru orice automorfism f al lui G avem f =f 1G = f g + f h si f g, f h ∈ Aut(G).

Putem acum proba principalul nostru rezultat. Mentionam ca nu am inclus ındemonstratie verificarea proprietatilor (a)-(c), pentru care poate fi consultata solutiaproblemei considerate.

3. Demonstratia Teoremei 1.1. Presupunem mai ıntai ca G satisface propri-etatea (P). Atunci el este abelian si, conform cu (1), admite o descompunere de tipulG ∼= G1 × G2,unde G1 este un 2-grup abelian, iar G2 este un grup abelian de ordinimpar. Din Teorema 2.2 deducem ca

Aut(G) ∼= Aut(G1)×Aut(G2),

ceea ce arata ca G1 si G2 satisfac, de asemenea, proprietatea (P). Este suficient saindicam structura lui G1. Avem

G1∼= Z2α1 × Z2α2 × · · · × Z2αk , unde 1 ≤ α1 ≤ α2 ≤ ... ≤ αk.

Daca, prin absurd, exista i ∈ 1, 2, ..., k astfel ıncat αj = αi, ∀ j=1, 2, ..., i−1,i+1, ..., k, atunci G1 poate fi scris sub forma G1 = H × K,unde H ∼= Z2αi si K ∼=Z2α1 × · · · ×Z2αi−1 ×Z2αi+1 × · · · ×Z2αk . In plus, remarcam ca H si K nu au factoridirecti comuni, asadar

Aut(G1)∼=§

f uv g

| f∈Aut(H), g∈Aut(K), u∈Hom(K,H), v∈Hom(H,K)

ªdin Teorema 2.3. Atunci

1G1 =

1H 00 1K

=

f1 u1

v1 g1

+

f2 u2

v2 g2

,

94

Page 11: format .pdf, 3.6 MB

unde fi ∈ Aut(H), gi ∈ Aut(K), ui ∈ Hom(K,H), vi ∈ Hom(H,K), i = 1, 2. Rezultaca 1H = f1 + f2, ceea ce constituie o contradictie (automorfismele lui H sunt de tipulx 7→ qx cu q impar, suma a doua astfel de automorfisme nefiind un automorfism).

Reciproc, este suficient sa aratam ca un 2-grup abelian de tipul

G = Z2α1 × Z2α2 × · · · × Z2αk cu |j | αj = αi| ≥ 2, ∀ i = 1, 2, ..., k,

sau, echivalent, de tipul

G = (Z2β1 )r1 × (Z2β2 )

r2 × · · · × (Z2βs )rs cu ri ≥ 2, ∀ i = 1, 2, ..., s,

are proprietatea (P). De asemenea, conform Observatiei 2.4, ne putem reduce la cazuls = 1. Avem astfel de probat ca pentru un 2-grup abelian

G = (Z2β )r= Z2β × Z2β × · · · × Z2β| z

r ori

cu r ≥ 2

automorfismul identic 1G poate fi scris ca suma a doua automorfisme ale luiG. Tinandcont ca automorfismele grupului abelian G coincid cu automorfismele Z2β -modulului(Z2β )

r, iar acestea se identifica cu matricele inversabile de ordin r peste Z2β , trebuie

sa aratam ca

(4) ∃Ar, Br ∈ GLr(Z2β ) astfel ıncat Ir = Ar +Br.

Vom verifica aceasta afirmatie prin inductie dupa r. Pentru r = 2 consideram

A2 =

1 1

1 0

si B2 =

0 −1

−1 1

,

iar pentru r = 3 consideram

A3 =

1 1 1

1 1 0

1 0 0

si B3 =

0 −1−1

−1 0 0

−1 0 1

.

Fie r ≥ 4. Presupunem ca (4) este adevarata pentru orice r′ satisfacand 2 ≤ r′ < rsi o verificam pentru r. Avem

Ir = Ar +Br,

unde matricele

Ar =

Ar−2 00 A2

si Br =

Br−2 00 B2

sunt ambele inversabile, ceea ce ıncheie demonstratia.

Bibliografie

1. J.N.S. Bidwell, M.J. Curran, D.J. McCaughan – Automorphisms of directproducts of finite groups, Arch. Math., 86 (2006), 481-489.

2. C. Hillar, D. Rhea – Automorphisms of an abelian p-group, Amer. Math. Monthly,114 (2007), 917-922.

3. I.D. Ion, N. Radu – Algebra, Editura Didactica si Pedagogica, Bucuresti, 1991.

95

Page 12: format .pdf, 3.6 MB

Cercurile mixtliniare adjuncte ınscriseasociate unui triunghi

Ion PATRASCU 1

Abstract. In this note there are defined the inscribed adjoint mixtlinear circles associated to atriangle. The main results are Theorem 1 and 2. E.g., the triangle determined by the centers of theinscribed adjoint mixtlinear circles and the triangle tangential to the given triangle are orthological.

Keywords: inscribed adjoint mixtlinear circles, Apollonius circle, Brocard′s angle, orthologicaltriangle.

MSC 2010: 67G40.

In acest articol definim cercurile mixtliniare adjuncte asociate unui triunghi sievidentiem cateva proprietati ale lor.

Definitie. Numim cerc mixtilinar adjunct ınscris al unui triunghi dat un cerctangent (interior) cercului circumscris triunghiului ıntr-un varf al sau si tangent lalatura opusa varfului considerat.

Observatie. Evident, pentru orice triunghi, avem trei cercuri mixtliniare ad-juncte ınscrise. Mai observam ca avem si trei cercuri mixtliniare adjuncte exınscrise,care sunt tangente exterior cercului circumscris triunghiului dat si ındeplinesc celelalteconditii din definitia de mai sus. Ne vom ocupa numai de cercurile mixtliniare ınscrise;le vom numi, dupa varful triunghilui ABC prin care trec, A-mixtliniar adjunct ınscrisetc.

Propozitia 1. Punctul de tangenta cu BC al cercului A-mixtliniar adjunct ınscriseste piciorul bisectoarei interioare a unghiului bA.

Demonstratie. Notam cu La centrul cercului A-mixtliniar adjunct ınscris sicu D contactul acestuia cu latura BC. Fie S intersectia tangentei ın varful A la

.

.

A

B C

O

La

D

M N

S

cercul circumscris triunghiuluiABC cu latura BC (ın figura

m(ÒB) > m(ÒC)). In mod obisnuit,cu 0 notam centrul cercului cir-cumscris.

In OAC avem: m(ÕOAC) =

90 − m(ÒB). Apoi, m(ÕOAD) =

m(ÖASLa) =1

2m(ÕASC) =

1

4[m(÷AC)−m(÷AB)] =

1

2[m(ÒB)−

m(ÒC)].Ca urmare, combinand rezul-

tatele obtinute, m(ÕCAD) = m(ÕOAC)+m(ÕOAD) = 90−m(ÒB)+1

2[m(ÒB)−m(ÒC)] =

1

2m( bA), adica D este piciorul bisectoarei interioare a unghiului bA.1Profesor, Colegiul National ,,Fratii Buzesti”, Craiova

96

Page 13: format .pdf, 3.6 MB

Analog se demonstreaza proprietatea ın cazul triunghiurilor obtuzunghice. Dacatriunghiul ABC este isoscel sau echilateral, demonstratia este imediata.

Propozitia 2. Cercul A-mixtliniar adjunct ınscris intersecteaza laturile AB siAC ın extremitatile unei coarde paralele cu BC.

Demonstratie. Notam cu M si N punctele de intersectie a cercului A-mixtliniar

adjunct ınscris cu AB, respectiv AC. Avem: m(ÕCDN) = m(ÕDAN) =1

2m( bA) si, pe

de alta parte, m(ÖDNM) = m(ÖMAD) =1

2m( bA). Rezulta ca ÕCDN ≡ ÖDNM , ceea ce

implica MN∥BC.

Observatie. Afirmatia MN∥BC decurge si din omotetia cercurilor A-mixtliniaradjunct ınscris si circumscris triunghiului, centrul de omotetie fiind varful A.

Propozitie. Raza rA a cercului A-mixtliniar ınscris este data de formula:

(1) rA =4p(p− a)R

(b+ c)2.

Demonstratie. Solutia I. Aplicand teorema sinusurilor ın AMN si ABC,obtinem: MN = 2rA sinA si a = 2R sinA, de unde

rA =R

a·MN.

Deoarece AMN ∼ ABC, avem ca MN =a

bAN . Ca urmare,

rA =R

bAN.

Puterea punctului C fata de cercul A-mixtliniar adjunct ınscris se scrie: CN · CA =

CD2 si cum CD =ab

b+ c(consecinta a teoremei bisectoarei), rezulta ca CN =

a2b

(b+ c)2. Cum AN = b − CN , decurge ca AN =

4p(p− a)b

(b+ c)2. Inlocuind aceasta

expresie gasita, vom avea

rA =R

b· 4p(p− a)b

(b+ c)2=

4p(p− a)R

(b+ c)2

si, deci, formula dorita.

Solutia II. In LaAD avem: LaA = LaD = rA, AD = la (=lungimea bisectoarei

din A) si m(ÖLaAD) =1

2[m(ÒB)−m(ÒC)]. Atunci,

rA =AD

2 cosÖLaAD=

la

2 cos B−C2

.

97

Page 14: format .pdf, 3.6 MB

Folosind formule ca la =2bc

b+ ccos

A

2, cos

A

2=

rp(p− a)

bcetc., dupa efectuarea cal-

culelor se obtine (1).

Observatie. Pentru raza rA retinem si formulele:

(2) rA =bc

b+ c·

cos A2

cos B−C2

= R

1−

a

b+ c

2.

Vom aminti acum cateva notiuni si rezultate necesare ın continuare.

..

A

B C

ω

ω

ω

Se numeste cerc adjunct alunui triunghi ABC un cerc cetrece prin doua varfuri ale sale siın unul dintre aceste varfuri estetangent laturii respective. Un tri-unghi are sase cercuri adjuncte.Vom nota cu CA cercul adjunctce contine varfurile C si A si estetangent ın A laturii AB. Cer-curile adjuncte AB, BC, CA auun punct comun Ω; analog, cer-curile BA, CB, AC au un punctcomun Ω′. Punctele Ω si Ω′ senumesc punctele lui Brocard: Ω este punctul direct al lui Brocard, iar Ω′ este punctul

retrograd. Punctele Ω si Ω′ sunt izogonal conjugate: ÕΩAB = ÕΩBC = ÕΩCA = ω siÕΩ′AC = ÖΩ′CB = ÖΩ′BA = ω. Unghiul ω se numeste unghiul lui Brocard. Cititorulpoate gasi alte informatii ın [1].

Sa observam legatura urmatoare: cercul A-mixtliniar adjunct ınscris asociat tri-unghiului ABC este cerc adjunct pentru triunghiurile ADB si ADC. Ca urmare,avem:

Propozitia 4. Intr-un triunghi ABC ın care D este piciorul bisectoarei interioarea unghiului bA, punctele A,D si punctele directe ale lui Brocard corespunzatoare tri-unghiurilor ADB si ADC sunt conciclice.

Urmatoarele doua teoreme indica proprietati remarcabile ale cercurilor mixtliniareadjuncte ınscrise.

Teorema 1. Triunghiul LaLbLc determinat de centrele cercurilor mixtliniareadjuncte ınscrise si triunghiul tangential TaTbTc sunt ortologice. Centrele lor de or-tologie sunt O si centrul radical al cercurilor mixtliniare adjuncte ınscrise.

Demonstratie. Perpendicularele duse din La, Lb, Lc pe laturile corespunzatoareale triunghiului tangential contin razele OA,OB, respectiv OC ale cercului circum-scris. Prin urmare, O este centrul de ortologie al triunghiurilor LaLbLc si TaTbTc.

98

Page 15: format .pdf, 3.6 MB

Tangentele duse din Ta la cercul circumscris, TaB si TaC, sunt egale. Ele sunt

.

A

B C

O

Tc

Ta

Tb

.

.

.

La

Lb

Lc

tangente si la cercurile B-mixtliniaradjunct ınscris, respectiv C-mixtliniaradjunct ınscris. In consecinta, Ta areputeri egale fata de aceste cercuri sica atare apartine axei radicale a lor.Cum axa radicala a doua cercuri esteperpendiculara pe linia centrelor aces-tora, urmeaza ca perpendiculara dinTa pe LaLc trece prin centrul radi-cal al cercurilor mixtliniare adjuncteınscrise, deci acest punct este centrude ortologie al triunghiurilor LaLbLc

si TaTbTc.

Fiind date trei cercuri de cen-tre diferite, se numeste cerc Apollo-nius al lor fiecare dintre cercurile tan-gente celor trei cercuri date. Evi-dent, cercul circumscris triunghuiuluiABC este cerc Apollonius al cercurilormixtliniare adjuncte ınscrise asociatetriunghiului.

Teorema 2. Cercul Apollonius tangent interior cercurilor mixtliniare adjuncteınscrise are cu acestea punctele de tangenta T1, T2, respectiv T3. Dreptele AT1, BT2

si CT3 sunt concurente.

Demonstratie. Demonstratia face apel la teorema lui D′Alembert (de ex., [2]):trei cercuri necongruente si ale caror centre nu sunt coliniare au cele sase centre deomotetie situate cate trei pe patru drepte.

Intr-adevar, A este centrul de omotetie directa a cercului circumscris (O) si acelui A-mixtliniar adjunct ınscris (La); T1 este centrul de omotetie directa a cerculuiApollonius tangent interior cercurilor mixtliniare adjuncte ınscrise si a cercului (La),iar J este centrul de omotetie directa a cercului Apollonius si a celui circumscris (O).Conform teoremei lui D′Alembert, rezulta ca punctele A, J, T1 sunt coliniare. Analogse arata ca B, J, T2 sunt coliniare si C, J, T3 sunt coliniare. In consecinta, punctul Jeste punctul de concurenta a dreptelor AT1, BT2 si CT3.

Bibliografie

1. R.A. Johnson – Advanced Euclidean Geometry, Dover Publications, Inc., Mineola,New York, 2007.

2. N.N. Mihaileanu – Lectii complementare de geometrie, Editura Didactica si Peda-gogica, Bucuresti, 1976.

99

Page 16: format .pdf, 3.6 MB

Cateva aplicatii ale inegalitatiiIonescu - Weitzenbock

Dumitru M. BATINETU-GIURGIU 1, Neculai STANCIU 2

Abstract. The purpose of the present note is to establish some applications of the inequalityIonescu- Weitzenbock.

Keywords: Ionescu-Weitzenbock inequality, Bergstrom′s inequality, Radon′s inequality.

MSC 2010: 51M16.

ION IONESCU (BIZET) (1870-1946) s-a nascut la 22 noiembrie/4 decembrie 1870 ın satul

Stoienoaia, comuna Creata-Lesile, judetul Ilfov. In anul 1889 a reusit primul la Scoala Nationalade Poduri si Sosele din Bucuresti, unde obtine diploma de inginer cu nota18,42 (nota maxima fiind 20).

In anul 1895 ınfiinteaza Gazeta Matematica ımpreuna cu Victor Ba-laban, Vasile Cristescu, Mihail Roco, Ion Zotta, Emanoil Davidescu,Mauriciu Kinbaum, Nicolae Nicolescu, Tancred Constantinescu si AndreiIoachimescu. In 31 august 1909, la o sedinta a redactiei Gazetei Mate-matice, tinuta la via lui Ion Ionescu se decide ınfiintarea Societatii GazetaMatematica. A fost unul din ”Stalpii Gazetei Matematice”, alaturi deVasile Cristescu, Andrei Ioachimescu si Gheorghe Titeica.

Din 1921 se dedica ınvatamantului de la Scoala Politehnica din Bu-curesti, profesor de baza al acestei institutii.

A condus lucrarile de proiectare si constructie a numeroase poduri,printre care podul peste Borcea (prima portiune a podului peste Dunare), podul de la Bobolia pe

Valea Prahovei etc. In 1900, la Giurgiu se impunea realizarea unui pod peste canalul Sf. Gheorghe,care sa lege portul situat pe malul Dunarii cu celelalte regiuni printr-o legatura dubla sosea-caleferata. Coordoneaza proiectarea si constructia podului peste un brat al Dunarii din bazinul de laGiurgiu. Conditiile de teren faceau aproape imposibila aceasta realizare, dar Ion Ionescu a gasito solutie exceptionala; podul dublu, cale ferata si sosea, are o forma curba, unica la vremea aceeaın lume. Prin aceasta lucrare, inaugurata ın 1905, Ion Ionescu-Bizet a ramas ın istoria orasuluiGiurgiu. Podul este folosit si ın prezent, dupa o utilizare de peste 100 de ani, si poarta numele dePodul Bizet.

A elaborat proiectul pentru santierul naval de la Turnu-Severin. Este autorul unui studiu dedeviere spre Prut a apelor Siretului ın vederea constructiei unei centrale hidroelectrice si a trans-formarii Prutului ıntr-un canal navigabil ıntre Galati si Iasi. A executat harta hidrografica a bazinuluiDunarii.

Pentru meritele sale, a fost ales ın 1919 membru corespondent al Academiei Romane, sectiastiintifica. Alaturi de Gheorghe Lazar, Gheorghe Asachi, Ion Heliade Radulescu, Spiru Haret, Pe-trache Poenaru si Gheorghe Duca, a fost ctitor al ınvatamantului tehnic romanesc.

A ıncetat din viata la 17 septembrie 1946. A lasat prin testament Societatii Gazeta Matematica

(astazi numita SSMR), casa sa din Str. Rasuri, nr. 25.

In [1] am aratat ca Ion Ionescu a publicat cu 22 de ani ınaintea lui RolandWeitzenbock inegalitatea a2 + b2 + c2 ≥ 4S

√3, adevarata ın orice triunghi. Intr-

adevar, ın Gazeta Matematica vol.III, nr. 2 (oct. 1897), p. 52, sub semnatura lui IonIonescu apare problema:

1Profesor, Colegiul National ,,Matei Basarab”, Bucuresti2Profesor, Scoala Generala ,,George Emil Palade”, Buzau

100

Page 17: format .pdf, 3.6 MB

273. Sa se arate ca nu exista nici un triunghiu pentru care inegalitatea

4S√3 > a2 + b2 + c2

sa fie satisfacuta.

Abia ın 1919, ın articolul Uber eine Ungleichung in der Dreiecksgeometrie publicatın revista Mathematische Zeitung, vol. 5, nr.1-2, pp. 137-146, apare inegalitatea careın mod curent poarta numele lui R. Weitzenbock.

Aceasta inegalitate a fost numita de noi ın [1] inegalitatea Ionescu-Weitzenbock.

In prezenta nota vom da cateva aplicatii ale sale.

Aplicatia 1. In orice triunghi ABC, are loc inegalitatea:

(1)a3

b ·R+ c · r+

b3

c ·R+ a · r+

c3

a ·R+ b · r≥ 4

√3

R+ rS.

Solutie. Avem, aplicand inegalitatea lui Bergstrom:

U =Xciclic

a3

b ·R+ c · r=Xciclic

a4

abR+ acr≥ 2

Xciclic

(a2)2

(a2 + b2)R+ (a2 + c2)r

≥ 2

(P

ciclic

a2)2Pciclic

R(a2 + b2) +P

ciclic

r(a2 + c2)= 2

(P

ciclic

a2)2

2(R+ r)P

ciclic

a2=

Pciclic

a2

R+ r.

Mai departe, aplicam inegalitatea Ionescu- Weitzenbock, adicaP

ciclic

a2 ≥ 4S√3, si

rezulta ca U ≥ 4S√3

R+r , ceea ce era de demonstrat.

In acelasi fel se stabileste si urmatorul rezultat:

Aplicatia 2. In orice triunghi ABC, are loc inegalitatea:

(2)m3

a

R ·mb + r ·mc+

m3b

R ·mc + r ·ma+

m3c

R ·ma + r ·mb≥ 3

√3

R+ rS.

Solutie. Avem, utilizand inegalitatea lui Bergstrom si formula bine-cunoscuta

m2a +m2

b +m2c =

3

4(a2 + b2 + c2) :

V =Xciclic

m3a

R ·mb + r ·mc=Xciclic

(m2a)

2

R ·ma ·mb + r ·ma ·mc

≤ 2Xciclic

(m2a)

2

R(m2a +m2

b) + r(m2a +m2

c)≥ 2

(P

ciclic

m2a)

2

RP

ciclic

(m2a +m2

b) + rP

ciclic

(m2a +m2

c)=

=

2(P

ciclic

m2a)

2

2RP

ciclic

m2a + 2r

Pciclic

m2a

=

Pciclic

m2a

R+ r=

3

4· a

2 + b2 + c2

R+ r,

101

Page 18: format .pdf, 3.6 MB

unde folosim inegalitatea Ionescu-Weitzenbock, i.e. a2 + b2 + c2 ≥ 4S√3, si deducem

ca:

V ≥ 3

4· 1

R+ r· 4S

√3 =

3√3

R+ rS.

Urmatoarele generalizari ale inegalitatilor (1) si (2) se obtin la fel, dar folosindinegalitatea lui Radon:

Aplicatia 3. Daca x ∈ R+, atunci ın orice triunghi ABC are loc inegalitatea:

(3)ax+2

(b ·R+ c · r)x+

bx+2

(c ·R+ a · r)x+

cx+2

(a ·R+ b · r)x≥ 4

√3

(R+ r)xS.

Aplicatia 4. Daca x ∈ R+, atunci ın orice triunghi ABC are loc inegalitatea:

(4)mx+2

a

(R ·mb + r ·mc)x+

mx+2b

(R ·mc + r ·ma)x+

mx+2c

(R ·ma + r ·mb)x≥ 3

√3

(R+ r)xS.

Bibliografie

1. D.M. Batinetu-Giurgiu, Neculai Stanciu – Inegalitati de tip Ionescu-Weitzenbock,Arch. Math., 86 (2006), 481–489.

Utilizand ca model urmatoarele scheme:

1 2 3

6 6

6

24 3

924

72

5 13

8 15

120

sa se completeze, ınlocuind stelutele cu numere naturale nenule, schemele:

4 7

15

* **

*

*

*

*

*

* **

*

**

*

(Raspuns la pag. 118)

102

Page 19: format .pdf, 3.6 MB

Cateva generalizari si rafinari ale inegalitatiix2 + y2 + z2 > xy + yz + zx

Mihaly BENCZE1

Abstract. In this paper we present some generalizations and refinements for classical inequlityx2 + y2 + z2 > xy + yz + zx.

Keywords: classical inequalities.

MSC 2010: 97H30.

Propozitia 1. Daca x > 0 si n ∈ N, atunci are loc inegalitatea

(1) (x2 + 1)n ≥ (xn − 1)2 + 2nxn,

cu egalitate daca si numai daca x = 1.Demonstratie. Fie functia f : R→R definita prin f(x) = (x2+1)n− (xn− 1)2−

2nxn. Vom scrie f ın doua moduri, dupa cum urmeaza (ın calcule se tine seama de

faptul ca 2nxn =nP

k=0

nk

xn):

f(x) =nX

k=0

n

k

x2(n−k) − (x2n − 2xn + 1)− 2nxn =

=nX

k=0

n

k

(x2(n−k) − xn)− x2n + 2xn − 1 =

n−1Xk=1

n

k

(x2(n−k) − xn);

f(x) =nX

k=0

n

k

x2k − (x2n − 2xn + 1)− 2nxn =

n−1Xk=1

n

k

(x2k − xn).

Prin adunare, obtinem:

2f(x) =n−1Xk=1

n

k

(xn−k − xk)2

si, deci, f(x) ≥ 0, adica inegalitatea (1).

Corolarul 1.1. Daca ak > 0, k = 1,m, atunci pentru orice n ∈ N au locinegalitatile

(2)Xciclic

a21 + a22

2

n

≥Xciclic

(a1a2)n +

1

2n

Xciclic

(an1 − an2 )2 ≥

Xciclic

(a1a2)n,

cu egalitate daca si numai daca a1 = a2 = · · · = am.

Demonstratie. In (1) luam x =a1a2

si obtinem:a21 + a22

2

n

≥ (a1a2)n +

1

2n(an1 − an2 )

2.

1Profesor, Colegiul National ,,Aprily Lajos”, Brasov

103

Page 20: format .pdf, 3.6 MB

Scriind si analoagele acesteia si adunand cele m inegalitati obtinute (convenim caam+1 ≡ a1), deducem prima inegalitate din (2), iar cea de-a doua este evidenta.

Corolarul 1.2. Daca ak > 0, k = 1,m, atunci au loc inegalitatile

(3)mX

k=1

a2k ≥Xciclic

a1a2 +1

2

Xciclic

(a1 − a2)2 ≥

Xciclic

a1a2,

cu egalitate daca si numai daca a1 = a2 = · · · = am.

Observatie. Evident, (3) reprezinta o generalizare si o rafinare a inegalitatiiclasice a21 + a22 + a23 ≥ a1a2 + a2a3 + a3a1.

Propozitia 2. Daca x, y > 0, n ∈ N∗ si fn(x, y) =1

n+ 1(xn + xn−1y + · · · +

xyn−1 + yn), atunci

(4)x+ y

2

n≤ fn(x, y) ≤

xn + yn

2,

cu egalitate daca si numai daca x = y.

Demonstratie. Procedam prin inductie completa atat pentru prima inegalitatedin (4) cat si pentru cea de-a doua.

Pentru n = 1 si n = 2 se obtin inegalitatile adevaratex+ y

2≤ x+ y

2six+ y

2

2≤

1

3(x2 + xy+ y2) (echivalenta cu (x− y)2 ≥ 0). Presupunand ca

x+ y

2

n≤ fn(x, y),

vom arata ca inegalitatea are loc si pentru n+ 1. Intr-adevar, deoarecex+ y

2

n+1

≤x+ y

2

· 1

n+ 1(xn + xn−1y + · · ·+ xyn−1 + yn),

este suficient sa aratam cax+ y

2

· 1

n+ 1(xn+xn−1y+···+xyn−1+yn) ≤ 1

n+ 2(xn+1+xny+···+xyn+yn+1),

ceea ce revine la

2(xny + xn−1y2 + · · ·+ xyn) ≤ n(xn+1 + yn+1).

Aceasta inegalitate rezulta observand mai ıntai ca xn−kyk+1 + xk+1yn−k ≤ xn+1 +yn+1 (echivalenta cu inegalitatea evidenta (xn−k − yn−k)(xk+1 − yk+1) ≥ 0), apoiscriind-o pe aceasta pentru k = 0, 1, · · ·, n−1 si ın final adunand inegalitatile obtinute.Asadar, prima inegalitate din (4) este adevarata.

A doua inegalitate din (4) se verifica imediat pentru n = 1 si n = 2; ıntr-adevar,

avem:x+ y

2≤ x+ y

2si

1

3(x2 + xy+ y2) ≤ x2 + y2

2(echivalenta cu (x− y)2 ≥ 0). Sa

104

Page 21: format .pdf, 3.6 MB

trecem la etapa inductiva. Presupunem ca are are loc fn(x, y) ≤xn + yn

2. Avem:

fn+1(x, y) =1

n+ 2

xxn + xn−1y + · · ·+ xyn−1 + yn

n+ 1

(n+ 1) + yn+1

≤ 1

n+ 2

xxn + yn

2(n+ 1) + yn+1

=

=(n+ 1)xn+1 + (n+ 1)xyn + 2yn+1

2(n+ 2).

In acelasi mod gasim si inegalitatea

fn+1(y, x) ≤(n+ 1)yn+1 + (n+ 1)yxn + 2xn+1

2(n+ 2).

Dupa adunarea lor, obtinem:

2fn+1(x, y) ≤(n+ 3)(xn+1 + yn+1) + (n+ 1)xy(xn−1 + yn−1)

2(n+ 2).

Ramane sa aratam ca

(n+ 3)(xn+1 + yn+1) + (n+ 1)xy(xn−1 + yn−1)

2(n+ 2)≤ xn+1 + yn+1,

care este echivalenta cu xy(xn−1+yn−1) ≤ xn+1+yn+1, deci cu (x−y)(xn−yn) ≥ 0,care este adevarata. Propozitia este complet demonstrata.

Corolarul 2.1. Daca ak > 0, k = 1,m, atunci

(5)mX

k=1

ank ≥Xciclic

fn(a1,a2) ≥Xciclic

a1 + a22

n≥Xciclic

(√a1a2)

n,

pentru orice n ∈ N∗, cu egalitate daca si numai daca a1 = a2 = · · · = am (cuam+1 ≡ a1).

Demonstratie. Observam camX

k=1

a2k ≥Xciclic

an1 + an22

si aplicam Propozitia 2.

Corolarul 2.2. In aceleasi conditii, avem:

(6)mX

k=1

a2k ≥Xciclic

f2(a1,a2) ≥Xciclic

a1 + a22

2≥Xciclic

a1a2.

Observatie. (6) ofera noi rafinari inegalitatii clasice a21+a22+a23 ≥ a1a2+a2a3+a3a1.

Bibliografie

1. Octogon Mathematical Magazine, (1993-2012).

105

Page 22: format .pdf, 3.6 MB

Caracterizarea unor proprietatide perpendicularitate

ın care sunt implicate punctele O, I,H,G,O9

Andi Gabriel BROJBEANU1

Abstract. In this Note, a couple of conditions of perpendicularity for some lines determinedby the vertices of a nonisosceles triangle and the points O, I,H,G,O9 (the center of the nine pointcircle) are characterized. The obtained results are formulated in Propositions 1-4.

Keywords: circumcenter, incenter, orthocenter, centroid, the nine point circle.

MSC 2010: 51M04.

In aceasta Nota avem ın vedere numai triunghiuri neisoscele. Pentru un triunghiABC de acest fel, punctele O, I,H,G si O9 (centrul cercului celor noua puncte) suntdistincte.

Amintim ca punctele O,H,G,O9 sunt coliniare, iar dreapta pe care se afla senumeste dreapta lui Euler. Se stie ca punctul I apartine dreptei lui Euler a unuitriunghi daca si numai daca triunghiul este isoscel. Asadar, ın conditia convenita,punctul I nu apartine dreptei lui Euler a triunghiului. Mai amintim ca HG = 2OGsi ca O9 este mijlocul segmentului HO (a se vedea, de exemplu, [1, 2]).

Intre elementele unui triunghi au loc urmatoarele egalitati binecunoscute:

ab+ bc+ ca = p2 + r2 + 4Rr,(1)

a2 + b2 + c2 = 2(p2 − r2 − 4Rr),(2)

abc = 4Rrp.(3)

Pentru distantele ıntre punctele O, I,H,G,O9 au loc formulele:

OI2 = R2 − 2Rr (Euler),(4)

OH2 = 9R2 + 8Rr + 2r2 − 2p2,(5)

IH2 = 4R2 + 4Rr + 3r2 − p2,(6)

IG2 =1

9(p2 + 5r2 − 16Rr),(7)

IO9 =1

2(R− 2r).(8)

Formula (8) se demonstreaza ın mod obisnuit utilizand teorema medianei ın IOHrelativ la IO9.

Vom folosi ca instrument de lucru urmatoarea

1Elev, cl. a X-a, Colegiul National ,,Constantin Carabella”, Targoviste

106

Page 23: format .pdf, 3.6 MB

Lema. Fie A,B,C,D patru puncte oarecare ın plan. Atunci

AB ⊥ CD ⇔ AC2 +BD2 = AD2 +BC2.

Acest rezultat se stabileste usor cu teorema lui Pitagora si poate fi privit ca oextindere a acesteia (la care se reduce daca unul dintre punctele C sau D coincide cuunul dintre A sau B).

Propozitia 1. Intr-un triunghi ABC neisoscel, avem:1) IO9 ⊥ BC ⇔ 2a = b+ c;2) IG ⊥ BC ⇔ 3a = b+ c.

Demonstratie. 1) Conform Lemei, avem:

IO9 ⊥ BC ⇔ IB2 +O9C2 = IC2 +O9B.

Aplicand de doua ori teorema bisectoarei, se obtin relatiile:

IB2 =ac(p− b)

p, IC2 =

ab(p− c)

p.

Pentru calculul termenilor O9B si O9C vom utiliza teorema medianei. Ca urmare,

IO9 ⊥ BC ⇔ ac(p− b)

p+

R2 + CH2

2− OH2

4=

ab(p− c)

p+

R2 +BH2

2− OH2

4

⇔ BH2 − CH2 =2a

p[c(p− b)− b(p− c)]

⇔ BD2 − CD2 = 2a(c− b)

⇔ c2 − b2 = 2a(c− b)

⇔ 2a = b+ c.

(D noteaza piciorul ınaltimii coborate din varful A).2) Avem:

IG ⊥ BC ⇔ IB2 +GC2 = IC2 +GB2

⇔ ac(p− b)

p+

2

3mc

2

=ab(p− c)

p+

2

3mb

2

⇔ ac+2

9

a2 + b2 − c2

2

= ab+

2

9(c2 + a2 − b2

2)

⇔ a(b− c) =2

9· 32(b2 − c2)

⇔ 3a = b+ c

(mb, mc noteaza lungimile medianelor din B, respectiv C).

Propozitia 2. Intr-un triunghi ABC neisoscel este adevarata afirmatia:

IO9 ⊥ IG ⇔ 2a = b+ c sau 2b = c+ a sau 2c = a+ b.

107

Page 24: format .pdf, 3.6 MB

Demonstratie. Conform teoremei lui Pitagora,

IO9 ⊥ IG ⇔ IO29 + IG2 = O9G

2.

Tinand seama de formulele (5), (7), (8) si faptul ca O9G = O9O − OG =1

2HO −

1

3HO =

1

6HO, rezulta ca

IO9 ⊥ IG ⇔ 1

4(R− 2r)2 +

1

9(p2 + 5r2 − 16Rr) =

1

36(9R2 + 8Rr + 2r2 − 2p2)

⇔ 9r2 − 18Rr + p2 = 0.

Prin calcul si utilizand formulele (1) si (3), avem:

(2p− 3a)(2p− 3b)(2p− 3c) = 8p3 − 12p2(a+ b+ c) + 18p(ab+ bc+ ca)− 27abc =

= 8p3 − 24p3 + 18p(p2 + r2 + 4Rr)− 27 · 4Rrp =

= 2p(9r2 − 18Rr + p2).

Combinand rezultatele precedente, vom obtine:

IO9 ⊥ IG ⇔ (2p− 3a)(2p− 3b)(2p− 3c) = 0

⇔ (b+ c− 2a)(c+ a− 2b)(a+ b− 2c) = 0

⇔ 2a = b+ c sau 2b = c+ a sau 2c = a+ b.

Propozitia 3. Intr-un triunghi ABC neisoscel sunt adevarate afirmatiile:1) IO ⊥ AG ⇔ 2bc = ca+ ab,2) IO9 ⊥ AG ⇔ 2(b− c)2 = (c− a)2 + (a− b)2.

Demonstratie. 1) Utilizand din nou Lema, avem:

IO ⊥ AG ⇔ IA2 +OG2 = IG2 +OA2 ⇔ IA2 +1

9OH2 = IG2 +R2

⇔ bc(p− a)

p+

1

9(9R2 + 8Rr + 2r2 − 2p2) =

1

9(p2 + 5r2 − 16Rr) +R2

⇔ bc− 4Rr +1

9(9R2 + 24Rr − 3r2 − 3p2) = R2

⇔ 3bc = p2 + r2 + 4Rr

⇔ 3bc = ab+ bc+ ca

⇔ 2bc = ca+ ab.

2) Notam cu A1 mijlocul laturii BC. Atunci,

IO9 ⊥ AG ⇔ IO9 ⊥ AA1 ⇔ IA2 +O9A21 = IA2

1 +O9A2

⇔ bc(p− a)

p+

R

2

2

=

IB2 + IC2

2− a2

4

+

AH2 +AO2

2− OH2

4.

108

Page 25: format .pdf, 3.6 MB

Cum AH = 2R cosA, obtinem ca AH2 = 4R2 cos2 A = 4R2(1 − sin2 A) = 4R2 − a2.Tinand seama de (5), (3) si (2), avem:

IO9 ⊥ AG ⇔ bc(p− a)

p+

R2

4=

ac(p− b) + ab(p− c)

2p− a2

4+

5R2 − a2

2− OH2

4

⇔ bc− 4Rr +R2

4=ac+ ab

2− 4Rr − a2

4+

5R2

2− a2

2− 9R2 + 8Rr + 2r2 − 2p2

4

⇔ 4bc = 2ac+ 2ab− 3a2 + 2(p2 − r2 − 4Rr)

⇔ 4bc = 2ac+ 2ab− 3a2 + (a2 + b2 + c2)

⇔ 2(b2 + c2 − 2bc) = (a2 + c2 − 2ac) + (a2 + b2 − 2ab)

⇔ 2(b− c)2 = (c− a)2 + (a− b)2.

Propozitia 4. Intr-un triunghi ABC neisoscel avem

IO ⊥ AI ⇔ bc = 6Rr.

Demonstratie. Intr-adevar,

IO ⊥ AI ⇔ IO2 + IA2 = OA2

⇔ R2 − 2Rr +bc(p− a)

p= R2

⇔ −2Rr + bc− 4Rr = 0

⇔ bc = 6Rr.

Procedand ın acelasi mod, se pot gasi si alte rezultate de acest fel. Nu ıntotdeaunaproprietatea geometrica se caracterizeaza cu o conditie de forma simetrica, simpla.

Propunem spre verificare urmatoarele afirmatii:

1. IH ⊥ AG ⇔ (b− c)2 = a(2a− b− c);

2. IO ⊥ AI ⇔ 6Rr = bc ⇔ 3 sinA

2= 2 cos

B

2cos

C

2.

Bibliografie

1. T. Lalescu – Geometria triunghiului, Editura Tineretului, Bucuresti, 1958.2. D. Sachelarie – Geometria triunghiului. Anul 2000, Matrix Rom, Bucuresti, 2000.

109

Page 26: format .pdf, 3.6 MB

La cissoıde, podaire de la parabole

Adrien REISNER1

Abstract. The cissoid (C) is a pedal curve of a parabola (P ) with respect to the point Ap3,−p

of (P ). The polar equation of (C) is

ρ = −p

(cos θ − sin θ)2

cos θ.

The inverse curve with center A and radius k =9p2

4of (C) is a parabola P.

Keywords: pedal curve, cissoid, inversion.

MSC 2010: 14H50.

Dans un repere orthonorme (Ox,Oy) soitent les points Fp2, 0

et Ap2,−p

,

ou p est une longueur constante. Designons par (P ) la parabole ayant F pour foyeret O pour sommet. Le point A appartient a cette parabole. Tout point M de (P )est repere par son ordonnee egale a pt, ou t est un reel variable. La perpendiculairedu point A sur la tangente en M a (P ) coupe cette tangente en H. La tangente au

point M

1

2pt2, pt

a (P ) admet pour equation: x− ty+

1

2pt2 = 0, la perpendiculaire

a cette tangente issue de A a pour equation : tx + y − 1

2pt + p = 0. Finalement, le

point H admet pour coordonnees:

(1) H : x = −pt

1 + t2, y =

p

2× t3 + t− 2

1 + t2.

Dans notre cas on a la definition suivante:

Definition 1. Lorsque M decrit la parabole (P ), l′ensemble (C) des points H estappele podaire de (P ) pour le pole A.

Son equation parametrique est donnee par (1).On construit la courbe (C) sans dificulte:

dx

dt= p× t2 − 1

(1 + t2)2,

dy

dt=

p

2× (t+ 1)(t3 − t2 + 3t+ 1)

(1 + t2)2;

dy

dx=

t3 − t2 + 3t+ 1

2(t− 1)(t = 1).

Le tableau de variations est le suivant:

1TELECOM ParisTech; e-mail: adrien [email protected]

110

Page 27: format .pdf, 3.6 MB

t −∞ −1 t0 1 +∞dxdt + 0 − − 0 +x 0 p

2 −p2 0

y −∞ −p 0 +∞dydt + 0 − 0 + +

La courbe (C) presente un point stationnaire pour t = −1: c′ est le point A. Latangente en A est parallele a la premiere bissectrice et A est un point de rebrousse-

ment de premiere espece

d2x

dt2=

d2y

dt2= −p

2,d3x

dt3= −3p

2,d3y

dt3= 0

– voir figure –.

Theoreme 2. Trois points Mi(ti) ∈ (C),i : 1, 2, 3, sont allignes si et seulement si on ala relation:

(2) t1t2t3 − (t1 + t2 + t3) = 2.

Demonstration. Soit ux + vy + w = 0l′ equation d′une droite quelconque du plan:elle coupe (C) en trois points dont les troisparametres sont les racines de l′equation

pvt3 + 2wt2 + p(v − 2u)t+ 2w − 2pv = 0.

La courbe (C) est une cubique (trois pointscommuns avec une droite). Les parametres

cherches verifient: t1 + t2 + t3 = −2w

pvet

t1 × t2 × t3 = −2w

pv+ 2, i.e. la relation (2).

Reciproquement, soient trois points H1, H2

et H3 de (C) dont les parametres verifient larelation (2) du Theoreme. La droite H1H2 re-coupe (C) en H ′

3 correspondant a t′3; ainsi H1,H2 et H

′3 sont alignes et par suite, compte tenu

du Theoreme, on a: t1t2t′3 − (t1 + t2 + t′3) = 2.

Or, par hypothese nous avons: t1t2t3 − (t1 +t2 + t3) = 2. On en deduit immediatementt3 = t′3; donc les points H3 et H ′

3 sont confondus, et ainsi le theoreme est demontre.

Corollaire 3. Le point t = 2 : x = −2

5× p, y =

4

5× p est point d′inflexion de la

courbe (C).Demonstration. La tangente d′inflexion a (C) coupe cette courbe en trois points

allignes et confondus, donc t1, t2 et t3 ont la meme valeur t. Les inflexions se trouventparmis les points dont les parametres satisfont a: t3 − 3t − 2 = 0. La racine doublecorrespond au point A de rebroussement. L′inflexion correspond a la troisieme racine

t = 2 de cette equation. On trouve le point (x = −2

5× p, y =

4

5× p).

111

Page 28: format .pdf, 3.6 MB

Soit H(t0) ∈ (C). La tangente en H a (C) recoupe (C) au point K(θ): K estappele le tangentiel de H. On a le

Corollaire 4. Si les trois points Mi(ti) ∈ (C), i : 1, 2, 3, sont alignes, alors il enest de meme de leurs tangentiels.

Demonstration. La tangente en H(t0) a (C) coupe la courbe (C) en trois pointsallignes: H,H et K. A partir de la relation (2) du Theoreme 2, on en deduit:

t20 θ − (2t0 + θ)− 2 = 0 d′ou θ =2

t0 − 1avec (t0 = 1).

On en deduit alors avec des notations evidentes:

θ1θ2θ3 − (θ1 + θ2 + θ3)− 2 = 22− t1t2t3 + t1 + t2 + t3(t1 − 1)(t2 − 1)(t3 − 1)

.

Le corollaire en resulte immediatement.

Cas particuliers. 1) Le tangentiel du point a l′infini de (C) est le point D(θ = 0)situe sur l′asymptote Oy de (C) (t → ∞ donne θ = 0).

2) Si H tend vers le point E(t = 1) de (C) situe sur l′axe Ox, son tangentiel Ks′eloigne a l′infini sur (C) ( t → ∞ ) ce qui correspond au parallelisme de la tangenteen E et de l′asymptote.

Remarque. Si un point est confondu avec son tangentiel, on doit avoir t =2

t− 1;

on trouve le point de rebroussement A(t = −1) et le point d′inflexion t = 2.

Soit R = (AX,AY ) un repere orthonorme dont l′origine est A et dont les axessont paralleles a Ox et Oy.

Theoreme 5. L′equation cartesienne de la courbe (C) dans le repere orthonormeR est

X(X2 + Y 2) +p

2(X − Y )2 = 0.

Demonstration. Dans le repere R nous avons:

X = x− p

2= −p

2× (1 + t)2

1 + t2, Y = y + p =

p

2× t(1 + t)2

1 + t2.

On en deduit t = −Y

Xqui correspond au fait que AH est perpendiculaire a la tangente

en M a la parabole, cette tangente ayant une pente egale a1

t. En eliminant t on

obtient l′equation cherchee.

Remarque. La courbe (C) est une cubique circulaire ((C) passe par les pointscycliques: points de coordonnees homogenes (1,±i, 0)). Elle possede un rebrousse-ment en A et la tangente de rebroussement admet pour equation X − Y = 0. Lacourbe (C) est une cissoıde.

112

Page 29: format .pdf, 3.6 MB

L′equation polaire de la cissoıde (C) s′ecrit:

(3) ρ cos θ +p

2(cos θ − sin θ)2 = 0.

Soit Inv

A,

9p2

4

l′inversion de pole A et de puissance

9p2

4. On a alors:

Theoreme 6. La courbe inverse de (C) dans l′inversion Inv

A,

9p2

4

est la

parabole de foyer F ′− 1

16p,− 7

16p

et de sommet S

7

32p,− 5

32p

.

Demonstration. La courbe inverse s′obtient a partir de l′equation polaire (3) de

la cissoıde en conservant θ et en changeant ρ en9p2

4× 1

ρet on revient en coordonnees

cartesiennes. On peut aussi changer X enkX

X2 + Y 2et Y en

kY

X2 + Y 2ou k =

9p2

4.

On obtient ainsi9p

2X + (X − Y )2 = 0

soit, en revenant aux axes initiaux:

9p

2

x− p

2

+

x− y − 3p

2

2

= 0.

C ′est une parabole (P ′) dont l′axe est parallele a la premiere bissectrice; elle passeen A (tangente X = 0 ). Cette parabole passe par O (ou la pente de la tangente est

−1

2). Le point B ou la tangente a une pente nulle a pour coordonnees B

−5

8p,

1

8p

.

Le foyer F ′ de (P ′) est la projection sur AB du point de concours des deux tangentesen A et en B puisque celles-ci sont perpendiculaires. Les coordonnees de F ′ sont:

F ′− 1

16p,− 7

16p

. Enfin, le sommet S de la parabole (P ′) a pour coordonnees:

S

7

32p,− 5

32p

; c′est l′intersection de la parabole avec son axe qui est issu de F ′ et

est parallele a la premiere bissectrice: la pente de la tangente en S est −1.

Remarque. De facon generale, la pente de la tangente en un point de la paraboleest donnee par

9p

4+

x0 − y0 −

3p

2

1− dy

dx

= 0,

dy

dx= −1 est bien l′ axe de la parabole – voir la figure – .

Bibliografie

1. G. Cagnac, E. Ramis, J. Commeau – Nouveau Cours de Mathematiques Spe-ciales. Tome 4. Applications de l′ Analyse a la Geometrie, Masson et Cie, Paris,1963.

113

Page 30: format .pdf, 3.6 MB

Another proof of the I. Patrascu′s theorem

Florentin SMARANDACHE 1

Abstract. In this note the author presents a new proof for the theorem of I. Patrascu.Keywords: median, symmedian, Brocard′s points.

MSC 2010: 97G40.

In [1], Ion Patrascu proves the following

Theorem. The Brocard ′s point of an isosceles triangle is the intersection of amedian and the symmedian constructed from the another vertex of the triangle′s base,and reciprocal.

We′ll provide below a different proof of this theorem than the proof given in [1]and [2].

We′ll recall the following definitions:Definition 1. The symmetric cevian of the triangle′s median in rapport to the

bisector constructed from the same vertex is called the triangle′s symmedian.Definition 2. The points Ω,Ω′ from the plane of the triangle ABC with the

property ÕΩBA ≡ ÕΩAC ≡ ÕΩCB, respectively ÖΩ′AB ≡ ÖΩ′BC ≡ ÕΩ′CA, are called theBrocard′s points of the given triangle.

Remark. In an arbitrary triangle there exist two Brocard′s points.

Proof of the Theorem. Let ABC an isosceles triangle, AB = AC, and Ω the

Brocard′s point, therefore ÕΩBA ≡ ÕΩAC ≡ ÕΩCB = ω.

We′ll construct the circumscribed circle to the triangle BΩC. HavingÕΩBA ≡ ÕΩCB

and ÕΩCA ≡ ÕΩBC, it results that this circle is tangent in B, respectively in C to thesides AB, respectively AC.

We note M the intersection point of the line BΩ with AC and with N the inter-section point of the lines CΩ and AB. From the similarity of the triangles ABM ,ΩAM , we obtain

(1) MB ·MΩ = AM2.

Considering the power of the point M in rapport to the constructed circle, we obtain

(2) MB ·MΩ = MC2.

From the relations (1) and (2) it results that AM = MC, therefore, BM is a median.If CP is the median from C of the triangle, then from the congruency of the

triangles ABM, ACP we find that ÕACP ≡ÖABM = ω. It results that the cevian CNis a symmedian and the direct theorem is proved.

1University of New Mexico, Gallup Campus, U.S.A.

114

Page 31: format .pdf, 3.6 MB

A

B C

M

N

P

w

ww

W

We′ll prove the reciprocal of this theorem. In the triangle ABC is known thatthe median BM and the symmedian CN intersect in the Brocard′s point Ω. We′llconstruct the circumscribed circle to the triangle BΩC. We observe that because

(3) ÕΩBA ≡ ÕΩCB,

this circle is tangent in B to the side AB. From the similarity of the trianglesABM, ΩAM it results AM2 = MB ·MΩ. But AM = MC, it results that MC2 =MB ·MΩ. This relation shows that the line AC is tangent in C to the circumscribedcircle to the triangle BΩC, therefore

(4) ÕΩBC ≡ ÕΩCA.

By adding up relations (3) and (4) side by side, we obtain ÕABC ≡ ÕACB, conse-quently, the triangle ABC is an isosceles triangle.

References

1. I. Patrascu – O teorema relativa la punctual lui Brocard, Gazeta Matematica, nr.9/1984, LXXXIX, 328-329.

2. I. Patrascu – Asupra unei teoreme relative la punctual lui Brocard, Revista Gamma,nr. 1-2/1988, Brasov.

115

Page 32: format .pdf, 3.6 MB

Comentarii pe marginea unor probleme

Mihai MONEA 1, Steluta MONEA 2

Abstract. We make some methodic remarks on solving geometry problems and we insist on theexistence of the geometric objects before proving their properties.

Keywords: Erdos-Mordell inequality, Brocard angle.

MSC 2010: 97D50.

Punctul de plecare al acestei Note este Problema 26624 din nr. 6-7-8/ 2012,Gazeta Matematica, seria B:

Problema 26624 ([2]). Fie ABC un triunghi si M un punct ın interiorul acestuiaastfel ıncat m(^MAB) = m(^MBC) = m(^MCA) = 30. Sa se arate ca

(1) MA2 +MB2 +MC2 ≥ 16S2

9R2.

Se pune ın mod firesc ıntrebarea daca constructia din ipoteza problemei poatefi realizata, adica: exista M care sa ındeplineasca cerintele problemei? In revisteledestinate cu precadere elevilor apar frecvent probleme ın care se trece usor pestecerinta existentei obiectului geometric pus ın discutie. Consecintele apar imediat.Chiar daca nu suntem ın cazul cel mai grav ın care obictul nu exista, apare totusiriscul ca ın rezolvarea problemei sa pornim pe cai mai complicate si nu pe cele firestisi simple.

In cazul problemei precedente, vom constata mai jos ca existenta unui punct Mce verifica conditiile cerute impune triunghiului ABC sa fie echilateral, iar ınsusipunctuluiM sa fie centrul triunghiului. Odata stabilite acestea, se va constata imediatca relatia (1) are loc cu semnul egal.

Sa presupunem ca problema are ca scop testarea ,,vigilentei” rezolvitorilor. Atunciadevaratul rezolvitor va urma etapele:

1) (partea delicata) arata ca ∆ABC este echilateral si M ≡ O (centrul triunghi-ului),

2) (partea banala) verifica relatia (1) ın care se ia semnul de egalitate.Pe cand, rezolvitorul superficial va stabili ıntr-un fel sau altul ca are loc relatia

ceruta, obtinand egalitate pentru cazul ın care ∆ABC este echilateral. Excludemsituatia ın care o problema cu unele imperfectiuni vede lumina tiparului. Fara altecomentarii...

Revenind la problema enuntata mai sus, vom stabili mai ıntai ca ∆ABC esteechilateral pe doua cai diferite. Instrumentele teoretice ce ne vor ajuta ın acest scopsunt: inegalitatea Erdos-Mordell (de ex., [1], 12.13, p.105.) si masura unghiului

1Profesor, Colegiul National ,,Decebal”, Deva2Profesor, Colegiul National ,,Decebal”, Deva

116

Page 33: format .pdf, 3.6 MB

Brocard ω trece ın al unui triunghi (de ex., [4], 17.13, p. 155). Reamintim acesterezultate ın lemele urmatoare:

Lema 1. Fie P un punct interior triunghiului ABC. Daca x, y, z reprezintadistantele de la P la laturile triunghiului atunci este adevarata inegalitatea

(2) PA+ PB + PC > 2 (x+ y + z) .

Egalitatea are loc daca si numai daca triunghiul este echilateral iar P este centrulsau.

Lema 2. ω ≤ 30 si ω este 30 daca si numai daca ∆ABC este echilateral.

Demonstratie. Fie Ω punctul lui Brocard (direct) si D,E.F proiectiile sale pelaturile BC,CA, si respectiv AB. Avem: m(^ΩBC) = m(^ΩCA) = m(^ΩAB) = ωsi deci au loc relatiile: ΩD = ΩB · sinω, ΩE = ΩC · sinω, ΩF = ΩA · sinω. Aplicandinegalitatea Erdos-Mordell obtinem: ΩA+ΩB +ΩC = 2(ΩB +ΩC +ΩA) · sinω, deunde 2 sinω ≤ 1, adica ω ≤ 30. Conform, lemei 1 are loc si partea a doua a afirmatiei.

Vom da acum doua solutii pentru Problema 26624.

Solutia I. Evident, un triunghi echilateral cu punctulM ın centrul sau ındeplinesteconditiile problemei. Aratam ca acesta este singurul caz posibil. Intr-adevar, dacaexista punctul M cu proprietatea data si notam cu U, V,W picioarele perpendicu-larelor duse din M pe laturile AB,BC si CA, atunci MA = 2MU si analoagele. Prinadunare obtinem MA +MB +MC = 2MU + 2MV + 2PW , adica egalitate ın (2)si, conform Lemei 1, ∆ABC este echilateral si M este centrul sau. Ca urmare, avem:

MA2 +MB2 +MC2 = 3R2 = 3 · a2

3= a2;

dar16S2

9R2=

16

9· 3a

4

16· 3

a2= a2,

deci relatia (1) are loc cu egalitate.

Solutia II. Din conditiile problemei si Lema 2 rezulta nemijlocit ca ∆ABC esteechilateral si M coincide cu centrul sau. Se continua rezolvarea ca ın solutia prece-denta.

Prezentam ınca o problema care se ıncadreaza ın contextul ın care ne aflam.

Problema 97 ([3]). Intr-un triunghi ABC, medianele AA′, BB′, CC ′ fac culaturile AB, BC si, respectiv CA unghiuri de 30. Demonstrati ca triunghiul esteechilateral.

Solutia autorului. Daca unul dintre unghiurile triunghiului are 60, atunci medianarespectiva este bisectoare, iar triunghiul va fi isoscel cu un unghi de 60, deci echila-teral. Daca toate unghiurile sunt diferite de 60, construim BA′′⊥AA′ , CB′′⊥BB′

si AC ′′⊥CC ′, unde A′′ ∈ AA′, B′′ ∈ BB′, C ′′ ∈ CC ′. Atunci, avem:

c

2= BA′′ < BA′ =

a

2,

a

2= CB′′ < CB′ =

b

2,

b

2= AC ′′ < AC ′ =

c

2,

117

Page 34: format .pdf, 3.6 MB

de unde obtinemc

2<

a

2<

b

2<

c

2,

ceea ce nu este posibil.

Solutie alternativa. Rezultatul este o consecinta a Lemei 2. Mai mult, conditiaca AA′, BB′, CC ′ sa fie mediane nu este necesara. Este importanta doar concurentalor.

Bibliografie

1. O. Bottema, R.Z. Djordjevic, R.R. Janic, D.S. Mitrinovic, P.M. Vasic –Geometric Inequalities, Wolters-Noordhoe Publishing, Groningen, 1969.

2. Problema 26624, Gazeta Matematica, seria B, nr. 6-7-8/2012, p.364 (solutie ın nr.1/2013, p.23).

3. Problema 97, Revista Minus, nr. 2/2008 (solutie ın nr. 1/2009, p.28).

4. T. Lalescu – Geometria triunghiului, Editura Tineretului, Bucuresti, 1958.

Raspuns la ,,recreatia” de la pag. 102:

1 11 11

12 7 3

4 2 9 75

3

463

15

Fiecare schema se scrie dupa regula: pe randul doi si de la stanga la dreapta suntscrise suma si produsul numerelor aflate pe randul ıntai, iar pe randul al treilea estescris cel mai mic multiplu comun al numerelor de pe al doilea rand.

Evident, numerele scrise pe primul rand se pot permuta (suma si produsul lorfiind aceleasi).

Vizitati pagina web a revistei Recreatii Matematice:

http://www.recreatiimatematice.ro

118

Page 35: format .pdf, 3.6 MB

Asupra unei clase de ecuatii/inecuatii

Dan POPESCU1

Abstract. The interested reader finds some methodic aspects and comment concerning someselected problems given at mathematical contests.

Keywords: real number, complex number, exponential equation.

MSC 2010: 97D40.

In aceasta Nota sunt stranse ıntr-o clasa un numar de probleme propuse elevilorla editiile din ultimii ani ale olimpiadelor de matematica, ıncepand cu etapa locala.In mare spus, este vorba de rezolvarea unor ecuatii/inecuatii exponentiale care, spredeosebire de cele studiate la orele de matematica, au ca necunoscuta si date numerereale si/sau numere complexe. In abordarea acestor ecuatii se trece frecvent dindomeniul real ın cel complex, se apeleaza la interpretari geometrice sau se utilizeazaajutator functii reale monotone. Asadar, rezolvarea problemelor de acest tip necesitaımbinarea cunostintelor din mai multe capitole. Consideram ca elevii care participala diferite concursuri scolare vor gasi ın aceasta Nota un material folositor pregatiriilor.

Problema 1. Fie a, b ∈ R si z ∈ C/R, astfel ıncat |a− b| = |a+ b− 2z|.a) Sa se arate ca ecuatia |z − a|x + |z − b|x = |a− b|x cu necunoscuta x ∈ R, are

solutie unica.b) Sa se rezolve inecuatia |z − a|x + |z − b|x ≤ |a− b|x, cu necunoscuta x ∈ R.

(Vasile Stanescu – Problema 26669, GM-B-10/2012 si ONM-2013, etapa judeteana).

Solutia I (barem olimpiada). a) Fie z1 = z − a si z2 = z − b. Rezulta ca z1, z2,z1 + z2 ∈ C\R si, ın conditia problemei, |z1 − z2| = |z1 + z2|. Avem |z1 − z2|2 =|z1 + z2|2 ⇔ (z1 − z2)(z1 − z2) = (z1 + z2)(z1 + z2) ⇔ z1z2 + z1z2 = 0, de unde|z1 + z2|2 + |z1|2 + |z22 | + z1z2 + z1z2 = |z1|2 + |z2|2, adica |z1 + z2|2 = |z1 − z2|2 =|z1|2 + |z2|2, ceea ce permite scrierea ecuatiei sub forma |z1|

|z1 − z2|

x

+

|z2||z1 − z2|

x

= 1

sau, echivalent, |z1|È

|z1|2 − |z2|2

x

+

|z2|È

|z1|2 + |z2|2

x

= 1.

Functia f : R → R, definita prin

f(x) =

|z1|È

|z1|2 + |z2|2

x

+

|z2|È

|z1|2 − |z2|2

x

1Profesor, Colegiul National ,,Stefan cel Mare”, Suceava

119

Page 36: format .pdf, 3.6 MB

este strict descrescatoare si ia valoarea 1 pentru x = 2. Ca urmare, ecuatia data arex = 2 ca unica solutie.

b) Multimea solutiilor inecuatiei este intervalul [2,∞).

Solutia II. De fapt, o varianta a primei solutii. Se face apel la interpretareageometrica a notiunii de numar complex. Cu notatiile de mai sus, avem ca |z1−z2| =|z1 + z2|, precum si faptul ca z1 + z2, z1 − z2 ∈ C\R.

Fie, ın planul euclidian raportat la sistemul de coordonate carteziene (O; i, j),punctele M1 si M2 cu afixele z1 si, respectiv, z2. Daca M este imaginea geometrica asumei z1 + z2, relatia |z1 + z2| = |z1 − z2| arata faptul ca OM1MM2 este dreptunghi,

deci vectorii de pozitie−−→OM1 si

−−→OM2 sunt ortogonali. Asadar, exista λ ∈ R∗, astfel

ca z1 = λiz1. Cum |z1 − z2| = |z1|√1 + λ2, ecuatia se scrie ın forma

1√1 + λ2

x

+

|λ|√1 + λ2

x

= 1.

Functia g : R → R, g(x) =

1√1 + λ2

x

+

|λ|√1 + λ2

x

este strict descrescatoare si

g(2) = 1. Atunci ecuatia are solutia unica x = 2.

Problema 2 (enunt modificat). Se considera numerele complexe z1 si z2, ıncat

z2 = 0. Sa se rezolve ecuatia |z1|x + |z2|x = |z1 − z2|x, x ∈ R, stiind ca ℜez1z2

= 0.

(Dan Popescu - ONM-2013, etapa locala, Suceava)

Solutie. Cazul z1 = 0 fiind nesemnificativ, fie z1 = 0. Avem:

ℜez1z2

= 0 ⇔ z1

z2+

z1z2

= 0 ⇔ z1z2 + z1z2 = 0.

Ca urmare, obtinem |z1 − z2|2 = (z1 − z2)(z1 − z2) = z1z1 − z1z2 − z2z1 + z2z2 =|z1|2 + |z2|2. De aici si din faptul ca z1 = 0 si z2 = 0, rezulta ca |z1| < |z1 − z2|, sianalog, |z2| < |z1 − z2|. Astfel, ecuatia se scrie echivalent |z1|

|z1 − z2|

x

+

|z2||z1 − z2|

x

= 1

sau |z1|È

|z1|2 + |z2|2

x

+

|z2|È

|z1|2 + |z2|2

x

= 1

sau f(x) = 1, cu functia f : R → R definita ca ın problema precedenta. Cum aceastafunctie este strict descrescatoare si f(2) = 1, obtinem ca 2 este singura solutie aecuatiei ın cazul z1, z2 ∈ C∗.

Problema 3. Sa se determine multimea M a punctelor din planul Euclidian deafix z ∈ C∗, stiind ca |ℜe||z| + |ℑm z||z| = |z||z|. (Serban Olteanu - ONM-2009etapa locala, Giurgiu)

120

Page 37: format .pdf, 3.6 MB

Solutie. In cazul ℜe z = 0, adica z = bi, multimea este M = (0, b) : b ∈ R∗.Totodata, daca ℑm z = 0, multimea va fi M = (a, 0) : a ∈ R∗.

Fie acum z = a+ bi, a, b ∈ R∗. Ecuatia se scrie echivalent:a√

a2 + b2

√a2+b2

+

b√

a2 + b2

√a2+b2

= 1.

In cazul√a2 + b2 < 2, se deduce inegalitatea

a√a2 + b2

√a2+b2

+

b√

a2 + b2

√a2+b2

>

a√

a2 + b2

2

+

b√

a2 + b2

2

= 1.

La fel, conditia√a2 + b2 < 2 este inacceptabila. Astfel, ın acest caz, multimea M

este cercul centrat ın origine si cu raza 2.

Problema 4. Fie z1, z2, z3 ∈ C∗ care ındeplinesc conditiile |z3|2 = |z1|2 + |z2|2si z1 + z2 + z3 = 0. Stiind ca a ∈ (0, 1) ∪ (1,∞), sa se determine x ∈ (0,∞) dinegalitatea

xloga |z1+z2| = xloga |z1+z3| + xloga |z2+z2|.

Solutie. Tinand cont de ipoteze, avem: xloga |z1+z2| = aloga |z1+z2|·loga x = |z1 +z2|loga x = |z3|loga x etc. Ecuatia data se scrie, echivalent,

|z3|loga x = |z2|loga x + |z1|loga x,

iar aceasta ecuatie are solutia unica loga x = 2, adica x = a2.

Problema 5. Fie a, b, c, d numere complexe distincte doua cate doua ıncat ℜe b− a

d− a=

ℜe b− c

d− c= 0. Sa se determine x ∈ R, daca |a − b|x + |a − d|x ≤ |b − d|x ≤

|b − c|x + |d − c|x. (Enunt propus ın Complex Numbers from A to...Z, BirkhauserBoston, 2006, autori Titu Andreescu si Dorin Andrica).

Solutie. Daca A,B,C,D sunt punctele planului cu afixele a, b, c, d, conditiile

ℜe b− a

d− a= ℜe b− c

d− c= 0 exprima faptul ca m(]BAD) = m(]BCD) = 90. Astfel,

|a− b| = AB, |a−d| = AD si |b− d| = BD si prima inegalitate devine ABx+ADx ≤BDx, adica, echivalent, x ≥ 2. Similar, a doua inegalitate conduce la x ≤ 2. Inconcluzie, x = 2 este solutia problemei.

In final, propunem cititorului o problema legata de continutul Notei:

Problema 6. Sa se determine x ∈ R, daca |1 + ab|2x + |a − b|2x = (1 + |a|2 +|b|2 + |ab|2)x, unde a, b sunt numere complexe. (Dan Popescu).

Indicatie. Mai ıntai se analizeaza cazurile ab = 0 si a = b, iar apoi se apeleazaidentitatea usor de dovedit |1 + ab|2 + |a− b|2 = 1 + |a|2 + |b|2 + |ab|2, ∀a, b ∈ C.

121

Page 38: format .pdf, 3.6 MB

Rezolvarea unor ecuatii si inecuatii integrale

Florin STANESCU1

Abstract. It is shown how the classical integral inequalities can be used, through the cases whenthey become equalities, for solving some types of integral equations or inequations. Chebyshev′sintegral inequality and Jensen′s integral inequality are pre-eminently used.

Keywords: monotone function, convex function, Cebyshev′s integral inequality, Jensen′s inte-gral inequality, integral equation.

MSC 2010: 97I70.

In aceasta Nota prezentam un procedeu de rezolvare aplicabil unui anumit tipde ecuatii sau inecuatii integrale sau unui tip de probleme ce apar adesea ın con-cursurile scolare si reviste. Procedeul consta ın urmatoarele: pentru rezolvarea uneiecuatii/inecuatii integrale se utilizeaza o inegalitate integrala clasica care se dovedestea fi egalitate ın conditiile problemei si, ın consecinta, suntem condusi la solutia pro-blemei.

Amintim inegalitatile integrale care vor fi utilizate cu precadere mai jos:

Inegalitatea lui Cebasev: Fie f, g : [a, b] → R doua functii monotone. Atunci:

a) (b−a)

Z b

af(x)g(x)dx ≥

Z b

af(x)dx·

Z b

ag(x)dx, daca f si g au aceeasi monotonie;

b) (b− a)

Z b

af(x)g(x)dx ≤

Z b

af(x)dx ·

Z b

ag(x)dx, daca f si g sunt de monotonii

diferite.

In acestea, avem egalitate daca si numai daca una dintre functiile f, g este con-stanta (eventual cu exceptia unei multimi numarabile).

Inegalitatea lui Jensen. Fie f : I → J , g : J → R doua functii continue.Atunci:

a) g

1

b− a

Z b

af(x)dx

≤ 1

b− a

Z b

ag(f(x))dx, daca g este convexa;

b) g

1

b− a

Z b

af(x)dx

≥ 1

b− a

Z b

ag(f(x))dx, daca g este concava.

Daca functia g este strict convexa (respectiv strict concava), atunci la punctul a)(respectiv punctul b)) avem egalitate daca si numai daca functia f este constanta.

Problema 1. Determinati functiile f : [0, 2] → R derivabile de doua ori, cuderivata f ′′ crescatoare pe [0, 2] si care verifica ecuatiaZ 2

1f(x)dx−

Z 1

0f(x)dx =

f(2)− f(0)

2.

1Profesor, Scoala ,,Serban Cioculescu”, Gaesti

122

Page 39: format .pdf, 3.6 MB

Solutie. Stim ca o functie φ : I → R este convexa pe intervalul I daca si numaidaca ∀x, y, z, t ∈ I, x < y < z < t, avem:

f(z)− f(x)

z − x≤ f(t)− f(y)

t− y.

Deoarece f ′′ este crescatoare, rezulta ca f ′ este convexa pe [0, 2]; ca urmare,daca x, y ∈ [0, 1], x < y, ceea ce ınseamna ca x < y < x + 1 < y + 1, avem:f ′(x + 1) − f ′(x) ≤ f ′(y + 1) − f ′(y), adica functia x → f ′(x + 1) − f ′(x) estecrescatoare pe [0, 1].

Conform inegalitatii lui Cebasev, putem scrie:Z 1

0x[f ′(x+ 1)− f ′(x)]dx ≥

Z 1

0x dx ·

Z 1

0[f ′(x+ 1)− f ′(x)]dx

=f(2)− 2f(1) + f(0)

2.(1)

Pe de alta parte, cu ajutorul formulei de integrare prin parti si tinand seama deecuatia din enunt, avem:Z 1

0x[f ′(x+ 1)− f ′(x)]dx = x[f(x+ 1)− f(x)]

10

−Z 1

0[f(x+ 1)− f(x)]dx

= f(2)− f(1)−Z 2

1f(t)dt+

Z 1

0f(x)dx

= f(2)− f(1)− f(2)− f(0)

2

=f(2)− 2f(1) + f(0)

2.

Asadar, ın (1) vom avea egalitate peste tot. Rezulta ca una dintre functiile carora le-am aplicat inegalitatea lui Cebasev este constanta, ceea ce revine la f ′(x+1)−f ′(x) =k (constanta reala), ∀x ∈ [0, 1]. Urmeaza ca

f ′′(x+ 1) = f ′′(x), ∀x ∈ [0, 1]

si deci f ′′(0) = f ′′(1) = f ′′(2). De aici si din faptul ca f ′′ este crescatoare pe [0, 2],deducem ca f ′′ este constanta pe [0, 2]. In concluzie, functiile f : [0, 2] → R careındeplinesc cerintele problemei au forma f(x) = ax2 + bc+ c, a, b, c ∈ R.

Problema 2. Determinati functiile f : [−1, 1] → R derivabile de doua ori si cuderivata f ′′ continua pe [−1, 1], crescatoare pe [−1, 0] si descrescatoare pe [0, 1] si careındeplinesc conditia f ′(1)− f ′(−1) = 2[f(1)− 2f(0) + f(−1)].

Solutie. Consideram functia g : [−1, 1] → R data de

g(x) =

¨x+ 1, x ∈ [−1, 0]

−x+ 1, x ∈ [0, 1]

123

Page 40: format .pdf, 3.6 MB

si observam ca ea este continua pe intervalul [−1, 1], crescatoare pe [−1, 0] si des-crescatoare pe [0, 1].

In conditiile problemei, putem aplica inegalitatea lui Cebasev pe intervalele [−1, 0]si [0, 1] functiilor g si f ′′:Z 1

−1g(x)f ′′(x)dx =

Z 0

−1g(x)f ′′(x)dx+

Z 1

0g(x)f ′′(x)dx

≥Z 0

−1g(x)dx ·

Z 0

−1f ′′(x)dx+

Z 1

0g(x)dx ·

Z 1

0f ′′(x)dx

=

Z 0

−1(x+ 1)dx · [f ′(0)− f ′(−1)] +

Z 1

0(−x+ 1)dx · [f ′(1)− f ′(0)]

=f ′(1)− f ′(−1)

2.

Pe de alta parte, integrand prin parti obtinem:Z 1

−1g(x)f ′′(x)dx =

Z 0

−1g(x)f ′′(x)dx+

Z 1

0g(x)f ′′(x)dx

= (x+ 1)f ′(x)

0−1

−Z 0

−1f ′(x)dx+ (−x+ 1)f ′(x)

10

+

Z 1

0f ′(x)dx

= f ′(0)− [f(0)− f(−1)]− f ′(0) + [f(1)− f(0)] =

= f(1)− 2f(0) + f(−1).

Combinand rezultatele obtinute si avand ın vedere relatia din problema, avem:

f(1)− 2f(0) + f(−1) =

Z 1

−1g(x)f ′′(x)dx ≥ f ′(1)− f ′(−1)

2= f(1)− 2f(0) + f(−1),

deci inegalitatea lui Cebasev devine egalitate pe fiecare din cele doua intervale si, caurmare, functia continua f este constanta pe [−1, 1]. In consecinta, pentru functia fgasim forma: f(x) = ax2+ bx+ c, x ∈ [−1, 1] (a, b, c ∈ R arbitrare). Se verifica directca aceste functii satisfac cerintele problemei.

Problema 3. Se considera f : [−1, 1] → R o functie continua si strict convexa,iar a, b ∈ [−1, 1] astfel ıncat

a

Z b

0f(x)dx− b

Z −a

0f(x)dx = 2ab · f

b− a

4

.

Aratati ca ab = 0.

Solutie. Presupunem, prin absurd, ca ab < 0 (analog se rationeaza si pentruab > 0). Egalitatea din enunt poate fi scrisa sub forma

f

b− a

4

=

1

2

1

a

Z 0

−af(x)dx+

1

b

Z b

0f(x)dx

.

124

Page 41: format .pdf, 3.6 MB

Definim functia g : [−1, 1] → R prin

g(x) =

¨ax, x ∈ [−1, 0]

bx, x ∈ [0, 1].

Ea este continua si nu este constanta. Conform inegalitatii lui Jensen, putem scrie:

f

1

2

Z 1

−1g(x)dx

<

1

2

Z 1

−1f(g(x))dx

⇔ f

1

2

Z 0

−1ax dx+

Z 1

0bx dx

<1

2

Z 0

−1f(ax)dx+

Z 1

0f(bx)dx

⇔ f

b− a

4

<

1

2

1

a

Z 0

−af(x)dx+

1

b

Z b

0f(x)dx

.

Ultima inegalitate este ın contradictie cu forma ın care a fost scrisa conditia din enunt.In concluzie, avem ab = 0.

Problema 4. Determinati functiile continue f : [0, 2] → R pentru care exista

a ∈ R astfel ıncat f(x3 + x) = sin ax, ∀x ∈ [0, 1] si

Z 2

0f(x)dx = 2 sin

5a

8.

Solutie. Functia g : [0, 1] → [0, 2] definita prin g(x) = x3 + x este bijectiva,continua si strict crescatoare. Prima conditie se scrie f(g(x)) = sin ax, x ∈ [0, 1], sau

f(x) = sin(a g−1(x)), x ∈ [0, 2]. A doua conditie se va scrie

Z 2

0sin(a g−1(x))dx =

2 sin5a

8. Aplicand inegalitatea lui Jensen, functia sinus fiind concava pe [0, 2], obtinem:

sin5a

8=

1

2

Z 2

0sin(a g−1(x))dx ≤ sin

a

2

Z 2

0g−1(x)dx

.

Putem calcula

Z 2

0g−1(x)dx cu inegalitatea lui Young (cazul de egalitate):Z 1

0g(x)dx+

Z 2

0g−1(x)dx = 2 · 1− 0 · 0 ⇔

Z 2

0g−1(x)dx = 2−

Z 1

0(x3 + x)dx =

5

4.

Introducand acest rezultat mai sus, constatam ca ın urma aplicarii inegalitatii luiJensen obtinem o egalitate. In consecinta, functia ag−1 este constanta pe [0, 2] decia = 0. Prima conditie din enunt devine f(x3 + x) = 0, x ∈ [0, 1], adica f(x) = 0,x ∈ [0, 2].

Bibliografie

1. N. Boboc – Analiza matematica, Editura Universitatii din Bucuresti, 1999.

2. M. Ganga – Teme si probleme de matematica, Editura Tehnica, Bucuresti, 1991.

3. C. Mortici – 600 de probleme de matematica pentru concursuri, Editura Gil, Zalau,2001.

125

Page 42: format .pdf, 3.6 MB

Merge si asa!

Marian TETIVA 1

Abstract. In this note, a couple of remarks, comments and solving methods regarding equationa cos t+ b sin t = c are presented for both cases when a, b, c ∈ R, respectively a, b, c ∈ C.

Keywords: equation, complex number, sinus, cosinus.

MSC 2010: 97D40.

Nu, nu ma refer cu acest titlu la binecunoscutul cuvant de ordine, aplicabil oride cate ori ceva functioneaza prost, dar totusi functioneaza. Nu, aici vom vorbichiar despre o alta metoda de a rezolva o problema: e vorba de aflarea sinusului sicosinusului unui numar real t stiind ca ele verifica o ecuatie de forma a cos t+b sin t = c,unde a, b, c sunt numere reale cu a2 + b2 = 0 si a2 + b2 ≥ c2 (conditia cunoscuta deexistenta a solutiilor). Pentru diverse metode de a face acest lucru se poate consultaorice manual de trigonometrie pentru ıncepatori. Metoda despre care vorbim noi ıncontinuare este la ındemana oricarui absolvent de clasa a zecea, dar cred ca nu preaati vazut-o folosita ın scoala elementara.

Vom utiliza numere complexe: consideram pe z = cos t + i sin t si observam caavem

cos t =1

2(z + z) si sin t =

1

2i(z − z),

unde z este conjugatul complex al lui z, prin urmare ecuatia se scrie

a1

2(z + z) + b

1

2i(z − z) = c.

Dar z este un numar complex de modul 1, deci z = 1/z, ceea ce ınseamna ca ecuatianoastra devine una de gradul al doilea (cu coeficienti complecsi), anume:

(a− bi)z2 − 2cz + a+ bi = 0.

Avand ın vedere ca a2 + b2 ≥ c2 discriminantul ecuatiei este ∆ = 4(c2 − a2 − b2) ≤ 0,deci ea are solutiile

z1,2 =c± i

√a2 + b2 − c2

a− bi,

adica

z1 =ac− b

√a2 + b2 − c2

a2 + b2+

bc+ a√a2 + b2 − c2

a2 + b2i

si

z2 =ac+ b

√a2 + b2 − c2

a2 + b2+

bc− a√a2 + b2 − c2

a2 + b2i.

1Profesor, Colegiul National ,,Gheorghe Rosca Codreanu”, Barlad

126

Page 43: format .pdf, 3.6 MB

Amintindu-ne ca z = cos t + i sin t si identificand partile reale, respectiv imaginare,vedem ca valorile posibile pentru cos t si sin t sunt

cos t =ac− b

√a2 + b2 − c2

a2 + b2si sin t =

bc+ a√a2 + b2 − c2

a2 + b2

sau

cos t =ac+ b

√a2 + b2 − c2

a2 + b2si sin t =

bc− a√a2 + b2 − c2

a2 + b2.

Desigur, ın cazul a2 + b2 = c2, obtinem, practic, o singura valoare pentru perechea(cos t, sin t) (de fapt: doua egale). Cititorul este invitat sa se convinga singur ca acestevalori verifica ecuatia initiala si ca ele reprezinta, ıntr-adevar, cosinusul, respectivsinusul unui numar real - cu alte cuvinte sa vada ca suma patratelor lor este egalacu 1.

Probabil ca nici de acum ıncolo profesorii nu vor folosi aceasta metoda de rezolvarea ecuatiei a cos t+ b sin t = c cu elevii lor. Totusi, trebuie sa-i recunoastem cel putinun prim merit: acela ca ne permite sa calculam efectiv cos t si sin t ın functie de a, b, c,fara prea mare efort si ıntr-un fel destul de elegant. (Faceti asta si rezolvand sistemulformat din ecuatiile a cos t + b sin t = c si cos2 t + sin2 t = 1, cum se procedeazade obicei: nu mai e la fel de atragator, nu?) Totusi, e clar ca atunci cand rezolviprima data ecuatii de acest tip nu poti folosi numere complexe, ın primul rand pentruca ınca nu s-au studiat. Pe de alta parte, dupa ce s-a ıncheiat si capitolul desprenumere complexe (cu tot cu forma lor trigonometrica) poate ca o asemenea privirepeste umar, retrospectiva, ar avea beneficiile ei - ca sa nu mai vorbim de faptul cautilizarea numerelor complexe ın probleme al caror enunt pare sa nu aiba de a face cuele reprezinta o metoda puternica (si omniprezenta ın matematica) pentru a solutionaprobleme dificile.

Stiu ca v-ati pus deja o ıntrebare (stiu, pentru ca este o ıntrebare naturala): putemoare stabili conditia de existenta a solutiilor ecuatiei prin aceasta metoda? Raspunsuleste: da, dar... Adica da, se poate, dar mai complicat decat se face clasic (de exemplufolosind, ıntr-o forma sau alta, inegalitatea Cauchy-Schwarz). Pentru asta sa admitemca am avea 0 = a2 + b2 < c2, deci ecuatia cu necunoscuta z are discriminantul numarreal si pozitiv si, prin urmare, solutiile sale sunt

z1,2 =c±

√c2 − a2 − b2

a− bi=

c±√c2 − a2 − b2

a2 + b2(a+ bi).

Acum ideea e ca noi cautam un asemenea z care, ın plus, are si modulul 1. Conditia|z1| = 1 sau |z2| = 1 se scrie

|c±p

c2 − a2 − b2| =pa2 + b2

(pentru o alegere a semnului) si e usor de verificat ca, dupa ridicarea la patrat obtinem(indiferent cu care dintre semnele plus/minus am pornit) a2 + b2 = 0 - contradictiecare arata ca ecuatia nu poate avea solutii daca a2 + b2 < c2 (si cel putin unul dintrea si b este nenul). Deci se poate: merge si asa!

127

Page 44: format .pdf, 3.6 MB

Mai departe mi-am pus urmatoarea ıntrebare (poate deja si cititorul a facut asta):ce se ıntampla daca a, b, c nu mai sunt, neaparat, numere reale? Putem gasi numeret astfel ıncat a cos t+ b sin t = c?

Sa ıncercam o alta abordare (ıntrucat, daca am vrea sa procedam ca mai sus, ne-am ımpotmoli la rezolvarea ecuatiei, neavand niciun control asupra discriminantului)si anume utilizarea conjugatului - o tehnica obisnuita pentru numere complexe. Dina cos t+b sin t = c rezulta (prin conjugare, si tinand seama de faptul ca numerele cos tsi sin t sunt reale) a cos t + b sin t = c. Acum sa interpretam aceste doua ecuatii cape un sistem liniar (cu necunoscutele cos t si sin t), pe care-l rezolvam cu regula luiCramer (admitand ca ea este aplicabila). Gasim

cos t =bc− bc

ab− absi sin t =

ac− ac

ab− ab,

daca presupunem ca ab = ab (ın particular aceasta metoda nu se poate aplica dacaa si b sunt ambele numere reale, ca ın cazul de manual studiat anterior). Desigurnumarul t exista daca si numai daca valorile obtinute pentru cos t si sin t sunt numerereale si suma patratelor lor este egala cu 1. Se poate usor vedea ca aceste numeresunt reale, deoarece fiecare este egal cu conjugatul sau (w ∈ R ⇔ w = w). Astfel amdemonstrat valabilitatea urmatorului enunt:

Problema 1. Fie a, b, c numere complexe astfel ıncat ab = ab. Exista un numarreal t care verifica a cos t+ b sin t = c daca si numai daca

(bc− bc)2 + (ac− ac)2 = (ab− ab)2.

Lasam (iar!) ın grija cititorului interesat detaliile acestei demonstratii (e un enuntcu ”daca si numai daca”!) precum si sa studieze ce se ıntampla daca ab = ab; deexemplu, daca a, b, c sunt toate reale, conditia este ındeplinita, dar existenta lui t nueste asigurata ın toate cazurile, asa cum am vazut ın prima parte a acestei note.

La sfarsit va propun (ca de obicei) ınca un exercitiu a carui rezolvare seamanamult cu cea de mai sus (daca va ganditi putin, vedeti ca si enunturile seamana, nu?).Observati, de asemenea, de unde am plecat si de unde am ajuns: ıntotdeauna seıntampla asa, daca vrei mai mult decat, pur si simplu, sa rezolvi o problema; adicadaca ıti pui ıntrebari si cauti mereu alte solutii pentru probleme deja rezolvate (chiaraparent epuizate).

Problema 2. Fie a, b, c trei numere complexe cu a = 0 si |a| = |c|. Ecuatiaaz2 + bz + c = 0 are o (singura) radacina de modul 1 daca si numai daca are locegalitatea

|ab− bc| = ||a|2 − |c|2|.

Observati, de asemenea, ca enuntul ramane valabil si pentru a = 0 (dar nu mai evorba despre o ecuatie de gradul al doilea). Ce se poate oare spune despre moduleleradacinilor ecuatiei daca avem |ab− bc| = ||a|2 − |c|2| si |a| = |c|?

128

Page 45: format .pdf, 3.6 MB

Colegiul National ,,Stefan cel Mare” Harlau

La Harlau, Alexandru Lapusneanu ınfiinta ın 1551 un Colegiu, prima unitatede ınvatamant laica de grad mediu din Moldova 1. A fost o scoala domneasca cuinternat, unde educatia era bine controlata si asigura o cultura ınalta. Programascolara avea la baza cunoasterea limbii latine, a scriitorilor clasici latini, matematicile,stiintele ın general, iar ınvatamantul religios redus.

In 1856 ia fiinta Scoala de baieti ,,Stefan cel Mare” din Harlau, care afunctionat ın actualul local al primariei pana ın 1881. Intre anii 1881-1944 va functionafara ıntrerupere ıntr-un local nou, construit ın 1881 ın gradina bisericii ,,Sf. Dumitru”din Harlau - ctitorie a lui Petru Rares. La ınceput a functionat cu trei ınvatatori, iarelevii erau ımpartiti ın patru clase (1856). Scoala a facut fata ınvatamantului publicprimar din Harlau aproape sapte decenii, pana ın anul 1949.

Asa cum arata V.M. Lisman1, unii absolventi ai ultimelor generatii, pe caremonografistul i-a cunoscut, evocau ravna si daruirea unor cadre didactice precum:Grigore Rozan, Ion Popovici-Puiu, Neculai C. Ursoi, Ion I. Onu, I. D. Dragan, TomaC. Zaharia si altii.

Din 1949 devine gimnaziu mixt, iar ın anul 1965, Scoala medie va deveni Liceude cultura generala conform H.C.M. nr. 903/1965. Din 1974 se numeste Liceu real-umanist, cu 12 clase.

Strans legat de procesul de industrializare are loc transformarea liceelor real-umaniste ın licee industriale; ca urmare, liceul nostru devine ın 1977 liceu industrial,

1Nicolae Iorga, Istoria romanilor prin calatori, I, p. 163.1Vasile M. Lisman, Harlau (Monografie), sub egida Comitetului de Cultura si Educatie Socia-

lista, Iasi, 1972, p. 70.

129

Page 46: format .pdf, 3.6 MB

patronat de Ministerul Industriei Chimice. Va avea la ınceput clase cu profil de chimieindustriala, mecanica si mai tarziu se vor adauga noi profiluri ca: industrie usoara,industria lemnnului si agrobiologie.

In 1990, ca urmare a dorintei cadrelor didactice ale scolii si reınnodand o perioadade ımpliniri, liceul devine Liceul Teoretic ,,Stefan cel Mare” prin ordinul nr.45816/1990 al Ministerului Invatamantului si Stiintei. Numele liceului este numelemarelui voievod moldav ca recunoastere pentru ,,chipul de viteaz, bun, cuminte sisfant al celui mai mare om ce s-a ridicat dintre romani”1.

Se reiau clasele cu profil real de chimie-biologie, matematica-fizica, fizica-chimiesi cu profil uman: filologie, filologie-limbi straine, istorie-stiinte sociale, iar din anulscolar 1992/1993 va lua fiinta o clasa cu profil de informatica.

In toamna anului 2011 liceul s-a transformat ın colegiu – Colegiul National,,Stefan cel Mare”, prin ordin ministerial nr. 4535 din 01.07.2011, recunoscandu-i-se astfel acestei scoli si dascalilor ei aportul avut ın ınvatamantul romanesc.

Dintre absolventii scolii noastre, cu nume de prestigiu ın domeniile ın care au ac-tivat, enumeram: Vasiliu Titu (1885-1961), elev si urmas al savantului Victor Babes,fondatorul scolii de anatomie patologica din Cluj, doctor ın medicina si chirurgie;Gheorghe Leahu, directorul Teatrului National din Timisoara (1956-1970); Lucia Ti-buleac, solista a Operei Romane din Bucuresti; Ioan Emanoil Gandrabura, prof. laUniversitatea ,,Al.I. Cuza” Iasi, Facultatea de geografie si geologie; Natan Cohen,prof. la Universitatea ,,Ben Gurion” din Negev, Israel; Carol Iancu, prof. de istoriecontemporana la Universitatea ,,Paul Valery” din Montpellier, Doctor Honoris Causaal Universitatii ,,Babes-Bolyai” Cluj si al Universitatii ,,Al.I. Cuza” Iasi si multi altii.

Colegiul National ,,Stefan cel Mare” din Harlau organizeaza anual Concursulnational ,,Micii Matematicieni”, aflat ın anul 2013 la editia a VIII-a, si publica revistacu acelasi nume. Aceasta manifestare este girata de SSMR si de ISJ Iasi.

Intre concursurile derulate de colegiul nostru se afla si Concursul literar ,,CezarPetrescu”, dedicat memoriei scriitorului nascut ın decembrie 1892 la Hodora, Cotnari.La concurs au participat: Colegiul National ,,M. Sadoveanu” Pascani, Liceul Teoretic,,I. Neculce” Tg. Frumos, Colegiul Tehnic de Cai Ferate ,,Unirea” Pascani, GrupulScolar ,,M. Busuioc” etc.

Colegiul National ,,Stefan cel Mare” Harlau organizeaza anual Concursul de web,,HWEB”, care consta ın crearea unei pagini web cu tema ,,Afacerea mea pe internet”si un test-grila referitor la cunostintele elevilor din domeniul IT si web.

Clubul de turism si ecologie ,,Ecomont” a fost ınfiintat ın Liceul Teoretic ,,Stefancel Mare” Harlau la ınceputul anului 1996 si a avut de-a lungul timpului numeroaseactivitati dedicate protejarii mediului, iar prin intermediul ei s-au organizat excursiide studiu si de agrement ın diverse zone ale tarii.

Cercul de pictura religioasa ,,Trepte catre cer” desfasoara ın timpul vacantei deiarna activitati de pictura religioasa si are ın vedere descoperirea si cultivarea talen-telor si abilitatilor artistice ale elevilor.

Cercul de arta dramatica ,,Arlechino” se afla ın al cincilea an de existenta, sereuneste periodic, de trei ori pe luna, si exerseaza pe diverse texte dramatice sub

1Nicolae Iorga, Istoria lui Stefan cel Mare, editie ıngrijita si tabel cronologic de Victor Iova,Bucuresti, Editura Minerva, 1978.

130

Page 47: format .pdf, 3.6 MB

ındrumarea prof. Iancu Mirela.Asociatia sportiva scolara ,,Olimpia” a fost ınfiintata ın anul 2010. Profesorii coor-

donatori sunt Butnariu Marius Doinas, Danielevici Iulian Constantin, iar presedinte- prof. Sacaleanu Ioan. Asociatia ısi popune promovarea ın randul elevilor a urma-toarelor sporturi: atletism, baschet, badminton, fotbal, gimnastica ritmica, handbal,tenis de masa, tenis de camp, oina, sah, volei.

Clubul ,,Micilor matematicieni” a fost ınfiintat ın anul 2002, mai ıntai sub denu-mirea de Cercul elevilor ,,Pro-matematica”. Are ca scop pregatirea elevilor colegiuluinostru pentru concursurile scolare, prin rezolvarea problemelor propuse ın reviste despecialitate. Initiatorul acestui cerc si apoi al revistei Micii matematicieni este prof.Ioan Sacaleanu. Iar din 2006 a luat fiinta Concursul Micii Matematicieni, coordonatde prof. Aurel Neicu, avand ca presedinte pe prof. Gheorghe Oancea.

Proiecte si campanii internationale. Mentionam doar doua dintre multele proiectepe care colegiul le-a derulat.

In spiritul cooperarii si al valorizarii sistemului educational din Europa, ın anulscolar 2011/2012 prof. Aurel Neicu a facut o aplicatie pentru proiectul internationalindividual Grundvig pe tema exploatarii si inovarii ın domeniul mediului si al is-toriei. Activitatile proiectului ,,Dry stone in Europe” s-au desfasurat ın localitateaArlanc/Franta si au constat ın reabilitarea unui vechi sit cu valenta culturala.

Tot ın anul scolar 2011/2012 a fost castigat prin concurs un proiect internationalComenius cu tema ,,Mostenirea istorica ın sprijinul construirii Europei” de catre oechipa formata din prof. Adriana Gandac, prof. Aurel Neicu, prof. Pusa Alexa.Proiectul se desfasoara ın intervalul 2012-2014 si va consta ın mobilitati partenerialeın Franta, Germania, Turcia, Italia, Bulgaria si Polonia.

Prin munca multor generatii de elevi si profesori Colegiul National ,,Stefan celMare” a dobandit un renume pe care actuala generatie de elevi si dascali se obligasa-l pastreze si sa-l sporeasca prin munca, talent si daruire, acest fapt fiind o obligatiesi un legamant de onoare.

Prof. Aurel NEICUDirector al C.N. ,,Stefan cel Mare” Harlau

ERRATUM

D-l Prof. Dumitru Barac semnaleaza faptul ca ın enuntul problemei X.127 dinRecreati Matematice nr. 1/2012 trebuie adaugata conditia a2 > bc.

Vizitati pagina web a revistei Recreatii Matematice:

http://www.recreatiimatematice.ro

131

Page 48: format .pdf, 3.6 MB

Concursul de matematica ,,Florica T. Campan”

Editia a XIII-a, Iasi, 2013

Clasa I

I.1. Doar 5 numere respecta regula de alcatuire a sirului. Care este intrusul?Incercuieste-l! 73, 28, 52, 19, 46, 55.

I.2. Descopera regula de formare a sirului, apoi completeaza:

2, 6, 4; 3, 7, 5; 4, 8, 6; 5, . . . , . . . ; 6, . . . , . . . .

I.3. Completeaza patratul din figura cu numerele 1, 2, 3 si 4 astfel ıncat sumanumerelor de pe fiecare linie (pe orizontal) si de pe fiecare coloana (pevertical) sa fie 10.

II.1. Un fluture zboara din floarea 1 ın floarea 3, apoi din aceasta ınfloarea 5 si asa mai departe. Dupa cate zboruri ajunge ın floarea de pecare a plecat?

12

3

456

7

8

9

II.2. Iepurila are 18 oua de ciocolata. Iepurica are cu 3 oua mai putin decatIepurila, dar cu 10 oua mai mult decat Rila. Sa se afle cate oua de ciocolata are Rila!

III.1. Corina a cules 13 lalele galbene si 4 lalele rosii. Ema a cules 7 lalele, unelegalbene, altele rosii. Fetitele pun toate lalelele ın vaza.

Care este cel mai mare numar de lalele galbene care ar putea fi ın vaza?

III.2. De la apartamentul meu cobor 4 etaje, apoi urc 3 etaje si observ ca sunt laetajul 9. La ce etaj locuiesc?

Clasa a II-a

I.1. Gasiti numerele care lipsesc din urmatorul sir: 1, 1, 2, 3, 5, 8, 13, . . . , . . . , . . ..

I.2. Varsta bunicii este un numar cuprins ıntre 60 si 70 de ani. Stiind ca diferentadintre cifra unitatilor si cea a zecilor din acel numar este 3, afla ce varsta are bunica.

II.1. Dintre Alin, Marin, Corina, Ioana si Raluca trebuie sa aleg membrii uneiechipe formate din doua fete si un baiat. In cate moduri o pot face?

132

Page 49: format .pdf, 3.6 MB

II.2. Timp de sase zile, veverita a carat ın scorbura sa ghinde astfel ın fiecare zicu 17 ghinde mai mult decat ın ziua precedenta. Stiind ca ın a patra zi a carat 68 deghinde, afla cate ghinde a adunat ın total, ın cele sase zile.

III.1. Daca doua papusi si o masinuta costa 8 lei, iar o papusa si doua masinutecosta 7 lei, cat costa noua papusi si noua masinute?

III.2. Pe trei rafturi sunt 75 de carti. Daca pe primul raft punem jumatatedin cartile de pe cel de-al doilea, atunci vom avea pe rafturi, ın ordinea lor, numereconsecutive de carti. Cate carti erau la ınceput pe fiecare raft?

Clasa a III-a

I. Veveritele Lia, Mia si Kia aduna alune. Impreuna au adunat un numar de aluneegal cu cel mai mare numar par de trei cifre. Lia si Mia au adunat un numar de aluneegal cu cel mai mare numar de trei cifre ce are suma cifrelor 5. Mia si Kia au adunatun numar de alune egal cu cel mai mare numar par de trei cifre ce are suma cifrelor10. Care din cele trei veverite este cea mai harnicuta? (Justificati!)

II.1. Un numar A se va numi angelic daca suma cifrelor lui A este egala cunumarul cifrelor lui A. (Spre exemplu 2011 si 200013 sunt numere angelice deoarece2 + 0 + 1 + 1 = 4 si 2 + 0 + 0 + 0 + 1 + 3 = 6). Scrieti toate numerele angelice careau trei cifre.

II.2. Aflati suma numerelor de trei cifre care au produsul cifrelor egal cu 8.

III. Petrut s-a nascut ın anul 19xy, iar tatal sau ın anul 19yx. Cand Petrut avea25 de ani, tatal sau avea 52 de ani. Aflati ın ce an s-a nascut Petrut si tatal sau,stiind ca ın anul 2013 au ımpreuna 105 ani.

Clasa a IV-a

I (Nu va jucati cu chibriturile!). a) Construim patrate din chibrituri. Primulpatrat are latura de un chibrit, al doilea din trei chibrituri, al treilea din cinci chi-brituri etc. Aflati cu cate chibrituri folosim mai mult la constructia patratului cunumarul 100 decat la constructia patratului cu numarul 99.

b) Care este suma perimetrelor primelor 100 de patrate astfel formate?

II (Feriti-va de aviara!). Speriate teribil de efectele virusului gripei aviare(H5N1), gainile dintr-o gospodarie au plecat grabite sa faca un control amanuntit lamedicul veterinar. Transpirate si obosite dupa drumul parcurs, s-au oprit ıntr-un parcsa se odhneasca. Ele constata ca daca s-ar aseza cate zece gaini pe o banca ramansase banci goale, iar pe o banca ar fi numai trei gaini. Daca s-ar aseza cate sase gainipe cate o banca, ar exista o singura banca cu numai trei gaini, iar restul bancilor arfi ocupate cu cate sase gaini. Cate gaini are gospodarul si cate banci sunt ın parc?

III (Razboiul numerelor ,,stelare”). Un numar S se numeste ,,stelar” dacasuma cifrelor lui S este egala cu numarul cifrelor lui (spre exemplu 2011 este numar,,stelar” deoarece are patru cifre si 2 + 0 + 1 + 1 = 4).

a) Scrieti toate numerele ,,stelare” de patru cifre.b) Determinati diferenta dintre cel mai mare si cel mai mic numar ,,stelar” care

au cate 100 de cifre. Care este suma cifrelor acestui numar obtinut ca diferenta?

133

Page 50: format .pdf, 3.6 MB

Clasa a V-a

I. Un album foto are paginile numerotate de la 1 la 80. Pe fiecare pagina care estemultiplu de 2, dar nu si de 3, sunt cate 2 fotografii. Pe paginile care sunt multipli de3, dar nu si de 2 sunt 5 fotografii. In rest, paginile sunt ocupate cu cate o fotografie.Cate fotografii apar ın album?

II. Un grup de copii care participa la o aniversare constata ca media varstelor loreste egala cu numarul lor. Dupa ce apare, neinvitat, Ionel, ın varsta de 15 ani, copiiiconstata ca noul grup are aceeasi proprietate. Cati copii erau, initial, la aniversare?

III. Daca numarul 22013 are m cifre si 52013 are n cifre, sa se afle m+ n.

Clasa a VI-a

I. a) Aratati ca39

65− 85

119+

133

209− 69

115+

145

203− 217

341este un numar natural.

b) Intr-o zi din ianuarie 2013, din autogara Iasi pleaca ın cursa trei autocare catreSpania, Italia si Germania, care se reıntorc ın autogara dupa 6, 8 si respectiv 11 zile.Dupa ce stationeaza fiecare ın autogara cate 4 zile are loc o noua plecare. Aflatide cate ori ın cursul anului 2013, cele trei autocare pleaca ın cursa ın aceeasi zi dinautogara Iasi.

II. Fie multimile U =

§x ∈ N

7x+ 2

11∈ Nª

si V =

§y ∈ N

2y + 7

11∈ Nª.

a) Aratati ca: U = ∅ si V = ∅;b) Demonstrati ca

8x+ 29

11∈ N, oricare ar fi x ∈ U si

61y − 1

11∈ N, oricare ar fi

y ∈ V .c) Determinati U ∩ V.

III. Punctele D,E si F sunt situate pe latura (BC) a triunghiului ascutitunghicABC astfel ıncat AD ⊥ BC, (AE) ≡ (AF ), E ∈ [BD] si F ∈ [CD]. PuncteleB′ si C ′ apartin laturilor (AC), respectiv (AB) astfel ıncat ^ACC ′ ≡ ^BAE si^ABB′ ≡ ^CAF , iar punctul O este intersectia dreptelor BB′ si CC ′.

a) Demonstrati ca (BO) = (CO).b) Daca punctul O este egal departat de varfurile triunghiului ABC, determinati

pozitiile punctelor E si F pe (BC).c) Daca BB′ ∩ AE = M si CC ′ ∩ AF = N, iar (AM) ≡ (AN), atunci ABC

este un triunghi isoscel.

Clasa a VII-a

I. Ana, Dan si Stefan si-au cumparat scutere. Astazi Ana a mers cu o ora maimult decat Dan si cu o viteza cu 5 km/h mai mare decat a acestuia. Stefan a mers cudoua ore mai mult decat Dan si cu o viteza cu 10 km/h mai mare decat a acestuia.Ana a parcurs cu 70 km mai mult decat Dan. Aflati care este diferenta dintre distantaparcursa de Stefan si cea parcursa de Dan.

II. O gradina ın forma de triunghi are aria de 2 ari. Proprietarul vrea sa oımprejmuiasca cu un gard cu trei randuri de sarma. Stabiliti daca ıi ajung 18 dam desarma. Justificati raspunsul.

134

Page 51: format .pdf, 3.6 MB

III. Zece elevi organizeaza un turneu de sah dupa urmatoarele reguli:- fiecare joaca exact o partida cu ceilalti noua jucatori;- se acorda un punct pentru cel ce castiga, zero puncte pentru egalitate si se scade

un punct ınvinsului.La sfarsitul turneului se constata ca mai mult de 70% din meciuri s-au terminat

la egalitate. Sa se arate ca exista doi jucatori care au acelasi punctaj.

Clasa a VIII-a

I (O lacusta saltareata). Pe un plan raportat la un reper cartezian XOY , sejoaca o lacusta sarind din punct ın punct dupa regula ,,daca la un moment dat lacustaeste ın puctul A(a, b), atunci ea poate sari ın oricare dintre punctele de coordonate:(a±1, b±3) sau la (a±3, b±1), semnele + si − pot fi luate ın toate modurile posibile(prin urmare lacusta, din A, poate sari ın unul din cele opt puncte enumerate)”.

Daca la momentul initial, lacusta este ın O(0, 0) se cere:a) Stabiliti un traseu prin care lacusta ajunge ın punctul M(1, 1).b) Aratati ca lacusta, oricate sarituri ar face, nu ajunge ın punctul N(2013, 2014).c) Poate ajunge lacusta ın punctul P (2001, 2013)? Justificare.

II (O problema dulce). Radu si Andrei cumpara fiecare cate o cutie cu bom-boane de ciocolata ın forma de sfera. Cutiile au forma cubica cu aceeasi muchie n ∈N∗. Radu are ın cutie bomboane cu raza r, iar Andrei bomboane cu raza 2r, r ∈ N∗.In cutii bomboanele sunt asezate ın straturi astfel ıncat fiecarestrat sa contina numarul maxim posibil. Stiind ca 4r dividen, stabiliti care dintre cutiile cumparate cantareste mai mult.

III (O problema cu ,,torturi”). Cofetarul Gica inven-

teaza la Cofetaria ,,Prajitura minunata” situata peste drumde McDonald′s, un tort de ciocolata ce urmeaza a fi utilizat lafestivitatea de comemorare a ,,Regelui dulciurilor”. MesterulGica face un tort din 30 de cubulete de ciocolata pe care leasaza piramidal, unul peste altul ca ın figura alaturata astfelıncat axele verticale ale cubuletelor sa fie situate pe aceeasidreapta.

Se mai stie ca lungimile muchiilor cubuletelor sunt expri-mate ın decimetri prin numerele rationale:

11

10,12

11,13

12, . . . ,

39

38si, respectiv

40

39.

a) Aratati ca ınaltimea tortului construit de mesterul Gicanu depaseste 32 dm.

b) Daca mesterul Gica dispune de un calup de cioco-lata care se afla ıntr-o cutie plina ın forma de paralelipipeddreptunghic cu dimensiunile de 1 dm, 1 dm si 37 dm, aratatica acesta poate face tortul.

135

Page 52: format .pdf, 3.6 MB

Solutiile problemelor propuse ın nr. 1/2013

Clasele primare

P.255. Dorina are un numar de mere egal cu cel mai mare numar scris cu ocifra. Eu as avea cat Dorina, daca as mai avea 2 mere. Cate mere am eu?(Clasa I ) Inst. Maria Racu, Iasi

Solutie. 9− 2 = 7 (mere).

P.256. Doamna ınvatatoare a scris pe tabla sase litere m, sase litere i si saselitere n. Cate bastonase a folosit doamna ınvatatoare pentru scrierea acestor litere?(Clasa I ) Dumitrita Grigoriu, eleva, Iasi

Solutie. Litera m contine doua bastonase si o zala, litera n contine un bastonassi o zala, iar litera i este formata dintr-un singur bastonas. S-au folosit 24 bastonase.

P.257. Completati casutele goale din sirul 1, 1, 2, 1, 2, 3, 1, 2,,,,,,,.(Clasa I ) Mihaela Buleandra, eleva, Iasi

Solutie. Casutele trebuie completate cu cifrele: 3, 4, 1, 2, 3, 4, 5.

P.258. Aflati numerele a, b, c si d, stiind ca a− 18 = b− 15 = c− 8 = b− c− 1.(Clasa a II-a) Tatiana Ignat, eleva, Iasi

Solutie. Din egalitatea b− 15 = b− c− 1 obtinem 15 = c+ 1, de unde c = 14 sic− 8 = 6. Urmeaza ca a− 18 = 6, a = 24 si b− 15 = 6, b = 21.

P.259. Fie sirul de numere 1, 2, 3, . . . , 12. Se formeaza grupe de cate trei numereluate din acest sir. Cate dintre grupe pot avea suma numerelor egala cu 30?(Clasa a II-a) Ionut Airinei, elev, Iasi

Solutie. Observam ca ın aceasta suma nu putem avea termeni mai mici ca 7,deoarece 6 + 11 + 12 = 29 < 30. Analog, ın suma nu putem avea toti termenii maimari ca 9, caci 10 + 11 + 12 = 33 > 30. Ramane ca ıntr-o suma trebuie sa avem unuldintre numerele 7, 8, 9. Cercetand cazurile posibile, obtinem sumele: 7+11+12 = 30,8 + 10 + 12 = 30 si 9 + 10 + 11 = 30. Concluzionam ca exista numai trei grupe denumere care ındeplinesc conditiile problemei.

P.260. Bomboanele dintr-o punga se ımpart la cinci copii. Fiecare copil primestecel mult patru bomboane si cel mult trei copii au acelasi numar de bomboane. Careeste cel mai mare numar posibil de bomboane din punga?(Clasa a II-a) Alexandra Tololoi, eleva, Iasi

Solutie. In punga trebuie sa avem cel mult 4 + 4 + 4 + 3 + 3 = 18 bomboane.

P.261. Scrieti numarul 66 ca suma de numere naturale al caror produs este tot66. Cate solutii are problema? (Nu se va tine seama de ordinea termenilor.)(Clasa a III-a) Monica Raileanu, eleva, Iasi

136

Page 53: format .pdf, 3.6 MB

Solutie. 66=2·3·11 = 6·11 = 2·33 = 3·22. Avem: 66=1 + 1 + . . .+ 1| z 50 de 1

+2+3+11;

66 = 1 + 1 + . . .+ 1| z 49 de 1

+6+11; 66 = 1 + 1 + . . .+ 1| z 31 de 1

+2+33; 66 = 1 + 1 + . . .+ 1| z 41 de 1

+3+22.

P.262. Aratati ca un patrat din carton poate fi ımpartit, fara pierdere de material,ın 31 de patrate.(Clasa a III-a) Andreea Simona Simion, eleva, Iasi

Solutie. Dam urmatorul algoritm de ımpartire a unui patrat:

...

4 patrate (4+3x1) patrate (4+3x2) patrate

Deoarece 31 = 4 + 3× 9, ımpartirea este posibila.

P.263. Tatal termina o lucrare ın 10 ore, iar fiul ın 15 ore. In cate ore terminalucrarea cei doi, daca lucreaza ımpreuna?(Clasa a III-a) Paula Balan, eleva, Iasi

Solutie. Reprezentam volumul lucrarii printr-un segment, pe care ıl ımpartim ın30 de parti egale. Intr-o ora, tatal acopera 3 parti, iar fiul 2 parti, ın total 5 parti.Rezulta ca lucarea va fi terminata ın 30 : 5 = 6 ore.

P.264. Daca extragem o coala dintr-un ziar, observam ca suma celor patru numerecare indica paginile este 50. Puteti spune cu ce numere este paginata coala din mijloc?(Clasa a III-a) Codruta Filip, eleva, Iasi

Solutie. Suma celor patru numere care indica paginileunei coale este aceeasi. Pecoala din mijloc avem 4 numere consecutive cu suma 50. Acestea sunt: 11, 12, 13, 14.

P.265. Fie numerele nenule a, b, c, d astfel ıncat a+b = 18, b+c = 14, c+d = 10.Calculati suma 5a+ 6b+ 8c+ 7d.(Clasa a IV-a) Nicolae Ivaschescu, Craiova

Solutie. 5a+6b+8c+7d = 5×(a+b)+(b+c)+7×(c+d) = 5×18+14+7×10 == 90 + 14 + 70 = 174.

P.266. Maria are astazi de sase ori varsta pe care o avea cand fratele ei, Alexan-dru, avea varsta ei actuala. Cand ea va avea varsta de azi a fratelui ei, cei doi voravea ımpreuna 27 de ani. Ce varsta are acum Maria?(Clasa a IV-a) Irina Capraru, Iasi

Solutie. Sa figuram varstele celor doi frati, raportandu-ne la prezent (P):

Ap

pM

(P)

27 ani

137

Page 54: format .pdf, 3.6 MB

11p+ 16p = 27 ⇒ 27p = 27 ⇒ p = 1 (an).

Maria are acum 6× 1 an = 6 ani.

P.267. Pentru un grup de copii s-au adus de trei ori mai multe mandarine decatportocale. Fiecare copil trebuia sa primeasca 2 portocale si 5 mandarine, ınsa dingreseala doi copii au primit cate o mandarina ın plus. Stiind ca au ramas 19 portocalesi 92 mandarine, sa se afle cati copii sunt ın grup.

(Clasa a IV-a) Mariana Nastasia, eleva, Iasi

Solutie. Daca mandarinele ar fi fost ımpartite corect, atunci ar fi ramas 92+2 =94 mandarine. Daca notam cu ,,a” numarul copiilor, atunci 3(2a+ 19) = 5a+ 94, deunde a = 37 (copii).

P.268. Pe o foaie sunt scrise numerele de la 1 la 20. Sa se arate ca nu pot fiformate, din cele 20 de numere, grupe de cate patru numere astfel ıncat ın fiecaregrupa suma a doua numere sa fie triplul sumei celorlaltor doua.

(Clasa a IV-a) Andreea Bızdıga, eleva, Iasi

Solutie. 1 + 2 + 3 + . . . + 20 = (1 + 20)20 : 2 = 210. Daca numerele ar puteafi grupate cate 4, satisfacand conditia problemei, atunci suma numerelor din fiecaregrupa se ımparte exact la 4, deci si suma celor 20 numere se ımparte exact la 4, ceeace este fals.

Clasa a V-aV.158. Determinati restul ımpartirii numarului 201220132014 la 36 (fara a efec-

tua ımpartirea).

Iulian Oleniuc, elev, Iasi

Solutie. Numarul N este divizibil cu 9 (suma cifrelor find 18), cu 2, dar nu cu 4.Cum (2, 9) = 1, rezulta ca N este divizibil cu 18, dar nu cu 36. Prin urmare, restulımpartirii lui N la 36 este 18.

V.159. Determinati numerele xyz cu proprietatea ca (x+ y + z)3 = xyz.

Nicolae Ivaschescu, Craiova

Solutie. Cuburile perfecte de trei cifre sunt 125, 216, 343, 512 si 729. Calculandpentru fiecare dintre ele cubul sumei cifrelor, obtinem singura solutie (5+1+2)3 = 512.

V.160. Determinati ultimele doua cifre ale numarului A = 30+31+32+. . .+32013.

Mirela Marin, Iasi

Solutie. Avem: A = 1+3+32(1+3+32+33)+36(1+3+32+33)+ . . .+32010(1+3+32+33) = 4+9·40·(1+34+. . .+32008). Cei 503 termeni din paranteza se termina ın1, prin urmare suma lor se termina ın 3. Atunci A = 4+360·. . . 3 = 4+. . . 80 = . . . 84,adica ultimele doua cifre ale lui A sunt 84.

V.161. Se considera numerele rationale (nenegative) a, b, c, d si e cu proprietateaca a2 + b2 + c2 + d2 + e2 = 4. Demonstrati ca a3 + b3 + c3 + d3 + e3 ≤ 8.

Mihai Craciun, Pascani

Solutie. Evident ca a2 ≤ 4, deci a ≤ 2 si atunci a3 ≤ 2a2. Analog se arata cab3 ≤ 2b2, c3 ≤ 2c2, d3 ≤ 2d2 si e3 ≤ 2e2. Prin adunarea acestor relatii, obtinem caa3 + . . . + e3 ≤ 2(a2 + . . . + e2) = 2 · 4 = 8. Egalitatea se atinge atunci cand unuldintre cele cinci numere este 2, iar celelalte sunt 0.

138

Page 55: format .pdf, 3.6 MB

V.162. Vom spune ca un numar este istet daca cifra unitatilor sale este egala cusuma cuburilor celorlaltor cifre. Daca ordonam crescator numerele istete, determinatial cincisprezecelea termen al sirului obtinut.

Silviu Boga, IasiSolutie. Numere istete de doua cifre sunt 11 si 28, cele de trei cifre sunt 101, 112,

129, 208 si 219, iar cele de patru cifre sunt 1001, 1012, 1029, 1102, 1209, 2008, 2019 si2109. Rezulta ca al cincisprezecelea numar istet este 2109.

V.163. Putem aseza pe un cerc numerele 1, 2, 3, . . . , n (n ∈ N, n ≥ 3) astfel ıncatsuma oricaror trei numere alaturate sa fie numar impar?

Gheorghe Iurea, IasiSolutie. Raspunsul este negativ; pentru a demonstra acest lucru, sa presupunem

contrariul. Evident, nu este posibil ca orice doua numere alaturate sa aiba paritatidiferite (ıntr-o secventa de tipul i, p, i, suma este para). Daca exista doua numerealaturate impare, ar trebui ca toate sa fie impare, imposibil. Daca exista doua numerealaturate pare, asezarea numerelor va fi de tipul p, p, i, p, p, i, p, p, i, . . .; vom avea pe

cerchn3

inumere impare si n−

hn3

inumere pare si, cum

hn3

i< n−

hn3

i, ajungem la

o contradictie si ın aceasta situatie.

V.164. Scriem toate numerele naturale de trei cifre pe cate un cartonas si intro-ducem cele 900 de cartonase ıntr-o cutie. Care este numarul minim de cartonase pecare trebuie sa le extragem, fara a ne uita la ele, pentru a fi siguri ca vom obtine celputin sapte numere cu aceeasi suma a cifrelor?

Sergiu Prisacariu, IasiSolutie. Sumele posibile ale cifrelor numerelor de pe cartonase sunt 1, 2, 3, . . . , 27.

Exista un singur cartonas cu suma cifrelor 1 (anume 100) si doar unul cu suma cifrelor27 (anume 999). Avem ın cutie cate trei cartonase cu suma cifrelor 2 (anume 101, 110si 200) sau 26 (anume 899, 989 si 998). Exista cate sase cartonase cu suma cifrelor3 (anume 111, 102, 120, 201, 210 si 300) sau 25 (anume 799, 979, 997, 889, 898 si 988).Celelalte sume ale cifrelor (4, 5, . . . , 24) apar fiecare pe mai mult de sapte cartonase.In concluzie, numarul minim de cartonase extrase trebuie sa fie 2·1+2·3+2·6+7 = 27.

Clasa a VI-a

VI.158. Determinati perechile de numere ıntregi (x, y) care verifica simultanconditiile x+ y = 6 si 2xy − 3x− 3y + 2 = 0.

Eugeniu Blajut, Bacau

Solutie. Din a doua relatie rezulta ca y =3x− 2

2x− 3. Cum y ∈ Z, se impune

conditia 2x−3|3x−2 si atunci 2x−3|2(3x−2)−3(2x−3), adica 2x−3|5, prin urmarex ∈ −1, 1, 2, 4. A doua relatie este verificata de perechile (−1, 1), (1,−1), (2, 4) si(4, 2) si, dintre ecestea, doar ultimele doua verifica si prima conditie. In concluzie,S = (2, 4); (4, 2).

VI.159. Determinati numerele ıntregi a, b, c, d, daca 3a + 3b + 3c + 3d = 10.Vasile Chiriac, Bacau

Solutie. Pentru ınceput, sa presupunem ca a ≤ b ≤ c ≤ d. Daca a ≤ −2, nuexista b, c, d ∈ Z pentru care suma 3a + 3b + 3c + 3d sa fie numar ıntreg, iar daca

139

Page 56: format .pdf, 3.6 MB

a = −1, trebuie ın mod necesar sa avem b = c = −1, altfel 3a + 3b + 3c + 3d ∈ Q\Z.Deducem ca singura solutie avand componente negative (ın ipoteza asumata) estea = b = c = −1, d = 2. Daca a ≥ 0, atunci 3a + 3b + 3c ≥ 1 + 1 + 1 = 3, deci 3d ≤ 7;rezulta ca d = 1 si, apoi, se arata usor ca a = 0, b = c = d = 1. In concluzie, solutiileecuatiei date sunt (−1,−1,−1, 2), (0, 1, 1, 1) si ınca sase care se obtin din acestea prinpermutari circulare.

VI.160. Impartind 2013 la a, obtinem catul b si restul c. Determinati numerelenaturale a, b si c, stiind ca reprezinta lungimile laturilor unui triunghi isoscel.

Ioana Maria Popa, eleva, IasiSolutie. Avem ca 2013 = a · b + c, cu c < a; din ipoteza problemei, rezulta

ca a = b sau b = c. In primul caz, 2013 = a2 + c, cu a2 < a2 + c < a(a + 1).Cum 442 = 1936, 44 · 45 = 1980 < 2013 si 452 = 2025 > 2013, aceasta situatienu conduce la nicio solutie. In al doilea caz, 2013 = c(a + 1), cu c < a; obtinemca (a, c) ∈ (2012, 1); (670, 3); (182, 11); (60, 33). Se verifica inegalitatea triunghiuluidoar daca a = 60, b = c = 33.

VI.161. Determinati valorile naturale ale lui n pentru care exista numarul naturalx astfel ıncat (2x+ 9, 3x+ 1) = 5n.

Gheorghe Iacob, PascaniSolutie. Daca 5n|2x+9 si 5n|3x+1, atunci 5n|3(2x+9)−2(3x+1), adica 5n|25;

rezulta ca n ∈ 0, 1, 2. Daca x = 0, avem ca (2x + 9, 3x + 1) = (9, 1) = 1 = 50.Daca x = 3, atunci (2x + 9, 3x + 1) = (15, 10) = 51. Pentru x = 8, avem ca(2x + 9, 3x + 1) = (25, 25) = 52. In concluzie, toate cele trei valori posibile ale lui nsunt convenabile.

VI.162. Un calculator defect mai face doar patru operatii: poate ınmulti un numarcu 2 sau cu 5 sau poate ımparti un numar la 2 sau la 5, daca ımpartirea se efectueazaexact. Un copil pleaca de la numarul 20 si face aleator 2013 astfel de operatii. Esteposibil ca rezultatul final sa fie tot 20?

Petre Batranetu, GalatiSolutie. Observam ca 20 = 22 · 5, cu suma exponentilor numar impar. La

fiecare operatie, suma exponentilor se mareste sau se micsoreaza cu 1, deci ısi schimbaparitatea. Dupa un numar impar de operatii (2013), suma exponentilor va fi para,prin urmare rezultatul final nu poate fi tot 20.

VI.163. Fie M si N mijloacele laturilor AB, respectiv AC ale triunghiului ABC.Cercul de diametru AB intersecteaza dreapta MN ın punctele D si E. Demonstrati

ca BD si BE sunt bisectoarele (interioara si exterioara) ale unghiului ÕABC.Ioan Sacaleanu, Harlau

Solutie. Presupunem, ca ın figura, ca D ∈ IntABC. Triunghiul MBD esteA

B C

D

M NE

isoscel, ıntrucat MB si MD sunt raze ale cercului;

rezulta ca ÖMBD ≡ ÖMDB. Pe de alta parte, cum

MN∥BC, avem ca ÖMDB ≡ ÕDBC (alterne interne).

Deducem ca ÖMBD ≡ ÕDBC, adica BD este bisectoarea

interioara a unghiului ÕABC. Analog se arata ca BE

este bisectoarea exterioara a lui ÕABC.

140

Page 57: format .pdf, 3.6 MB

A

B CD

G

E

H

F

L

K

H

G

VI.164. Pe laturile AC si AB ale triunghiu-lui ABC se considera punctele E, respectiv F ast-

fel ıncat m(ÕABE) < m(ÕCBE) si m(ÕACF ) <

m(ÕBCF ). Fie D un punct pe latura BC si G ∈(AB), H ∈ (AC) pentru care DG ⊥ BE si DH ⊥CF. Demonstrati ca AG+AH > AB +AC −BC.

Petru Asaftei, IasiSolutie. Notam L = DG ∩ BE, K =

DH ∩ CF si fie G′,H ′ simetricele punctelor G si

H fata de L, respectiv fata de K. Cum m(ÖG′BE) = m(ÕABE) < m(ÕCBE), rezultaca G′ ∈ (DL) si atunci BG = BG′ < BD; analog se arata ca CH < CD. Astfel,AG+AH = AB +AC − (BG+CH) > AB +AC − (BD+DC) = AB +AC −BC.

Clasa a VII-a

VII.158. Determinati numerele naturale m cu proprietatea ca m(m+17) se poatescrie ca produs de doua numere naturale consecutive.

Lucian Tutescu si Petrisor Rocsoreanu, CraiovaSolutie. Fie n ∈ N cu proprietatea ca m(m + 17) = n(n + 1); atunci 4m2 +

68m + 289 − 288 = 4n2 + 4n + 1, deci (2m + 17)2 − (2n + 1)2 = 288. Obtinem ca(m+n+9)(m− n+8) = 72 si, cum m− n+8 < m+ n+9 si cele doua numere suntnaturale, de paritati diferite, avem de studiat trei posibilitati: (m−n+8,m+n+9) ∈(1, 72); (3, 24); (8, 9). Solutiile problemei sunt m ∈ 0, 5, 28.

VII.159. Daca m,n, p sunt numere reale astfel ıncat m2+n2+p2 = 3, demonstratica

√2m2 + 5 +

√2n2 + 5 +

p2p2 + 5 ≤ 3

√7.

Mihai Craciun, PascaniSolutie. Notam x =

√2m2 + 5, y =

√2n2 + 5 si z =

p2p2 + 5; atunci x2 + y2 +

z2 = 2(m2 + n2 + p2) + 15 = 21. Folosind inegalitatea dintre media aritmetica si ceapatratica, obtinem ca (x + y + z)2 ≤ 3(x2 + y2 + z2) = 63, deci x + y + z ≤ 3

√7.

Egalitatea se atinge pentru m = n = p = 1.VII.160. Determinati triunghiurile dreptunghice cu lungimile laturilor exprimate

prin numere naturale, care au aria egala cu 24.Neculai Stanciu, Buzau

Solutie. Daca laturile a, b, c ale unui triunghi dreptunghic sunt numere naturale,atunci a = x2 + y2, b = x2 − y2 si c = 2xy, unde x, y ∈ N∗, x > y. Aria triunghiului

este A =bc

2= xy(x − y)(x + y) > y · y · 1 · y, prin urmare y3 < 24, deci y ∈ 1, 2.

Daca y = 1, atunci x(x + 1)(x − 1) = 24, de unde x = 3. Daca y = 2, obtinem ca2x(x − 2)(x + 2) = 24, fara solutii ın numere naturale. In concluzie, x = 3 si y = 1,asadar triunghiul cautat are laturile a = 10, b = 8, c = 6.

VII.161. Se considera trapezul ABCD cu baza mare AB egala cu diagonala AC.Fie E = AD ∩ BC, O = AC ∩ BD, iar P ∈ (AD) este astfel ıncat OP∥AB.Demonstrati ca CP si CE sunt bisectoarele interioara, respectiv exterioara ale un-

ghiului ÕACD.

Claudiu-Stefan Popa, Iasi

141

Page 58: format .pdf, 3.6 MB

Solutie. Din asemanari evidente avem caDP

PA=

DO

OB=

DC

AB. Cum AB = AC,

A B

CD

P

O

Eobtinem caDP

PA=

DC

ACdeci, conform recipro-

cei teoremei bisectoarei aplicata ın triunghiulACD, rezulta ca CP este bisectoarea (inte-

rioara) unghiului ÕACD. Tinand seama defaptul ca triunghiul ABC este isoscel, avem:

m(ÕABC) + m(ÕBCD) = 180 ⇒ m(ÕABC) +

m(ÕACB) + m(ÕACD) = 180 ⇒ 2m(ÕACB) +

2m(ÕACP ) = 180 ⇒ m(ÕACB) + m(ÕACP ) =

90 ⇒ m(ÕBCP ) = 90, prin urmare CP ⊥ CE.

De aici rezulta ca CE este bisectoarea exterioara a unghiului ÕBCD.

VII.162. Demonstrati ca diagonala unui trapez isoscel este mai lunga decat liniamijlocie a acestuia.

Nicolae Bourbacut, Sarmizegetusa

Solutia 1. Fie P proiectia varfului C pe baza mare AB; ın triunghiul

A B

CD

O

M N

P

dreptunghic PAC, ipotenuza AC este mai lunga decatcateta AP . Pe de alta parte, PN este mediana ıntriunghiul dreptunghic PBC, asadar PN = BN , de

unde ÕNPB ≡ ÒB ≡ bA. Atunci AM∥PN si, cumAP∥MN, rezulta ca APNM este paralelogram. De-ducem ca MN = AP si, de aici, urmeaza concluziaproblemei.

Solutia 2 (Adina Onofrei, eleva, Rosiori (Bacau)). Stim ca ıntr-un triunghimediana este mai mica decat semisuma laturilor care pleaca din acelasi varf. Folosindsi inegalitatea triunghului, avem:

MN < OM +ON <OD +OA

2+

OC +OB

2=

AC +BD

2= AC.

VII.163. Fie D un punct pe latura BC a triunghiului ABC. Paralela prin D laAC taie AB ın E, iar paralela prin D la AB taie AC ın F . Daca M = CE ∩BF,aratati ca suprafetele AEMF si BMC sunt echivalente.

Dan Popescu, Suceava

Solutie. Notam N = AD∩CE, P = AD∩BF. In trapezele ABDF si ACDEA

B CD

E

F

MN

P

..

.

au loc egalitatile ABPD = AAPF , respectivADNC = AANE . Adunand aceste egalitati sireducand din ambii membri AMNP , obtinem caABMND+ADNC = AAPF+AAEMP , deciABMC =AAEMF . Remarcam ca aria triunghiului MNPpoate fi si nula, ın cazul ın care D este mijlocullui BC.

Nota. Rezultatul constituie o generalizare a

142

Page 59: format .pdf, 3.6 MB

problemei 26567 din Gazeta Matematica 2/2012, ın care D este piciorul bisectoareidin A.

VII.164. Fie ABCD un patrulater convex, ale carui diagonale se intersecteazaın O. Daca oricare ar fi punctele E ∈ (AB si F ∈ (CD cu AE = CF , avem caO ∈ EF , demonstrati ca ABCD este paralelogram.

Gheorghe Iurea, Iasi

Solutie. Presupunem, prin absurd, ca AB ∦ CD si fie B′ ∈ BC pentru care

A

B

B

E E

M M

O

C

DFFAB′∥CD. Consideram punctele E si F ca ın enunt, iar

M = EF ∩ AB′. Din AOM ≡ COF (U.L.U.)rezulta ca AM = CF si, cum CF = AE, triunghiulAME este isoscel. Procedam analog pentru alte douapuncte E′, F ′ ca ın enunt si obtinem ca AM ′E′ este

isoscel. Astfel, m(ÖAME) = m(×AM ′E′) =1

2(180 −

m(ÖBAB′)), prin urmareME∥M ′E′, ceea ce contrazicefaptul ca ME ∩M ′E′ = O. Ramane ca AB∥CD.

Pentru o pereche de puncte E si F ca ın enunt, avem ca AECF este paralelogram(AE = CF,AE∥CF ), deci O va fi mijlocul diagonalei AC a patrulaterului ABCD cuAB∥CD. De aici, deducem imediat ca ABCD este paralelogram.

Clasa a VIII-a

VIII.158. Rezolvati ın R ecuatia27x2 + 54x+ 15

98x2 − 84x+ 10=

147x2 − 126x+ 24

18x2 + 36x+ 16.

Constantin Dragomir, Pitesti

Solutie. Inmultind cu2

3ambii membri ai ecuatiei si apoi scazand 1 din ambii

membri ai ecuatiei obtinute, deducem ca−40x2 + 60x

49x2 − 42x+ 5=

40x2 − 60x

9x2 + 18x+ 8. De aici,

40x2 − 60x = 0 cu solutiile x1 = 0 si x2 =3

2sau 49x2 − 42x + 5 = −9x2 − 18x − 8,

adica 58x2 − 24x+10 = 0, ecuatie care nu are solutii reale. Observand ca valorile lui

x obtinute nu anuleaza numitorii fractiilor, rezulta ca S =

§0,

3

2

ª.

VIII.159. Demonstrati ca 1 + 3x+ 6x2 + 9x3 + 9x4 >1

4, ∀x ∈ R.

Ionel Tudor, Calugareni

Solutia 1. Membrul stang se descompune ın factori ca (1 + 3x2)(1 + 3x + 3x2).

Observam ca 1 + 3x2 ≥ 1 (cu egalitate pentru x = 0), iar 1 + 3x + 3x2 ≥ 1

4(cu

egalitate cand x = −1

2). Inmultind membru cu membru aceste inegalitati si tinand

seama de faptul ca egalitatile nu se ating simultan, obtinem cerinta problemei.

Solutia 2 (Adina Onofrei, eleva, Rosiori (Bacau)). Avem succesiv: 36x4 +

36x3+24x2+12x+4 > 1 ⇔ 12x4+12x3+4x2+4x2+4x+1 > 0 ⇔ 12x2

x2 + x+

1

3

+

143

Page 60: format .pdf, 3.6 MB

(2x+ 1)2 > 0 ⇔ 12x2

x+

1

2

2

+1

12

+ (2x+ 1)2 > 0, ultima fiind evidenta.

VIII.160. Se considera dreapta fixa d, punctul fix A /∈ d si planul variabil α carecontine dreapta d. Notam cu M proiectia punctului A pe planul α. Determinati loculgeometric al lui M .

Aida-Andreea Iacob, Iasi

Solutie. Fie π planul care contine punctul A si este perpendicular pe dreaptad si B = d ∩ π; evident, B este un punct fix. Dreapta AM este inclusa ın planulπ, iar triunghiul ABM este dreptunghic ın M . Rezulta ca M apartine cercului C dediametru AB, inclus ın planul π. Reciproc, se observa usor ca, pentru orice punctM al lui C, exista un plan α = (d,MB) astfel ıncat M este proiectia lui A pe α. Inconcluzie, locul geometric dorit este cercul C.

VIII.161. Se dau zece cutii cubice cu muchiile de 1 cm, 2 cm, . . ., 10 cm, fiecarefiind umpluta cu cuburi de muchie 1 cm. Spunem ca p cutii formeaza un bicub dacauna dintre ele contine tot atatea cuburi cate contin celelalte p − 1 la un loc. Putemforma un bicub folosind cateva dintre cutiile date?

Geanina Havarneanu, Iasi

Solutie. Raspunsul este afirmativ: 13 + 33 + 43 + 53 + 63 = 73. Mai mult, oanaliza (plicticoasa!) arata ca aceasta este singura solutie a problemei.

VIII.162. Determinati ultimele trei cifre ale numarului natural A = 3733+3743+3755 + . . .+ 6283.

Mihai Haivas, Iasi

Solutie. Daca a, b ∈ Z, atunci a3 + b3 = M(a+ b). Astfel, A = (3733 + 6273) +(3743 +6263)+ . . .+(4993 +5013)+ 5003 +6283 = M1000 +6283 = M1000 + . . . 172 =. . . 172.

VIII.163. Determinati valorile ıntregi ale lui m pentru care numarul

É4m+ 3

m− 5este rational.

Bogdan Chiriac, Bacau

Solutie. Fie

É4m+ 3

m− 5=

a

b, cu a, b ∈ N∗, (a, b) = 1 (evident ca

É4m+ 3

m− 5

este strict pozitiv pentru m ∈ Z). Obtinem ca m =−5a2 − 3b2

4b2 − a2= 5 − 23b2

4b2 − a2.

Cum (4b2 − a2, b2) = (a2, b2) = 1, rezulta ca 4b2 − a2 divide 23, deci (2b − a)(2b +a) ∈ ±1,±23. Dintre sistemele care se pot forma, singurul cu solutii naturale este2b− a = 1, 2b+ a = 23, prin urmare a = 11, b = 6, adica m = −31.

VIII.164. Fie a, b, c, d patru numere reale strict pozitive astfel ıncat ab(c+ d) ≥(a+ b)cd si ab+ cd ≥ (a+ b)(c+ d). Comparati numerele a+ b si c+ d.

Dan Nedeianu, Drobeta Tr. Severin

Solutie. Vom demonstra ca a + b > c + d; pentru aceasta, sa presupunem, prinabsurd, ca a + b ≤ c + d. Inmultind membru cu membru inegalitatile din enunt,obtinem ca ab(c + d)(ab + cd) ≥ (a + b)2 · cd · (c + d). Insa (a + b)2 ≥ 4ab si atunciab(c + d)(ab + cd) ≥ 4abcd(c + d), deci ab + cd ≥ 4cd, asadar ab ≥ 3cd. Pe de alta

144

Page 61: format .pdf, 3.6 MB

parte, ab + cd ≥ (a + b)(c + d) ≥ (a + b)2 ≥ 4ab, de unde cd ≥ 3ab. Ar rezulta cacd ≥ 3ab ≥ 9cd, imposibil; ramane astfel adevarata afirmatia initiala.

Clasa a IX-a

IX.136. Fie a, b, c ∈ (0,∞) cu a · b · c = 1. Demonstrati ca

ab

b2 + c2+

bc

c2 + a2+

ca

a2 + b2≥ 9

a3 + b3 + c3 + a+ b+ c.

Sven Cortel si Kinga Ratiu, elevi, Satu MareSolutie. Conform inegalitatii lui Bergstrom, membrul stang al inegalitatii din

enunt este cel putin egal cu(√ab+

√bc+

√ca)2

2(a2 + b2 + c2). Din inegalitatea mediilor, avem ca

√ab+

√bc+

√ca ≥ 3 3

√abc = 3, iar 2a2 ≤ a+ a3, 2b2 ≤ b+ b3, 2c2 ≤ c+ c3, de unde

rezulta cerinta problemei. Egalitatea se atinge pentru a = b = c = 1.

IX.137. Fie ABCD un patrulater inscriptibil. Notam cu H1,H2,H3 si H4

ortocentrele triunghiurilor DAB, ABC, BCD respectiv CDA si cu G1, G2, G3 siG4 centrele de greutate ale triunghiurilor AH1B, BH2C, CH3D respectiv DH4A.Demonstrati ca patrulaterele ABCD si G1G2G3G4 au acelasi centru de greutate dacasi numai daca ABCD este dreptunghi.

Florin Stanescu, GaestiSolutie. In raport cu centrul O al cercului circumscris patrulaterului ABCD,

vectorii de pozitie ai punctelor din problema sunt: −→rH1 = −→rD + −→rA + −→r B ,−→rH2 =

−→rA +−→rB +−→rC , −→rH3 = −→rB +−→rC +−→rD, −→rH4 = −→rC +−→rD +−→rA; −→rG1 =1

3(2−→rA + 2−→rB +−→rD),

−→rG2 =1

3(2−→rB+2−→rC+−→rA), −→rG3 =

1

3(2−→rC+2−→rD+−→rB), −→rG4 =

1

3(2−→rD+2−→rA+−→rC). Centrul

de greutate al patrulaterului ABCD are vectorul de pozitie −→r1 =1

4(−→rA+−→rB+−→rC+−→rD),

iar centrul de greutate al patrulaterului G1G2G3G4 are vectorul de pozitie −→r2 =5

12(−→rA +−→rB +−→rC +−→rD). Evident ca −→r 1 = −→r2 ⇔ −→rA +−→rB +−→rC +−→rD =

−→0 ⇔ ABCD

este dreptunghi; propunem cititorului sa demonstreze riguros ultima echivalenta.

IX.138. Fie M un punct variabil ın interiorul sau pe laturile triunghiului ascu-titunghic ABC. Aratati ca MB2 +MC2 −MA2 ≤ AB2 +AC2.

Ovidiu Pop, Satu MareSolutie. Fie D simetricul punctului A fata de mijlocul O al segmentului BC.

A

B C

D

O

M

Folosind teorema medianei ın triunghiurile MBC si

MAD, obtinem ca MO2 =2(MB2 +MC2)−BC2

4=

2(MA2 +MD2)−AD2

4, de unde MB2 + MC2 −

MA2 = MD2 +BC2 −AD2

2. Cum unghiul A este

ascutit, cea mai mare distanta dintre doua puncte situ-ate pe laturile sau ın interiorul paralelogramului ABDC

145

Page 62: format .pdf, 3.6 MB

este diagonala AD. Rezulta ca MD2 ≤ AD2 = 4AO2 = 2(AB2 +AC2)−BC2. Ast-

fel, MB2 +MC2 −MA2 ≤ AD2 +BC2 −AD2

2=

BC2 +AD2

2= AB2 +AC2, ceea

ce trebuia demonstrat.

IX.139. Se considera triunghiul dreptunghic ABC cu m( bA) = 90 si m(ÒC) = 30.Fie O si F picioarele medianei, respectiv ınaltimii din A, iar E este un punct astfelıncat F ∈ (AE), FE = FB. Notam cu M simetricul lui A fata de E si cu Dintersectia dreptelor MO si AC. Demonstrati ca CO = CD.

Eugeniu Blajut, BacauSolutia 1. Fie AB = 2a; se observa imediat ca OF = FB = FE = a, AF = a

√3,

A B

C

D

N

M

O

FE

ME = a(1 +√3), FM = a(2 +

√3). In triunghiul

dreptunghic FOM, avem ca tgÖMOF =MF

OF=

a(2 +√3)

a= 2 +

√3, prin urmare m(ÖMOF ) = 75.

Rezulta ca m(ÕCOD) = 75, de unde m(ÕCDO) =180 − 30 − 75 = 75. Astfel, triunghiul COD esteisoscel, cu CO = CD.

Solutia 2 (a autorului). Notam cu N mijloculsegmentului OM ; atunci EN este linie mijlocie ın

AOM, deci NE =1

2OA = a si NE∥AO. De-

ducem ca m(ÕNEF ) = 180 − m(ÕOAF ) = 150, iar triunghiul EFN este isoscel,

cu EF = NE = a. Obtinem ca m(ÕNFE) = 15, asadar m(ÕOFN) = 75. Pe dealta parte, NF este mediana ipotenuzei ın FOM, prin urmare NF = NO, de unde

m(ÕNOF ) = 75 si solutia continua ca mai sus.

IX.140. Determinati numerele naturale nenule n pentru care

(x+ y)n − (xn + yn) = nxy(x+ y)(x2 + xy + y2)n−32 , ∀x, y ∈ R.

Ionel Tudor, Calugareni

Solutie. Pentru x = y = 1, obtinem ca 2n − 2 = 2n · 3n−32 . Aceasta egalitate

este falsa daca n este par, ıntrucat membrul stang este numar rational, iar ce drepteste irational. Fie deci n = 2k + 1, k ∈ N, astfel ıncat 22k+1 − 2 = 2(2k + 1)3k−1,adica 4k = 1+(2k+1)3k−1; aceasta egalitate se verifica pentru k ∈ 1, 2, 3 si nu esteadevarata pentru k = 0 si k ≥ 4, ıntrucat se dovedeste usor, prin inductie matematica,inegalitatea 4k > 1+(2k+1)3k−1, ∀k ≥ 4. Inseamna ca singurele valori ale lui n carear putea fi solutii ale problemei sunt n ∈ 3, 5, 7. Identitatile:

(x+ y)3 − (x3 + y3) = 3xy(x+ y);

(x+ y)5 − (x5 + y5) = 5xy(x+ y)(x2 + xy + y2);

(x+ y)7 − (x7 + y7) = 7xy(x+ y)(x2 + xy + y2)2

fiind adevarate pentru orice x, y∈R, rezulta ca valorile cautate ale lui n sunt 3, 5 si 7.

146

Page 63: format .pdf, 3.6 MB

Clasa a X-a

X.136. Fie numerele naturale nenule a1, a2, . . ., an, n≥2. Determinati numerele

complexe z1, z2, . . ., zn, de modul 1, cu proprietatea canP

i=1

|1−zaii |+

nPi=1

|1+zaii |=2n

√2.

Sven Cortel, elev, Satu MareSolutie. Fie a ∈ N∗ si z = cos 2t+ i sin 2t un numar complex de modul 1; atunci

|1−za| =È(1− cos 2at)2 + sin2 2at =

√2− 2 cos 2at = 2| sin at| si, analog, |1+za| =

2| cos at|, deci |1− za|+ |1+ za| = 2(| sin at|+ | cos at|) ≤ 4

rsin2 at+ cos2 at

2= 2

√2.

Egalitatea se atinge cand sin at = ± cos at, adica pentru t =π

4a+

a, k ∈ Z sau

t =3π

4a+

a, k ∈ Z.

Din cele de mai sus rezulta ca, ın general, are loc inegalitateanP

i=1

|1 − zaii | +

nPi=1

|1 + zaii | ≤ 2n

√2. Intrucat se atinge egalitatea, obtinem ca zi = cos

(4k + 1)π

2ai+

i sin(4k + 1)π

2ai, k ∈ Z sau zi = cos

(4k + 3)π

2ai+ i sin

(4k + 3)π

2ai, k ∈ Z, oricare ar fi

i ∈ 1, 2, . . . , n.X.137. Fie a, b ∈ N. Rezolvati ın C ecuatia |z + a|+ |z − b|+ |a− b| = 2.

Gheorghe Iurea, IasiSolutie. Daca z ∈ C verifica relatia din enunt, atunci 2 = |z+a|+|z−b|+|a−b| ≥

|z+a− z+ b|+ |a− b| = |a+ b|+ |a− b|. Din a, b ∈ N si |a+ b|+ |a− b| ≤ 2, rezulta ca(a, b) ∈ (0, 0); (1, 0); (0, 1); (1, 1). Daca a = b = 0, ecuatia |z| = 1 admite solutiilez = cos t + i sin t, t ∈ R. Daca a = 1, b = 0, ecuatia devine |z + 1| + |z| = 1, cumultimea solutiilor S = [−1, 0]. Daca a = 0, b = 1, ecuatia devine |z|+ |z−1| = 1, cumultimea solutiilor S = [0, 1]. In sfarsit, daca a = b = 1, ecuatia |z + 1|+ |z − 1| = 2are solutiile S = [−1, 1]. Pentru alte valori ale numerelor a si b, ecuatia nu are solutii.

X.138. Rezolvati ecuatia

log25(24x + 22−x) + 24x + 22−x = 4 + log5(2

8x + 23x+3 + 24−2x).

Dan Nedeianu, Drobeta Tr. SeverinSolutie. Deoarece (24x+22−x)2 = 28x+23x+3+24−2x, ecuatia se poate scrie sub

forma [log5(24x + 22−x)− 1]2 = 5− (24x + 22−x) si atunci 24x + 22−x ≤ 5. Insa, din

inegalitatea mediilor, avem ca 24x+22−x = 24x+1

2x+

1

2x+

1

2x+

1

2x≥ 5, cu egalitate

cand 24x = 2−x, i.e. x = 0. Se verifica faptul ca x = 0 este solutie a ecuatiei, prinurmare S = 0.

X.139. Rezolvati ecuatiile:a) [ n

√n+

n√n2] = n;

b) [ n−1√n+

n−1√n2] = n.

Ionel Tudor, Calugareni

147

Page 64: format .pdf, 3.6 MB

Solutie. a) Pentru a avea sens radicalii, se impune ca n ∈ N, n ≥ 2. Prin inductiematemtica se arata ca 3n ≥ n3, ∀n ∈ N, de unde n

√n ≤ 3

√3, ∀n ∈ N, n ≥ 2. Rezulta

can√n2 ≤ 3

√9, prin urmare n

√n+

n√n2 ≤ 3

√3+ 3

√9 < 2+ 3 = 5. Deducem ca ecuatia

data nu are solutii n ≥ 5. Verificand direct valorile n ∈ 2, 3, 4, gasim ca unica solutiea ecuatiei este n = 3.

b) Se impune conditia n ∈ N, n ≥ 3. Se verifica faptul ca n = 3 si n = 5 nu verificaecuatia, iar n = 4 este solutie; vom arata ca nu exista solutii n ≥ 6. Prin inductiematematica se demonstreaza ca 3n−1 > n3, ∀n ≥ 6, deci n−1

√n < 3

√3, ∀n ∈ N, n ≥ 6.

Atuncin−1√n2 < 3

√9, asadar n−1

√n +

n−1√n2 < 3

√3 + 3

√9 < 2 + 3 < 6. Ramane ca

ecuatia data are unica solutie n = 4.

X.140. Daca ın triunghiul ABC, cu notatiile uzuale, are loc egalitatea a2 + b2 +c2 = p2 + 9r2, atunci triunghiul este echilateral.

Catalin Calistru, Iasi

Solutie. Observam ca pr2 =S2

p= (p−a)(p−b)(p−c) = p3−(a+b+c)p2+(ab+

bc+ca)p−abc = −p3+(ab+bc+ca)p−4Rrp, de unde ab+bc+ca = p2+r2+4Rr. Pede alta parte, ab+ bc+ ca ≤ a2+ b2+ c2 = p2+9r2 si atunci p2+ r2+4Rr ≤ p2+9r2,prin urmare R ≤ 2r. Rezulta ca se atinge egalitatea ın inegalitatea lui Euler R ≥ 2r,adica triunghiul este echilateral.

Clasa a XI-a

XI.136. Fie A,B ∈ M2(R) astfel ıncat detA = 9, detB = 4 si det (A+ B) = 1.Determinati x ∈ R pentru care numarul det(A+ xB) este minim.

Razvan Ceuca, student, IasiSolutie. Fie f(x) = det(A + xB) = detA + mx + (detB)x2 = 4x2 + mx + 9.

Cum f(1) = det(A + B) = 1, obtinem ca 4 + m + 9 = 1, deci m = −12. Atunci

f(x) = 4x2 − 12x+9 = (2x− 3)2 ≥ 0, valoarea minima 0 atingandu-se pentru x =3

2.

XI.137. Fie a ∈ (0, 1) ∪ [2,∞) si sirul (xn)n≥1 definit prin x1 =√a, xn+1 =

√a+ xn, ∀n ≥ 1. Notam yn =

√2a− xn+1

a− xn, ∀n ∈ N∗.

a) Demonstrati ca yn >2

2√2a+ 1 +

√1 + 4a

, ∀n ∈ N∗.

b) Daca a ∈ (0, 1), aratati ca, ın plus, yn <1√

2a+ a, ∀n ∈ N∗.

Gheorghe Costovici, Iasi

Solutie. a) Se demonstreaza, prin inductie matematica, faptul ca xn <1 +

√1 + 4a

2,

∀n ∈ N∗. Atunci yn =

√2a−

√a+ xn

a− xn=

a− xn

(a− xn)(√2a+

√a+ xn)

=1√

2a+ xn+1

>

1√2a+ 1+

√1+4a2

=2

2√2a+ 1 +

√1 + 4a

, ∀n ∈ N∗.

b) Se demonstreaza prin inductie ca, pentru a ∈ (0, 1), xn > a, ∀n ∈ N∗. Atunci

yn =1√

2a+ xn+1

<1√

2a+ a, ∀n ∈ N∗.

148

Page 65: format .pdf, 3.6 MB

XI.138. Fie (xn)n≥1 un sir de numere reale monoton si convergent. Aratati ca

sirul (yn)n≥1, yn =nP

k=1

(xk+1 − xk)2013 are aceeasi monotonie cu (xn)n≥1 si este, de

asemenea, convergent.

Silviu Boga, Iasi

Solutie. Cum yn+1 − yn = (xn+2 − xn+1)2013, diferenta xn+2 − xn+1 are semn

constant pentru n ∈ N∗ si 2013 este numar impar, rezulta ca (yn)n≥1 are aceeasimonotonie cu (xn)n≥1. Datorita convergentei lui (xn)n≥1, sirul zn = (xn+1 − xn)

2012

este marginit superior de M ; atunci |yn| =nP

k=1

|xk+1 − xk|zk ≤nP

k=1

|xk+1 − xk|M =

|xn+1−xn|M (am tinut seama de faptul ca toate diferentele xk+1−xk au acelasi semn).Deoarece (xn)n≥1 este marginit, rezulta ca (yn)n≥1 este marginit, deci convergent.

XI.139. Daca p ∈ N, p ≥ 2, calculati limn→∞

ne

1n+1+

1n+2+...+ 1

pn − p.

Lucian Tutescu si Liviu Smarandache, Craiova

Solutie. Observam ca lim

1

n+ 1+

1

n+ 2+ . . .+

1

pn

=

limn→∞

1 +

1

2+ . . .+

1

pn− ln pn

−1 +

1

2+ . . .+

1

n− lnn

+ ln pn− lnn

= c −

c+ ln p = ln p, asadar avem de-a face cu o nedeterminare de tipul ∞· 0. Notand xn =1

n+ 1+

1

n+ 2+ . . .+

1

pn− ln p, limita din enunt devine: L = lim

n→∞n(exn+ln p−eln p) =

limn→∞

n · p(exn − 1) = limn→∞

np · exn − 1

xn· xn = lim

n→∞p · xn

1n

= p · limn→∞

xn+1 − xn1

n+1 − 1n

= p ·

limn→∞

pPi=1

1np+i −

pp(n+1)

− 1n(n+1)

= −p· limn→∞

pPi=1

1

np+i −1

np+p

1

n(n+1)

= −p limn→∞

p−1Pi=1

(p− i)n(n+ 1)

(np+ i)(np+ p)=

−pp−1Pi=1

limn→∞

n(p− i)

np2 + pi

= −p

p−1Pi=1

p− i

p2= − p

p2· p(p− 1)

2=

1− p

2.

XI.140. Daca A,B ∈ M2(R), aratati ca 2(detA)2+det (AB+BA)+2(detB)2 ≥det (A2 −B2) + 4detAB.

Mihaly Bencze, Brasov

Solutie. Daca U, V ∈ M2(C), are loc egalitatea

(∗) det(U + V ) + det(U − V ) = 2detU + 2detV.

Considerand ın (*) U = A2+B2 si V = i(AB−BA), unde i este unitatea imaginara,obtinem ca

2det(A2 +B2)− 2det(AB −BA) = det(A2 + iAB − iBA+B2)+

+ det(A2 − iAB + iBA+B2) = det(A− iB)det(A+ iB)+

+ det(A+ iB)det(A− iB) = 2det(A+ iB)det(A+ iB) =

= 2|det(A+ iB)|2 ≥ 0,

149

Page 66: format .pdf, 3.6 MB

prin urmare det(A2+B2) ≥ det(AB−BA). Insa, conform (*), avem ca det(A2+B2) =2detA2+2detB2−det (A2−B2), iar det(AB−BA) = 2detAB+2detBA−det(AB+BA) = 4detAB − det(AB +BA); ınlocuind, obtinem concluzia problemei.

Clasa a XII-a

XII.136. Calculati

Zx2013 + ax1006

(x1007 + b)2013dx, x ∈ (0,∞), a, b > 0.

Catalin Cristea, Craiova

Solutie.

Zx2013 + ax1006

(x1007 + b)2013dx =

Zx1006(x1007 + b)

(x1007 + b)2013dx +

Zx1006(a− b)

(x1007 + b)2013dx =

1

1007

Z(x1007 + b)′

(x1007 + b)2012dx+

a− b

1007

Z(x1007 + b)′

(x1007 + b)2013dx = − 1

2011 · 1007· 1

(x1007 + b)2011−

a− b

1007 · 2012· 1

(x1007 + b)2012+ C.

XII.137. Determinati primitivele functiei f :h0,

π

2

→ R, f(x) =

cos 2011x

cos2013 x.

Liviu Smarandache si Lucian Tutescu, Craiova

Solutie. Avem:

Zf(x)dx =

Zcos(2012x− x)

cos2013 xdx =

Zcos 2012x · cosx

cos2013 xdx +Z

sin 2012x · sinxcos2013 x

dx =

Zcos 2012x

cos2012 xdx+

Z 1

cos2012 x

′· sin 2012x

2012dx =Z

cos 2012x

cos2012 xdx+

sin 2012x

2012 · cos2012 x−Z

1

cos2012 x

sin 2012x

2012

′dx =

Zcos 2012x

cos2012 xdx+

sin 2012x

2012 · cos2012 x−Z

cos 2012x

cos2012 xdx =

sin 2012x

2012 · cos2012 x+ C.

XII.138. Aratati ca radacinile polinomului X3 + aX2 + bX + c sunt ın progresiegeometrica daca si numai daca a3c = b3.

Temistocle Bırsan, Iasi

Solutie. Tinand cont de relatiile lui Viete, avem: a3c = b3 ⇔ (x1 + x2 + x3)3 ·

x1x2x3 = (x1x2 + x2x3 + x3x1)3 ⇔ x1x2x3(x

31 + x3

2 + x33) = x3

1x32 + x3

2x33 + x3

1x33 ⇔

(x21−x2x3)(x

32x

23−x1x

33−x1x

32+x2

1x2x3) = 0 ⇔ (x21−x2x3)(x

22−x1x3)(x

23−x1x2) =

0 ⇔ x21 = x2x3 sau x2

2 = x1x3 sau x23 = x1x2, adica radacinile polinomului dat sunt

ın progresie geometrica.

XII.139. Pe multimea nevida G se considera operatia asociativa ,,·” ın raport cucare are loc regula de simplificare la stanga si astfel ıncat exista a ∈ G cu axa = x3,∀x ∈ G. Demonstrati ca (G, ·) este grup abelian.

D.M. Batinetu-Giurgiu, Bucuresti si Neculai Stanciu, Buzau

Solutie. Avem: a(ax)a = (ax)3 ⇒ a(axa) = a·x ·(axa) ·x ⇒ axa = x ·(axa)·x ⇒x3 = x·x3 ·x ⇒ x = x3, ∀x ∈ G. In particular, a3 = a si atunci a3x = ax, ∀x ∈ G, decia2x = x, ∀x ∈ G. Inlocuind pe x cu xa, deducem ca a2xa = xa ⇒ a · (axa) = xa ⇒ax3 = xa ⇒ ax = xa, ∀x ∈ G. Atunci a2x = a ·(ax) = a ·(xa) = (ax)a = (xa)a = xa2

si, cum a2x = x, ∀x ∈ G, rezulta ca a2x = xa2 = x, ∀x ∈ G, adica a2 este elementulneutru al operatiei ,,·”; notam e = a2.

150

Page 67: format .pdf, 3.6 MB

Am vazut ca x3 = x = xa2, ∀x ∈ G. Simplificand la stanga cu x, obtinem cax2 = e, ∀x ∈ G. Aceasta egalitate arata ca orice x ∈ G este inversabil, cu x−1 = x.In plus, xy = (xy)−1 = y−1x−1 = yx, ∀x, y ∈ G, asadar (G, ·) este grup abelian.

XII.140. Fie a > 0 si cercurile de ecuatii C1 : (x − a)2 + y2 = a2 siC2 : (x+ a)2 + y2 = a2. Determinati aria minima a unei elipse care are drept axe desimetrie axele de coordonate si este tangenta exterior ın cate doua puncte la fiecaredintre cercurile date.

Adrian Corduneanu, Iasi

Solutie. Fie A(a, 0) centrul lui C1, E :x2

λ2+

y2

µ2= 1 ecuatia unei elipse ca ın enunt,

.

y

d

P

O A

x

P (x0, y0) punctul de tangenta situat ın primul cadran,iar d tangenta comuna ın P la cele doua curbe. Panta

tangentei la C1 ın P este md = − 1

mAP=

a− x0

y0<

0, iar panta tangentei la elipsa ın P se obtine prin

derivare, din2x

λ2+

2y · y′

µ2= 0, pentru x = x0, deci

este y′(x0) = −µ2

λ2· x0

y0. Egaland cele doua pante,

obtinem ca µ2 = λ2

1− a

x0

. Cum P ∈ E , avem ca

x20

λ2+

y20µ2

= 1, iar din P ∈ C1 deducem ca (x0−a)2+y20 = a2. Din aceste trei egalitati,

rezulta ca λ2 =ax2

0

x0 − a, iar µ2 = ax0. Aria elipsei va fi A = πλµ =

πax3/20

(x0 − a)1/2.

Minimul ariei se atinge odata cu patratul sau, deci avem de determinat minimul

functiei f(x) =x3

x− a, cu a < x < 2a. Gasim imediat ca punctul de minim este

x0 =3a

2, iar valoarea minima a ariei este Amin =

3√3

2· πa2.

Solutiile problemelor pentru pregatireaconcursurilor propuse ın nr. 1/2013

A. Nivel gimnazialG236. Determinati numerele naturale a, b, c, d si e, strict mai mari ca 1, cu

proprietatea ca a+ b+ c+ d+ e = abcde− 95.Titu Zvonaru, Comanesti

Solutie. Presupunem ca a ≥ b ≥ c ≥ d ≥ e ≥ 2; atunci 95 + 5a ≥ 95 + a + b +c+ d+ e = abcde ≥ a · 2 · 2 · 2 · 2 = 16a, de unde a ≤ 8.

Daca a = 8, rezulta ca 103 + b + c + d + e = 8bcde si, procedand ca mai ınainte,obtinem ca 103 + 4b ≥ 64b, deci 60b ≤ 103 si nu avem solutii ın acest caz. La fel searata ca nu avem solutii cand a = 7. Daca a = 6, deducem ca 101+b+c+d+e = 6bcde,deci 101 + 4b ≥ 48b, prin urmare b = 2. Rezulta ca c = d = e = 2 si, ınlocuind ın

151

Page 68: format .pdf, 3.6 MB

ecuatia initiala, obtinem o contradictie. Analog, nu vor conveni nici valorile a = 5 sia = 4.

Deoarece 95 + 5 · 2 = 25, ramane doar posibilitatea a = 3. Atunci, unul, doua,trei, patru sau cinci dintre numerele cautate sunt egale cu 3, iar celelalte sunt 2. Prinverificari directe, solutiile sunt (3, 3, 3, 2, 2) si permutarile acesteia.

G237. Aratati ca exista n numere naturale distincte a1, a2, . . . , an pentru caresuma a1 + a1 + . . .+ an este patrat perfect, iar a21 + a22 + . . .+ a2n este cub perfect.

Gheorghe Iurea, IasiSolutie. Cautam numere de forma ai = k · i, i = 1, 2, . . . , n, unde k ∈ N∗; atunci

S = a1+a2+ . . .+an = k · n(n+ 1)

2, iar T = a21+a22+ . . .+a2n = k2

n(n+ 1)(2n+ 1)

6.

Pentru ca S sa fie patrat perfect, vom considera k =n(n+ 1)

2· u2, u ∈ N∗. In acest

fel, T devine egal cun3(n+ 1)3

8·u3 ·u · 2n+ 1

3si, alegand u = 3(2n+1)2, vom avea T

cub perfect. In concluzie, putem considera ai = k · i, unde k =9

2· n(n+ 1)(2n+ 1)4.

G238. Se considera numerele reale x, a1, a2, . . . , a100. Daca 51 dintre numerele

a1,a1 + a2

2, . . . ,

a1 + a2 + . . .+ a100100

sunt egale cu x, aratati ca macar doua dintre

numerele ai, i = 1, 100, sunt egale cu x.Catalin Budeanu, Iasi

Solutie. Fie b1 = a1 − x, b2 = a2 − x, . . . , b100 = a100 − x; atuncib1 + b2

2=

a1 + a22

− x, . . . ,b1 + b2 + . . .+ b100

100=

a1 + a2 + . . .+ a100100

− x. Din ipoteza rezulta

ca 51 dintre numerele b1,b1 + b2

2, . . . ,

b1 + b2 + . . .+ b100100

sunt nule, prin urmare 51

dintre sumele s1 = b1, s2 = b1 + b2, . . . , s100 = b1 + b2 + . . . + b100, ın numar de100, sunt nule. Macar doua perechi dintre cele 51 de sume nule au indici consecutivi:sm = sm+1 = 0 si sn = sn+1 = 0, cu m,n ∈ 1, 2, . . . , 99, m = n. Atunci bm+1 =sm+1 − sm = 0 si bn+1 = sn+1 − sn = 0, prin urmare am+1 = an+1 = x.

G239. Determinati valorile numarului real k, daca

a31 + . . .+ a32013 + 4026 ≥ k(a1 + . . .+ a2013),∀ai ∈ [−2,∞).

Lucian Tutescu, Craiova si Marian Voinea, BucurestiSolutie. Considerand a1 = . . . = a2013 = 1, obtinem k ≤ 3. Pentru a1 = . . . =

a2013 = −2, gasim k ≥ 3 si, de aici, k = 3. Demonstram ın continuare ca valoareak = 3 este convenabila.Cum x3−3x+2 = (x+2)(x−1)2 ≥ 0, ∀x ∈ [−2,+∞), rezultaca x3 + 2 ≥ 3x, ∀x ∈ [−2,∞). Dand lui x valorile a1, a2, . . . , a2013 si sumand relatiileobtinute, deducem ca a31 + . . .+ a32013 + 4026 ≥ 3(a1 + . . .+ a2013), ∀ai ∈ [−2,∞).

G240. Se considera E(x1, x2, . . . , xn) =nP

i=1

xi+2P

1≤i<j≤n

xixj, unde x1, x2, . . . , xn

∈ R, n ≥ 16, cunP

i=1

x2i = 1. Determinati valorile extreme ale acestei expresii.

Petru Asaftei, Iasi

152

Page 69: format .pdf, 3.6 MB

Solutie. Observam ca E =nP

i=1

xi +

nPi=1

xi

2

−nP

i=1

x2i =

nPi=1

xi

2

+nP

i=1

xi +

1

4− 5

4=

nPi=1

xi +1

2

2

− 5

4≥ −5

4. Pentru x1 = x2 = x4 = x6 = . . . = x16 = −1

4,

x3 = x5 = . . . = x15 =1

4, xk = 0 pentru k > 16, obtinem pentru E valoarea −5

4, prin

urmare Emin = −5

4.

Utilizand inegalitatea (n − 1)

nPi=1

x2i

≥ 2

Pi<j

xixj , deducem ca

nPi=1

xi

2

=

1 + 2Pi<j

xixj ≤ 1 + (n − 1) · 1 = n, decinP

i=1

xi ≤√n si atunci E ≤

√n + (n − 1).

Pentru x1 = x2 = . . . = xn =1√n

se atinge egalitatea, asadar Emax = n+√n− 1.

G241. In triunghiul ABC se considera cevienele concurente AA′, BB′ si CC ′

astfel ıncat A′B = A′C, iar dreptele BC si B′C ′ se intersecteaza ın M .

a) Demonstrati ca2

MA′ =

1

A′B− 1

A′C

.b)Determinati lungimea segmentului MA′ ın functie de laturile triunghiului, atunci

cand AA′ este bisectoare, respectiv ınaltime.Neculai Roman, Mircesti (Iasi)

Solutie. a) Presupunem, ca ın figura, ca A′B < A′C; atunci2

MA′ =

A

B

A

C

M B C

1

A′B− 1

A′C⇔ A′B · AC + MA′ · A′B = MA′ ·

A′C − A′B · A′C ⇔ A′B · MC = A′C · MB ⇔MB

MC=

A′B

A′C(∗). Aplicand teorema lui Menelaus

ın ABC cu transversala M − C ′ − B′, obtinem

caMB

MC· B

′C

B′A· C

′A

C ′B= 1. Insa, din teorema lui

Ceva,A′B

A′C· B

′C

B′A· C

′A

C ′B= 1 si, de aici, rezulta ca

MB

MC=

A′B

A′C, adica (*) este adevarata.

b) Daca AA′ este bisectoare, atunci BA′ =ac

b+ csi A′C =

ab

b+ c, deci

2

MA′ =b+ c

ac− b+ c

ab

= |b2 − c2|abc

, iar MA′ =abc

|b2 − c2|.

Daca AA′ este ınaltime, atunci A′B = c cosB = c · a2 + c2 − b2

2ac=

a2 + c2 − b2

2a,

iar A′C =a2 + b2 − c2

2a. Obtinem MA′ =

a · 1

a2 − b2 + c2− 1

a2 + b2 − c2

−1

.

Nota. In cazul ın care triunghiul este ascutiunghic si cele trei ceviene suntınaltimile sale, regasim problema 25480 din Gazeta Matematica 2/2006.

153

Page 70: format .pdf, 3.6 MB

G242. In triunghiul ABC latura AB este fixa, iar lungimea laturii AC este

constanta. Fie AA′ bisectoarea unghiului ÕBAC, cu A′ ∈ BC. Aratati ca dreaptaperpendiculara ın A′ pe AA′ trece printr-un punct fix.

Claudiu-Stefan Popa, IasiSolutie. Fie d perpendiculara ın A′ pe AA′, D = d ∩ AB, E = d ∩ AC,

A

N

Ed

CB

D

A

M

A′M∥AC, M ∈ AB si A′N∥AB, N ∈ AC. Vom arata caD este punctul fix cautat; cum [AB] este fix, ar fi sufi-cient sa aratam ca lungimea AD este constanta. Evidentca triunghiul ADE este isoscel, deci A′ este mijlocul luiDE. Atunci A′N este linie mijlocie ın ADE, prinurmare AD = 2A′N = 2AM (deoarece AMA′N esteromb). Aplicand de doua ori teorema lui Thales ın tri-

unghiul ABC, obtinem ca AM =AB ·AC

AB +AC. In con-

cluzie, AD =2AB ·ACAB +AC

= constant, deci d trece prin punctul fix D.

G243. Fie O intersectia diagonalelor trapezului ABCD, cu baza mare CD.Punctele M si N sunt astfel ıncat AD separa M si O, BC separa N si O, iarMAD ∼ NCB ∼ OAB. Aratati ca BD ·AC > AB ·MN.

Cosmin Manea si Dragos Petrica, Pitesti

Solutie. Cum ÖAMD ≡ ÕAOB, rezulta ca patrulaterul AMDO este inscriptibil,

prin urmareÖAMO ≡ ÕADO. In plus, m(ÖMAO) = m(ÖMAD)+m(ÕDAO) = m(ÕOAB)+

m(ÕDAO) = m(ÕDAB), asadar MAO ∼ DAB. Rezulta caAO

AB=

MO

BD, deci

AO · BD = AB ·MO. Analog, din asemanarea NOC ∼ BDC obtinem ca BD ·CO = NO·DC. Adunand aceste relatii, deducem ca BD·AC = AB ·MO+DC ·NO >AB ·MO +AB ·NO = AB(MO +NO) ≥ AB ·MN.

Observatie. De fapt, puncteleM,O siN sunt coliniare: m(ÖMON) = m(ÖMOA)+

m(ÕAOB) + m(ÕBON), iar ÖMOA ≡ ÖMDA ≡ ÕABO, ÕBON ≡ ÕBCN ≡ ÕBAO, asadar

m(ÖMON) = m(ÕABO) +m(ÕAOB) +m(ÕBAO) = 180.G244. Fie V ABC un tetraedru, iar M,N,P mijloacele muchiilor V A, V B res-

pectiv V C. Demonstrati ca 2(AMBC +ANCA +APAB) < AV BC +AV CA +AV AB +3AABC .

Mihaly Bencze, BrasovSolutie. Completam tetraedrul V ABC la prisma triunghiulara V BCV1AC1 si

V

V

P

C

C

A

B

P1

1

1

fie P1 mijlocul muchiei V1C1. Vom demonstra ca2APAB < AV AB +AABC(1). Inmultind cu 2 ambiimembri ai inegalitatii, avem:

(1) ⇔ 2AABPP1 < AABV V1 +AABCC1 ⇔⇔ 2 · PP1 · d(A,PP1) < CC1 · d(A,CC1)+

+ V V1 · d(A, V V1) ⇔⇔ 2d(A,PP1) < d(A,CC1) + d(A, V V1) ⇔⇔ 2 ·AP2 < AC2 +AV2, (2)

154

Page 71: format .pdf, 3.6 MB

unde P2, C2, V2 sunt intersectiile dintre planul α si PP1, CC1 respectiv V V1, iar α esteplanul ce contine punctul A si este perpendicular pe planul (V CC1). Se observa usorca AP2 este mediana ın triunghiul AC2V2 si atunci are loc inegalitatea (2), deci si (1).Scriem cele doua inegalitati analoage lui (1) si, prin adunarea lor, rezulta concluzia.

G245. Un triunghi echilateral are varfurile ın interiorul sau pe laturile unuihexagon regulat de latura 1 si nu contine centrul hexagonului ın interiorul sau. Careeste lungimea maxima posibila a laturii triunghiului?

Marian Tetiva, BarladSolutie. Raspunsul este 2/

√3. Daca hexagonul este ABCDEF, triunghiul AMN

pentru care latura MN are ca dreapta suport mediatoarea comuna a segmentelor BCsi EF realizeaza acest maxim (ınaltimea sa are lungimea 1).

Pentru a vedea ca, ıntr-adevar, aceasta este maximul cautat, sa consideram untriunghi echilateral XY Z ale carui varfuri sunt situate ın interiorul sau pe laturilehexagonului regulat ABCDEF si astfel ıncat centrul O al hexagonului nu apartine in-teriorului triunghiului XY Z. Interiorul triunghiului este intersectia benzilor deschisedelimitate de catre o latura a triunghiului si paralela la acea latura prin varful opus,de aceea centrul hexagonului nu apartine macar uneia dintre cele trei benzi.

Sa admitem ca O nu apartine benzii delimitate de latura Y Z a triunghiului siparalela prin X la Y Z si sa consideram paralela la Y Z prin O. Aceasta paralelaintersecteaza doua laturi opuse ale hexagonului si, tot fara a restrange generalitatea,putem considera ca aceasta paralela intersecteaza pe AB ın P si pe DE ın Q. Deasemenea, datoria simetriei, putem presupune ca XY Z este inclus ın pentagonulPBCDQ. (La limita, cand paralela prin O la Y Z trece prin varfuri ale hexagonului,pentagonul devine patrulater.) E clar ca ınaltimea din X a triunghiului XY Z estecel mult egala cu cea mai mare dintre distantele de la punctele B,C,D ls dreaptaPQ, iar la randul ei, aceasta distanta (oricare ar fi ea) este cel mult egala cu 1. Deexemplu, daca ne referim la distanta de la C la PQ, aceasta e cel mult egala cuCO = 1 (perpendiculara e mai scurta decat oblica). In concluzie, lungimea ınaltimiioricarui triunghi echilateral care ındeplineste conditiile din enunt este cel mult egalacu 1, ceea ce ınseamna ca lungimea laturii unui astfel de triunghi este cel mult egala

cu2√3si cu asta solutia este completa.

B. Nivel liceal

M

A

I

N

B

JC

O

W

L236. Pe sfera de centru Ω si raza 7 se considerapunctele A,B,C astfel ıncat BC = 4, CA = 5 siAB = 6. Perpendiculara pe planul triunghiului ABC ıncentrul cercului ınscris ın acest triunghi intersecteazasfera ın punctele M si N . Determinati lungimea seg-mentului MN .

Temistocle Bırsan, IasiSolutie. Notam cu O si I centrele cercurilor cir-

cumscris, respectiv ınscris ale triunghiului ABC. Evi-dent, O este proiectia lui Ω pe planul (ABC). In

155

Page 72: format .pdf, 3.6 MB

ABC cunoastem lungimile laturilor, prin urmare p =a+ b+ c

2=

15

2, S =È

p(p− a)(p− b)(p− c) =15√7

4, r =

S

p=

√7

2, R =

abc

4S=

8√7, iar OI2 = R2 −

2Rr =8

7. Daca J este mijlocul segmentului MN , atunci OIJΩ este dreptunghi, deci

ΩJ =8

7. Din triunghiul dreptunghic ΩJM obtinem ca JM2 = MΩ2−ΩJ2 =

É335

7,

asadar MN = 2JM = 2

É355

7.

L237. Fie A1, B1, C1 mijloacele laturilor BC,CA respectiv AB ale triunghiuluiascutitunghic ABC. Coarda comuna a cercurilor de diametre BB1 si CC1 inter-secteaza B1C1 ın A2; construim analog punctele B2 si C2. Demonstrati ca drepteleA1A2, B1B2 si C1C2 sunt concurente.

Neculai Roman, Mircesti (Iasi)Solutie. Fie E al doilea punct de intersectie dintre cercul de diametru BB1

A

B C

E D

A1

B1C

1 A2

si dreapta B1C1, iar D al doilea punct de intersectiedintre cercul de diametru CC1 si dreapta B1C1. Din

B1C1∥BC si m(ÖB1EB) = m(ÖC1DC) = 90, rezultaca BCDE este drepuntghi. Folosind triunghiurile

dreptunghice BEC1 si CDB1, obtinem cactgB

ctgC=

EC1

BE:

DB1

DC=

EC1

DB1. Punctul A2 fiind pe axa

radicala a cercurilor de diametre BB1 si CC1, avemA2E ·A2B1 = A2C1 ·A2D ⇔ A2C1 ·A2B1+EC1 ·A2B1 = A2C1 ·A2B1+A2C1 ·DB1 ⇔EC1 · A2B1 = A2C1 · DB1 ⇔ EC1

DB1=

A2C1

A2B1. Astfel,

ctgB

ctgC=

A2C1

A2B1si, analog,

ctgC

ctgA=

B2A1

B2C1, iar

ctgA

ctgB=

C2B1

C2A1. De aici,

A2C1

A2B1· C2B1

C2A1· B2A1

B2C1= 1 si reciproca

teoremei lui Ceva conduce la concluzia problemei.

Nota. D-l. Titu Zvonaru observa ca punctul de concurenta a dreptelor A1A2,B1B2 si C1C2 este, de fapt, izotomicul (ın A1B1C1) ortocentrului triunghiuluiA1B1C1.

A

B

C

PN

R

M

a

b

g

L238. Fie R un punct ın interiorul triunghiuluiABC, iar M = AR ∩ BC, N = BR ∩ AC, P =CR ∩ AB. Notam cu α, β si γ masurile unghiurilorÖAMB, ÕBNC, respectiv ÕCPA. Daca α−β+ γ−C +A,−α+β+γ−B+C si α+β−γ−A+B se afla ın inter-

valulh0,

π

2

i, aratati ca R este centrul cercului ınscris

ın triunghiul ABC.Marius Dragan, Bucuresti

Solutie. Din teorema lui Ceva (forma trigonome-

trica) obtinem:sinMAB

sinMAC· sinNBC

sinNBA· sinPCB

sinPCA= 1 ⇔ sin(α+B)

sin(α− C)· sin(β + C)

sin(β −A)·

156

Page 73: format .pdf, 3.6 MB

sin(γ +A)

sin(γ −B)= 1 ⇔ sin(α+B)·sin(β+C) sin(γ+A) = sin(α−C) sin(β−A) sin(γ−B) ⇔

cos(α+B − γ −A) sin(β + C)− cos(α+B + γ +A) · sin(β + C) = cos(α− C − γ −B) sin(β − A) − cos(α − C + γ − B) sin(β − A). Dupa transformarea produselor ınsume si efectuarea calculelor, gasim ca sinA cos(−α+β+γ−B+C)+sinB · cos(α−β+ γ+A−C)+ sinC · cos(α+β− γ−A+B) = 0 de unde, tinand seama de ipoteza

problemei, rezulta ca α−β+γ+A−C = −α+β+γ−B+C = α+β−γ+B−A =π

2.

Solutia acestui sistem este α =1

2A + C, β =

1

2A + C, γ =

1

2C + B, prin urmare R

este intersectia bisectoarelor triunghiului ABC.

L239. Fie ABCD patrulater circumscriptibil cu AB∥CD, iar A′, B′, C ′ si D′

sunt punctele de tangenta ale cercului ınscris cu laturile AB,BC,CD respectiv DA.Se noteaza cu A′′, B′′, C ′′, D′′ simetricele punctelor A′, B′, C ′ respectiv D′ fata demijloacele laturilor pe care se afla. Demonstrati ca:

a) SA′B′C′D′ ≤ SA′′B′′C′′D′′ ;b) SA′B′C′D′ + SA′′B′′C′′D′′ ≤ SABCD.

Marius Olteanu, Rm. ValceaSolutie. Notam AA′ = AD′ = A′′B = DD′′ = u, BB′ = A′B = A′′A = B′′C =

v, CC ′ = B′C = C ′′D = BB′′ = w si DC ′ = DD′ = D′′A = CC ′′ = t; fie ıncaO centrul cercului si r raza acestuia. Aplicand teorema ınaltimii ın triunghiuriledreptunghice AOD si BOC, obtinem ca ut = vw = r2. Apoi, avem

SDC′′D′′ =uw sinD

2=

uw

2 tg D2

1 + tg2 D2

= uw ·rt

1 + r2

t2

=uwtr

r2 + t2

si ınca trei relatii similare. Astfel,

SA′′B′′C′′D′′ = SABCD − r

uwt

r2 + t2+

vwt

r2 + w2+

uvw

r2 + v2+

uvt

r2 + u2

.

Mai observam ca SDC′D′ =t2 sinD

2= r · t3

r2 + t2, prin urmare

SA′B′C′D′ = SABCD − r

t3

r2 + t2+

w3

r2 + w2+

v3

r2 + v2+

u3

r2 + u2

.

a) Tinand seama de cele de mai sus, avem:

SA′B′C′D′ ≤ SA′′B′′C′′D′′ ⇔X t3

r2 + t2≥X uwt

r2 + t2⇔X t3

ut+ t2≥X uwt

ut+ t2⇔

⇔ t2 + u2

t+ u+

v2 + w2

v + w≥ uw + vt

u+ t+

uw + vt

v + w⇔

⇔ (t2 + u2)(v + w) + (t+ u)(v2 + w2) ≥ (uw + vt)(u+ v + w + t) ⇔

⇔ (tw − uv)(w + t− u− v) ≥ 0 ⇔tr2

v− r2

tv

r2

v+ t− r2

t− v

≥ 0 ⇔

⇔ (t2 − v2)(r2t− r2v + t2v − v2t) ≥ 0 ⇔ (t− v)2(t+ v)(r2 + tv) ≥ 0,

157

Page 74: format .pdf, 3.6 MB

relatie adevarata. Se constata ca egalitatea se atinge ın cazul ın care ABCD esteromb.

b) Cum SABCD = r(u+ v + t+ w), inegalitatea de demonstrat revine laX uwt

r2 + t2+X t3

r2 + t2≥ u+ v + t+ w ⇔

⇔ t2 + u2

t+ u+

v2 + w2

v + w+

uw + vt

t+ u+

uw + vt

v + w≥ u+ v + t+ w ⇔

⇔ (t2 + u2)(v + w) + (t+ u)(v2 + w2) ≥ (uw + vt)(u+ v + t+ w) ⇔

⇔ (tv − uw)(t+ v − u− w) ≥ 0 ⇔tr2

w− r2

tw

t+

r2

w− r2

t− w

≥ 0 ⇔

⇔ (t− w)2(t+ w)(r2 + tw) ≥ 0,

inegalitate adevarata. Egalitatea se atinge ın cazul ın care ABCD este trapez isoscel.

Nota. S-a primit solutie corecta de la prof. Nela Ciceu, Bacau.

L240. Demonstrati ca ın orice triunghi are loc inegalitatea

2r

R

X bc

ab+ ac+X c

a+ b≤ 3.

Marian Tetiva, BarladSolutia 1 (a autorului). Inegalitatea rezulta ın mod evident din identitatea

(∗) 2r

R

X abc

(b+ c)(a2 − (b− c)2)+X c

a+ b= 3.

Pentru a vedea de unde se obtine aceasta, observam ıntai ca avem

cosB + cosC = 2 cosB + C

2cos

B − C

2= 2 sin

A

2cos

B − C

2=

= 2 sinA

2sin

B

2sin

C

2·cos B

2 cos C2 + sin B

2 sin C2

sin B2 sin C

2

=r

R

a+ b

a+ b− c,

unde, pentru utlima egalitate, am folosit formulele sinA

2=

r(p− b)(p− c)

bc, cos

A

2=r

p(p− a)

bcsi cele analoage. Pe de alta parte, cu teorema cosinusului, avem imediat

cosB + cosC =(b+ c)(a2 − (b− c)2)

2abc.

Folosind acestea, precum si formulele obtinute prin permutari circulare, deducem caX 1

cosB + cosC=

R

r

3−

X c

a+ b

= 2

X abc

(b+ c)(a2 − (b− c)2),

de unde rezulta identitatea (*) si, de aici, inegalitatea din enunt.

158

Page 75: format .pdf, 3.6 MB

Solutia 2 (Daniel Vacaru, Pitesti). Vom demonstra ca

(∗∗) 2r

R· bc

ab+ ac≤ 2(p− a)

b+ c;

odata demonstrata aceasta inegalitate, scriindu-le si pe celelalte doua similare sisumand, obtinem cerinta problemei.

Inegalitatea (**) se rescrie, succesiv, astfel:

(∗∗) ⇔ r

R· bca

≤ p− a ⇔ r

R· abca2

≤ p− a ⇔

⇔ S

p· 4Sa2

≤ p− a ⇔ 4p(p− a)(p− b)(p− c)

pa2≤ p− a ⇔

⇔ 4(p− b)(p− c) ≤ a2 ⇔ (a+ c− b)(a+ b− c) ≤ a2.

Aceasta ultima inegalitate rezulta imediat din inegalitatea mediilor MG ≤ MA si,astfel, solutia este completa.

Solutia 3 (Titu Zvonaru, Comanesti). Deoarece2r

R=

2S

p· 4Sabc

=

8p(p− a)(p− b)(p− c)

pabc=

8(p− a)(p− b)(p− c)

abc, avem:

2r

R

X ab

bc+ ca+X c

a+ b− 3 =

X8(p− a)(p− b)(p− c)

c2(a+ b)+

c

a+ b− 1

=

=X

8(p− a)(p− b)(p− c)

c2(a+ b)− 2(p− c)

a+ b

=X 2(p− c)[4(p− a)(p− b)− c2]

c2(a+ b)=X 2(p− c)[(c− a+ b)(c+ a− b)− c2]

c2(a+ b)= −

X 2(p− c)(a− b)2

c2(a+ b)≤ 0.

Avem egalitate daca si numai daca a = b = c.

Nota. S-a primit solutie corecta de la elevul Ovidiu Pauca, Trusesti (Botosani).

L241. Aratati casin3 x

(1 + sin2 x)2+

cos3 x

(1 + cos2 x)2≤ 2

√2

9, ∀x ∈ R. (In legatura cu

problema L211 din RecMat-2/2011.)Dumitru Barac, Sibiu

Solutie. Notand sinx = a, cosx = b, avem de aratat caa3

(1 + a2)2+

b3

(1 + b2)2≤

2√2

9, ∀a, b ∈ R cu a2 + b2 = 1. Observam ca pentru a = b =

1√2

are loc egali-

tatea. Folosim metoda parabolei tangente: cautam A,B ∈ R astfel ıncatt3

(1 + t2)2≤

At2 + B, ∀t ∈ R (de fapt, este suficient pentru t ∈ [−1, 1]). Impunem conditiile

t3

(1 + t2)2

t= 1√

2

= (At2 + B)

t= 1√

2

si

t3

(1 + t2)2

′ t= 1√

2

= (At2 + B)′t= 1√

2

; obtinem

159

Page 76: format .pdf, 3.6 MB

√2

9=

A

2+B, respectiv A =

5√2

27, deci B =

√2

54. Demonstram acum ca

(∗) t3

(1 + t2)2≤ 5

√2

27t2 +

√2

54, ∀t ∈ R.

Avem succesiv:

(∗) ⇔ 27t3√2 ≤ (10t2 + 1)(t4 + 2t2 + 1) ⇔ 10t6 + 21t4 − 27t3

√2 + 12t2 + 1 ≥ 0

⇔ (t√2− 1)2[(5t2 + 5t

√2 + 16)t2 + (2t2 + 2t

√2 + 1)] ≥ 0,

iar aceasta inegalitate este adevarata ıntrucat trinoamele 5t2 + 5t√2 + 16 si 2t2 +

2t√2 + 1 au discriminantii negativi. Acum, luand ın (*) t = a si t = b si adunand

membru cu membru inegalitatile obtinute, rezulta cerinta problemei.

Notam ca marginea2√2

9este efectiv atinsa si este mai buna decat estimarea

3√3

16din problema L211 citata.

L242. Un paralelipiped dreptunghic are dimensiunile x, y, z si diagonala d. Aratatica

d4

ad4 + bx4+

d4

ad4 + by4+

d4

ad4 + bz4≤ 27

9a+ b,

oricare ar fi a, b > 0, 6a ≥ 5b. (In legatura cu L231 din RecMat-2/2012.)Titu Zvonaru, Comanesti

Solutie. Avem:d4

ad4 + bx4− 9

9a+ b=

b(d2 + 3x2)(d2 − 3x2)

(ad4 + bx4)(9a+ b)=

b

9a+ b

(4x2 + y2 + z2)(y2 − x2)

ad4 + bx4+

(4x2 + y2 + z2)(z2 − x2)

ad4 + bx4

, prin urmare

X d4

ad4 + bx4− 27

9a+ b=

b

9a+ b

X(4x2 + y2 + z2)(y2 − x2)

ad4 + bx4+

(4x2 + y2 + z2)(z2 − x2)

ad4 + bx4

=

b

9a+ b

X(4x2 + y2 + z2)(y2 − x2)

ad4 + bx4+

(x2 + 4y2 + z2)(x2 − y2)

ad4 + by4

=

=b

9a+ b

X(x2 − y2)

x2 + 4y2 + z2

ad4 + by4− 4x2 + y2 + z2

ad4 + bx4

=

=b

9a+ b

X (x2 − y2)2[(b− 3a)(x4 + y4)− 3az4 + (5b− 6a)x2y2 + (b− 6a)(x2z2 + y2z2)

(ad4 + by4)(ad4 + bx4),

iar acest numar este cel mult egal cu 0 ın conditiile din enunt (deoarece 5b ≤ 6aimplica b < 3a si b < 6a). Avem egalitate daca si numai daca x = y = z.

L243. Pentru m,n ∈ N∗ si a, b, c ∈ R∗+, demonstrati inegalitatea

ambmcm(an + bn + cn)2

a3m+2n + b3m+2n + c3m+2n≤ 3.

(In legatura cu problema VIII.149 din RecMat-1/2012.)Neculai Stanciu, Buzau

160

Page 77: format .pdf, 3.6 MB

Solutia 1 (Emanuel Necula, elev, Campulung Muscel, Titu Zvonaru, Co-manesti si Daniel Vacaru, Pitesti). Putem presupune ca a ≥ b ≥ c. Folosindinegalitatea lui Cebısev, inegalitatea mediilor MA ≥ MG si binecunoscuta 3(x2 +y2 + z2) ≥ (x+ y + z)2, obtinem:

3(a3m+2n + b3m+2n + c3m+2n) = 3(a3m · a2n + b3m · b2n + c3m · c2n) ≥≥ (a3m + b3m + c3m)(a2n + b2n + c2n) ≥ 3ambmcm(a2n + b2n + c2n) ≥≥ ambmcm(an + bn + cn)2.

Avem egalitate daca si numai daca a = b = c.

Solutia 2 (a autorului). Folosind inegalitatea

√a+

√b+

√c

3

2

≤ a+ b+ c

3,

avem:

ambmcm(an + bn + cn)2 = [am2 b

m2 c

m2 (an + bn + cn)]2 =

= [(am+2nbmcm)12 + (ambm+2ncm)

12 + (ambmcm+2n)

12 ]2 ≤

≤ 3(am+2nbmcm + ambm+2ncm + ambmcm+2n).

Din inegalitatea mediilor ponderate obtinem ca

am+2nbmcm ≤ m+ 2n

3m+ 2na3m+2n +

m

3m+ 2nb3m+2n +

m

3m+ 2nc3m+2n

si ınca doua inegalitati similare. Adunandu-le membru cu membru, rezulta cerintaproblemei.

L244. Fie a, b, c ∈ R∗+, a ≥ b ≥ c. Demonstrati ca are loc inegalitatea

(a2 + c2)(ab+ ac+ bc)− 2ac(a2 + b2 + c2) ≥ 2c(a− b)(a− c)(b− c).

Gabriel Dospinescu, Paris si Marian Tetiva, BarladSolutia 1 (a autorului). Totul rezulta din identitatea

(a2 + c2)(ab+ ac+ bc)− 2ac(a2 + b2 + c2)− 2c(a− b)(a− c)(b− c) =

= c2(a− c)2 + 2c2(a− b)(b− c) + 2c(a− c)2(b− c) + (a− c)3(b− c).

Propunem cititorului sa verifice aceasta identitate, folosind eventual metoda normarii(a se vedea, de exemplu, Inegalitati -idei si metode de Mihai Onucu Drimbe, Edi-tura GIL, Zalau, 2003, pp. 79-80).

Inegalitatea din enunt ıntareste inegalitatea (G. Dospinescu):

a2 + c2

2ac≥ a2 + b2 + c2

ab+ ac+ bc, a ≥ b ≥ c > 0.

Solutia 2 (Daniel Vacaru, Pitesti). Pornim din membrul stang:

MS = (a2 + c2)(ab+ bc)− ac(a2 + c2)− 2ac · b2 ≥≥ (a2 + c2)(ab+ bc− ca)− (a2 + c2) · b2 =

= (a2 + c2)(ab+ bc− ca− b2) = (a2 + c2)(a− b)(b− c).

161

Page 78: format .pdf, 3.6 MB

Ramane sa probam ca a2 + c2 ≥ 2c(a− c); acest lucru revine la a2 + 3c2 ≥ 2ac, ceeace este evident, ıntrucat a2 + 3c2 ≥ a2 + c2 ≥ 2ac.

Nota. Am mai primit solutie corecta de la d-l. Titu Zvonaru.

L245. Fie f, g : [0, 1] → [0, 1] doua functii continue astfel ıncat

f kn− g

k

n

≤ 1

kf

g

k

n

, ∀k, n ∈ N∗, k ≤ n. Demonstrati ca cele doua functii sunt egale.

Florin Stanescu, GaestiSolutie (Moubinool Omarjee, Paris). Obtinem imediat ca

1

n

nXk=1

k

n

f kn− g

k

n

≤ 1

n

1

n

Xf

g

k

n

.

Functia f g fiind continua, deci integrabila, limita sirului1

n

nPk=1

f

g

k

n

exista si

este finita; atunci limn→∞

1

n

1

n

nPk=1

f

g

k

n

= 0. Deducem ca

limn→∞

1

n

nXk=1

k

n

f kn− g

k

n

= Z 1

0x|f(x)− g(x)|dx ≤ 0,

de unde

Z 1

0x|f(x) − g(x)|dx = 0. Insa functia x 7→ x|f(x) − g(x)| este continua si

nenegativa; avand integrala nula, rezulta ca functia este nula. De aici, obtinem caf(x) = g(x), ∀x ∈ (0, 1]. Faptul ca f(0) = g(0) urmeaza din continuitatea functiilorf si g ın 0.

Nota. Autorul problemei arata mai ıntai ca

Z 1

0P (x)f(x)dx =

Z 1

0P (x)g(x)dx,

oricare ar fi polinomul cu coeficientii reali P si foloseste apoi faptul ca orice functiecontinua pe un interval este limita uniforma a unui sir de functii polinomiale.

Am primit solutie corecta si de la d-l Daniel Vacaru.

Primul numar al Colectiei ,,Recreatii Matematice”

1. D. Branzei, Al. Negrescu – Probleme de pivotare,

Ed. ,,Recreatii Matematice”, Iasi, 2011 (208 pag.)

poate fi procurat printr-o simpla cerere la adresa: t [email protected] si indicareaadresei postale proprii. Cartea va fi trimisa cu plata ramburs la adresa indicata contrasumei de 25 lei (inclusiv taxe postale).

162

Page 79: format .pdf, 3.6 MB

Probleme propuse1

Clasele primare

P.269. Sapte forme geometrice sunt asezate astfel: Mutati trei forme astfel ıncat ın sirul obtinut triunghiul sa fie la mijloc, iar patratelesa fie de o parte si de alta a lui.(Clasa I ) Mariana Manoli, eleva, Iasi

P.270. Afati numerele naturale a si b astfel ıncat a− 26 = 4− b.(Clasa I ) Denisa Apetrei, eleva, Iasi

P.271. Din cei 28 elevi ai unei clase, 25 ındragesc fotbalul si 24 tenisul. Fiecaredintre elevi ındrageste cel putin un sport. Cati elevi ındragesc un singur sport?(Clasa I ) Mihaela Buleandra, eleva, Iasi

P.272. In trei cutii sunt 80 de nasturi. Cati nasturi sunt ın fiecare cutie, daca ınprimele doua cutii sunt 64 nasturi, iar ın ultimele doua 41?(Clasa a II-a) Maria Racu, Iasi

P.273. Exista 12 numere mai mari ca zero, pare, diferite ıntre ele si care sa aibasuma 154?(Clasa a II-a) Dumitrita Grigoriu, eleva, Iasi

P.274. Intr-o urna sunt 34 de jetoane, pe fiecare fiind scris un numar; existajetoane cu numere de paritati diferite. Dupa ce s-au scos mai mult de 14 jetoane cunumere pare, ın urna au ramas tot atatea jetoane cu numere pare. Cate jetoane cunumere impare sunt ın urna?(Clasa a II-a) Iustina Diaconu, eleva, Iasi

P.275. Se da sirul de numere 0, 3, 6, 9, 12, . . . Cate numere din sir, mai mici decat500, au suma cifrelor 18?(Clasa a III-a) Nicoleta Cumpata, eleva, Iasi

P.276. Sa se afle a si b stiind ca au loc egalitatile: a : 2− 15 = b : 3, a+ b = 90.(Clasa a III-a) Alexandra Tololoi, eleva, Iasi

A B C D E

FGHIJ

K L M N

OPP.277. In desenul alaturat, taiati un numar minimde segmente, de aceeasi lungime cu segmentul AB, astfelınat sa nu mai ramana nici un patrat.(Clasa a III-a) Maria Nastasia, eleva, Iasi

P.278. Suma a patru numere naturale diferite este S.Daca adaugam 2 la primul numar, scadem 2 din al doileanumar, ınmultim cu 2 al treilea numar si ımpartim la 2 alpatrulea numar, vom obtine acelasi rezultat. Aratati ca suma celor patru numere seımparte exact la 9.(Clasa a III-a) Andreea Bızdıga, eleva, Iasi

P.279. Suma a trei numere naturale nenule este zecimea produsului lor, iar sumaultimelor doua este zecimea primului numar. Aflati suma celor 3 numere.(Clasa a IV-a) Amalia Munteanu, eleva, Iasi

1Se primesc solutii pana la data de 15 ianuarie 2014.

163

Page 80: format .pdf, 3.6 MB

P.280. Fie abcde un numar cu suma cifrelor patru. Aratati ca, daca ımpartimacest numar la trei, obtinem restul 1.(Clasa a IV-a) Mihaela Gılca, eleva, Iasi

P.281. Fie un poligon cu 10 laturi astfel ıncat oricare trei varfuri nu se aflape aceeasi dreapta. Cate segmente se formeaza prin unirea oricaror doua varfurinealaturate?(Clasa a IV-a) Andreea Simion, eleva, Iasi

P.282. In 10 cosuri sunt mere, pere si gutui, ın total 44 de fructe. In fiecare cossunt fructe de toate felurile. Sa se arate ca exista doua cosuri care au acelasi numarde mere, doua cosuri care au acelasi numar de pere si doua cosuri care au acelasinumar de gutui.(Clasa a IV-a) Petru Asaftei, Iasi

Clasa a V-a

V.165. Aratati ca fractia3n+2 · 5n + 3n · 5n+2 − 15n

abcabceste reductibila.

Nicolae Ivaschescu, Craiova

V.166. Numere 1, 2, 2, 3, 3, 3, 4, 4, 4, 4, 5, 5, 5, 5, 5, . . . se scriu succesiv ın cate unadin 2013 casute dispuse circular. In momentul ın care ajungem la o casuta ocupata,stergem numarul deja existent si scriem noul numar. Aflati suma numerelor scrise ıncasute imediat dupa ce s-a scris primul 2013.

Silviu Boga, Iasi

14 9

16 25 4964 81 100 121..............................

V.167. In tabloul alaturat sunt scrise, ın ordine crescatoare,toate patratele perfecte care nu se divid cu 6. Daca pe randul58 al 25-lea numar este x2, determinati valoarea lui x.

Vlad Tuchilus, elev, Iasi

V.168. a) Aratati ca numarul 22013 nu poate fi scris ca suma de cel putin douanumere naturale consecutive.

b) Scrieti numarul 3·22013 ca suma de cel putin doua numere naturale consecutive.Cate asemenea modalitati de scriere exista?

Elena Iurea, Iasi

V.169. Demonstrati ca numarul N = 222013

+ 27 se divide cu 31.Tamara Culac, Iasi

V.170. Intr-o urna sunt 68 de bile. Doua persoane joaca urmatorul joc: fiecare,alternativ, scoate din urna ıntre una si cinci bile, pana cand urna se goleste. Castigacel care a scos ultimul trei bile deodata. Care dintre cei doi jucatori are strategie decastig?

Mihai Craciun, Pascani

V.171. Consideram numarul n = 123 . . .9, unde ın fiecare patratel se aflaunul dintre semnele ,,·” sau ,,:”. Stiind ca n este numar natural, aratati ca este parsi cel putin egal cu 70.

Radu Miron, elev, Iasi

164

Page 81: format .pdf, 3.6 MB

Clasa a VI-a

VI.165. Se considera un triunghi isoscel ABC, AB = AC, cu proprietatea ca ex-

ista punctul D pe latura BC astfel ıncat BD = 2DC si m(ÕBAD) = 90. Determinatimasurile unghiurilor triunghiului.

Elena Iurea, Iasi

VI.166. Se considera triunghiul isoscel ABC, AB = AC, si fie D si E picioarelebisectoarelor din B, respectiv C. Punctul F este astfel ıncat DF∥CE, DF = BD,iar E si F sunt separate de AC. Demonstrati ca punctele B,C si F sunt coliniare.

Ion Patrascu, Craiova

VI.167. Aratati ca nu exista numere prime de trei cifre, avand produsul acestorcifre egal cu 210.

Mirela Marin, Iasi

VI.168. Demonstrati ca numarul A =1

1007+

1

1008+ . . .+

1

2013nu este natural.

Doina Stoica si Mircea Stoica, Arad

VI.169. Fie a ∈ N∗ si numarul n = a2+a3+ . . .+a2013. Demonstrati ca numarula+ n

1 + a+ a2− n

1 + a+ a2 + a3este natural nenul.

Ionel Tudor, Calugareni

VI.170. Stabiliti care este cel mai mare numar de numere prime care pot fi gasiteprintre 15 numere naturale consecutive.

Constantin Dragomir, Pitesti

VI.171. Aratati ca exista o infinitate de numere naturale a cu proprietatea ca asi a+ 1 sunt, fiecare, suma a cate trei patrate perfecte nenule.

Nicolae Ivaschescu, Craiova

Clasa a VII-a

VII.165. Fie a, x, y numere reale strict pozitive cu proprietatea ca x(y − a) ≥y(a− x). Demonstrati ca a2 ≤ xy.

Gheorghe Iacob, Pascani

VII.166. Daca x, y, z sunt numere reale pozitive, aratati ca

x2 + 4x+ 4 + 8y

z+

y2 + 4y + 4 + 8z

x+

z2 + 4z + 4 + 8x

y≥ 48.

Bogdan Chiriac, Bacau

VII.167. Daca ecuatia |x− 1|+ |x− 2|+ . . .+ |x− 2013| = y(y+1) are o singurasolutie (x0, y0) ∈ R× R∗

+, calculati x0 + y0.Liviu Smarandache, Craiova

VII.168. Fie n ∈ N∗ fixat. Aratati ca exista p ∈ N astfel ıncat numarul n2 + k2

sa nu fie patrat perfect, oricare ar fi k ∈ N, k > p.Marian Pantiruc, Iasi

165

Page 82: format .pdf, 3.6 MB

VII.169. Fie D piciorul bisectoarei din A ın triunghiul ABC. Cercul de diametruAD intersecteaza a doua oara laturile AB si AC ın mijloacele lor. Ce particularitateare triunghiul ABC? Dar raza cercului?

Temistocle Bırsan, Iasi

VII.170. Se considera triunghiul ABC, dreptunghic ın A. Punctele D,E, P

si Q se afla pe segmentele AC,AB,CE, respectiv BD, astfel ıncat ÕABD ≡ ÕACE,DP ⊥ CE si EQ ⊥ BD.

a) Demonstrati ca punctele A,D,E, P si Q sunt conciclice.b) Aratati ca DEQP este trapez daca si numai daca AB = AC.

Petru Asaftei, Iasi

VII.171. Fie M un punct pe latura BC a triunghiului ABC, cu unghiul bAascutit. Perpendiculara ın M pe BC taie dreapta AC ın punctul N . Demonstrati caMN · sinA+MC · cosA ≤ CN. Cand se atinge egalitatea?

Catalin Calistru, Iasi

Clasa a VIII-a

VIII.165. Fie N si P centrele fetelor ABB′A′, respectiv ADD′A′ ale unui para-lelipiped dreptunghic ABCDA′B′C ′D′. Daca exista un punct M pe diagonala (AC ′),diferit de mijlocul acesteia, cu proprietatea ca MN ⊥ AB′ si MP ⊥ AD′, aratati caABCDA′B′C ′D′ este cub.

Stefan Dominte, elev, Iasi

VIII.166. Piramida patrulatera regulata V ABCD are muchia bazei AB = a siınaltimea V O = h. Determinati raza sferei ınscrisa ın piramida.

Adrian Corduneanu, Iasi

VIII.167. Determinati numerele ıntregi k pentru care numarul n = k4 + 8k3 −35k2 − 24k + 161 este natural prim.

Mihai Haivas, Iasi

VIII.168. Se considera sistemul de ecuatii¨ax+ by = α

cx2 + dy2 = β, a, b, c, d, α, β ∈ R∗.

Aratati ca sistemul are solutie unica daca si numai dacaa2

c+

b2

d=

α2

β. Aflati solutia

sistemului ın acest caz.Temistocle Bırsan, Iasi

VIII.169. Pentru a, b, c ∈ R∗+, demonstrati inegalitatea

a

b2 + c2+

b

c2 + a2+

c

a2 + b2≥ 3(a+ b+ c)

2(a2 + b2 + c2).

Marian Cucoanes, Marasesti

166

Page 83: format .pdf, 3.6 MB

VIII.170. Fie m ∈ N si numerele reale a si b cu a + b ≥ 0. Aratati ca a2m+1 +b2m+1 ≥ ambm(a+ b).

Ovidiu Pop, Satu Mare

VIII.171. Demonstrati ca nu exista numere naturale nenule a, b si c pentru carea(a2b2 + 1) = c2.

Cosmin Manea si Dragos Petrica, Pitesti

Clasa a IX-a

IX.141. Daca a, b, c, d sunt numere reale pozitive, aratati ca

a+ b2 + c4 + 4

d+

b+ c2 + d4 + 4

a+

c+ d2 + a4 + 4

b+

d+ a2 + b4 + 4

c≥ 28.

Catalin Cristea, Craiova

IX.142. Determinati functiile f : R → R cu proprietatea ca

f(x4 − y4) = (x− y)(x2 + y2)(f(x) + f(y)), ∀x, y ∈ R.

Lucian Tutescu, Craiova si Ion Nedelcu, Ploiesti

IX.143. Rezolvati ın R3 sistemul8><>:x+ y = −1

x2 + y2 + 2x2y2 sin z = 1

x3 + y3 − x6y6 sin3 z = 1.

Vasile Chiriac, Bacau

IX.144. Se considera triunghiul ADE, dreptunghic ın D, si triunghiul DAP ,dreptunghic ın A, astfel ıncat (AE) ∩ (DP ) = G si 2DE = AP.

a) Aratati ca exista o infinitate de triunghiuri ABC ın care AE este mediana, iarAD este ınaltime.

b) Demonstrati ca cercurile circumscrise tuturor triunghiurilor ABC trec prinpunctul P .

Romanta Ghita si Ioan Ghita Blaj

IX.145. Demonstrati ca ın orice triunghi ABC are loc inegalitateaha

ia≥É

2r

R,

unde ha si ia reprezinta lungimile ınaltimii, respectiv bisectoarei din A.Constantin Dragomir, Pitesti

Clasa a X-a

X.141. Rezolvati ecuatia 2013x3

+ log2013 x = 2013x2

.Lucian Tutescu, Craiova si Aurel Chirita, Slatina

X.142. Fie z1, z2, z3 trei numere complexe distincte, de modul 1, astfel ıncatz1z2z3 = (z1 + z2 + z3)(z1z2 + z1z3 + z2z3). Aratati ca cele trei numere sunt afixelevarfurilor unui triunghi dreptunghic.

Ion Nedelcu, Ploiesti si Dimitru Savulescu, Bucuresti

167

Page 84: format .pdf, 3.6 MB

X.143. Determinati numerele reale x ∈h−π

2,+∞

cu proprietatea ca 2013x+1 =

2012x + | sinx− cosx|.Sven Cortel, elev, Satu Mare

X.144. Fie H ortocentrul triunghiului ABC si MN∥BC, PQ∥AC, RS∥AB astfelıncat MN ∩PQ∩RS = H, iar punctele M,N,P,Q,R si S sunt situate pe laturiletriunghiului. Aratati ca:

a)MN

sinA+

PQ

sinB+

RS

sinC= 4R;

b) MN · PQ ·RS ≤ 8R3

3√3.

Bogdan Victor Grigoriu, Falticeni

X.145. Se considera triunghiul ABC cu m( bA) = 90 si m(ÒB) = 60, iar punctele

D si E de pe latura BC sunt astfel ıncat AE este bisectoarea unghiului ÕBAD, iar

AD = CE. Determinati masura unghiului ÕCAD.Titu Zvonaru, Comanesti

Clasa a XI-a

XI.141. Fie matricele A,B ∈ M2(R) cu AB = BA si det (A+ iB) = 0. Calculatideterminantul matricei A5 +A4B +AB4 +B5, functie de a = detA.

Dan Popescu, Suceava

XI.142. Consideram ecuatia X2+Y 2+Z2+ In = XY +XZ+Y Z, unde X,Y, Zsunt matrice din Mn(R), n ≥ 2, care comuta doua cate doua.

a) Aratati ca ecuatia nu are solutii pentru n impar.b) Daca n este par, aratati ca ecuatia are o infinitate de solutii.

Dumitru Craciun, Falticeni

XI.143. Intre unghiurile triunghiului ABC are loc relatia A2 = B2 + C2. De-

monstrati ca A ∈h(√2− 1)π,

π

2

.

Ioan Sacaleanu, Harlau

XI.144. Sirul (xn)n∈N este astfel ıncat x0 ∈ R\Q, iar xn(xn+1 − 1) = 1, ∀n ∈ N.Determinati limita sirului.

Mihaly Bencze, Brasov

XI.145. Se considera functia derivabila f : [a, b] → R cu f(a) = a si f(b) =b. Demonstrati ca oricare ar fi n ∈ N∗, exista x1, x2, . . . , xn ∈ (a, b) astfel ıncat

1

f ′(x1)+

1

f ′(x2)+ . . .+

1

f ′(xn)= n.

Dan Nedeianu, Drobeta Turnu Severin

Clasa a XII-a

XII.141. Daca f : R → R, f(x) =x4 + x+ 1

x2 + 1, calculati lim

x→∞

1

x3

Z x

0f(t)dt siZ 1

0

f ′′(x)

f(x)dx−

Z 1

0

f ′(x)

f(x)

2

dx.

Constantin Dragomir, Pitesti

168

Page 85: format .pdf, 3.6 MB

XII.142. Fie a ∈ R∗+ si functia continua f : R → R∗

+ cu proprietatea ca f(x) ·

f(−x) = 1, ∀x ∈ R. CalculatiZ a

−a

dx

(x2 + 2013)(1 + f(x)).

D.M. Batinetu-Giurgiu, Bucuresti si Neculai Stanciu, Buzau

XII.143. CalculatiR(x− sinx+ cosx)

√1− sinx ·

√1− cosxdx, x ∈

0,

π

2

.

Dan Nedeianu, Drobeta Turnu Severin

XII.144. Demonstrati ca nu exista numere naturale n si m astfel ıncat 19 sadivida 5n + 7m.

Marian Cucoanes, Marasesti

XII.145. Fie (A,+, ·) inel cu 1 = 0, avand un numar impar de elemente, ın careare loc implicatia: ,,daca x2 − 2xy+ y2 = 1+1+1+1, atunci x+ y = 1+1+1+1”.Daca 1 + 1 nu este divizor al lui zero, demonstrati ca A este izomorf cu Z3.

Florin Stanescu, Gaesti

Probleme pentru pregatirea concursurilor

A. Nivel gimnazialG246. Doi copii, A si B, joaca un joc. Acesta se desfasoara pe un careu format

din a × b patratelele, ın care a si b sunt numere naturale impare, propuse fiecare decatre unul dintre cei doi copii. Jucatorii bifeaza, pe rand, cate o casuta din careu,astfel: A ıncepe jocul prin bifarea unui patratel (m,n), unde m reprezinta linia, iarn coloana patratelului bifat. Apoi, B bifeaza unul dintre patratelele (m ± 1, n ± 3)sau (m ± 3, n ± 1), aflat ın interiorul careului. De fiecare data cand un jucator vinela rand, el alege o pozitie (p, q) deja bifata si are voie sa bifeze una dintre pozitiile(p± 1, q± 3) sau (p± 3, q± 1) care este ınca nebifata ın careu. Pierde jucatorul care,atunci cand ıi vine randul, nu mai are ce bifa.

Demonstrati ca A are strategie de castig.Silviu Boga, Iasi

G247. Fie A = 1, 2, 3, . . . , n, n ≥ 6, si X,Y doua submultimi disjuncte ale luiA, X ∪Y = A, avand fiecare cel putin trei elemente. Demonstrati ca exista x, y ∈ X,x = y si a, b ∈ Y, a = b, astfel ıncat x− y = a− b.

Gheorghe Iurea, Iasi

G248. Daca a ∈ N∗ aratati ca numarul 5a(a2 + 1) nu este patrat perfect.Gheorghe Iurea, Iasi

G249. Rezolvati ın numere naturale ecuatia 85m − n4 = 4.Cristinel Mortici, Targoviste

G250. Demonstrati ca a3+ b3 ≥ 2√ab(a−2b)(b−2a), oricare ar fi numerele reale

pozitive a si b.Gabriel Popa, Iasi

G251. Daca a, b, c sunt numere reale poztive cu ab + bc + ca = 3, aratati caa2(b+ c) + b2(c+ a) + c2(a+ b) ≥ 6.

Monica Golea, eleva, Craiova

169

Page 86: format .pdf, 3.6 MB

G252. Fie n ∈ N, n ≥ 2 si numerele reale pozitive x1, x2, . . . , xn. Daca S =x1 + x2 + . . .+ xn, demonstrati ca

max

x1 +

1

S − x1, x2 +

1

S − x2, . . . , xn +

1

S − xn

≥ 2√

n− 1.

Ani Draghici si Mariana Marculescu, Craiova

G253. Fie M un punct oarecare pe latura AB a patratului ABCD. Bisectoarea

unghiului ÖMDC intersecteaza BC ın N . Aratati ca BM +BN < AM + CN.Cecilia Deaconescu, Pitesti

G254. Diagonalele trapezului ABCD se intersecteaza ın O. Paralela prin O labaza AB intersecteaza latura BC ın P . Punctul Q este situat ın semiplanul opuscelui determinat de dreapta AD si punctul B, iar dreptele QB si QC intersecteazaAD ın R, respectiv S. Demonstrati ca dreptele PQ,BS si CR sunt concurente.

Claudiu-Stefan Popa, Iasi

G255. Fie AB si AC tangentele din punctul A la un cerc C (B si C fiind punctele

de tangenta) si R regiunea din plan determinata de arcul mic ÷BC al cercului C sisegmentele AB si AC. Demonstrati ca MN ≤ max(AB,AC), oricare ar fi puncteleM si N din R.

Marian Tetiva, Barlad

B. Nivel liceal

L246. Fie ABC cu m( bA) ≥ 90, ınscris ın cercul C. Pe latura BC se considera

puncteleD siD′ astfel ıncat ÕABC ≡ ÕCAD si ÕACB ≡ ÖBAD′. Cercul tangent dreptelorAD,BD si cercului C este tangent segmentului BD ın M . Cercul tangent dreptelor

AD′, CD′ si cercului C este tangent segmentului CD′ ın N . Aratati caMN

BC≤

√2−1.

Neculai Roman, Mircesti (Iasi)L247. Pe laturile BC, CA si AB ale triunghiului ABC se considera punctele

A1, B1, respectiv C1 astfel ıncat AB + BA1 = AC + CA1, AB + AB1 = BC + CB1

si AC +AC1 = BC +BC1. Daca A2, B2 si C2 sunt punctele de tangenta ale cerculuiınscris ın triunghiul ABC cu laturile BC,CA, respectiv AB, aratati ca A1B

21+B1C

21+

C1A21 ≥ A2B

22 +B2C

22 + C2A

22.

Marius Olteanu, Rm. ValceaL248. Demonstrati ca ın orice triunghi are loc inegalitatea

3(ra + rb + rc)

2p2≥ 1

ra + rb+

1

rb + rc+

1

rc + ra≥ 6

5R+ 2r.

Andi Gabriel Brojbeanu, elev, Targoviste

L249. Fie ABCD un patrulater atat inscriptibil, cat si circumscriptibil. Daca

notam cu e si f lungimile diagonalelor, demonstrati capÈ

2√2Rr

≥ e+ f√ef

.

Vasile Jiglau, Arad

L250. Stabiliti pentru care dintre numerele 1, 2, . . . , 9 este adevarata egalitatea

tgπ

n− tg

n+ tg

n=

√3n.

Ionel Tudor, Calugareni

170

Page 87: format .pdf, 3.6 MB

L251. Fie n ∈ N, n ≥ 2, si numerele reale nenegative x1, x2, . . . , xn cu pro-prietatea ca x2

1 + x22 + . . . + x2

n = 3n2. Demonstrati ca (x1 + x2 + . . . + xn)3 ≥

9n(x1x2 + x1x3 + . . .+ xn−1xn).Lucian Tutescu si Ionut Ivanescu, Craiova

L252. Fie n ∈ N, n ≥ 5, si numerele reale a1 < a2 < . . . < an. Se calculeazatoate sumele ai + aj , i = j, obtinand t rezultate distincte. Demonstrati ca t ≥ 2n− 3si ca t = 2n− 3 daca si numai daca a1, a2, . . . , an este progresie aritmetica.

Titu Zvonaru, Comanesti

L253. Fie a, b, c trei numere reale pozitive cu a ≤ c si x, y, z ∈ [a, c] astfel ıncat

x+ y+ z = a+ b+ c si1

x+

1

y+

1

z=

1

a+

1

b+

1

c. Aratati ca numerele x, y si z coincid

ıntr-o anumita ordine, cu a, b si c.Marian Tetiva, Barlad

L254. Determinati numerele reale x, y, z din intervalul [1, 3] astfel ıncat x2+y2+z2 = 14 si x3 + y3 + z3 = 36.

Marian Tetiva, Barlad

L255. Se considera numerele reale a < c < b si sirul (xn)n≥1. Orice subsirconvergent al sirului (xn) are limita a sau limita b. Notam An = k ∈ N|k ≤ n sixk ≤ c si Bn = k ∈ N|k ≤ n si xk > c. Daca exista si este finita si nenula limita

L = limn→∞

cardAn

cardBn, aratati ca sirul yn =

x1 + xn + . . .+ xn

neste convergent si aflati

limita sa (functie de a, b si L). Studiati si cazurile L = 0 si L = +∞.Cristinel Mortici, Targoviste

Training Problems for Mathematical Contests

A. Junior Highschool Level

G246. Two children, A and B, play a game. This takes place on a rectangularsquare consisting of a × b small squares, where a and b are odd natural numbers,each of them proposed by one of the two children. The players mark, successivelyand once at a time, a cell in the table as it follows : A begins the game by marking asmall square (m,n), where m represents the row and n − the column of the markedcell. Then, B marks one of the cells (m± 1, n± 3) or (m± 3, n± 1), situated insidethe big table. Every time when a player comes to his turn, he chooses an alreadymarked position (p, q) and he is allowed to mark one of the positions ( p ± 1, q ± 3)or ( p ± 3, q ± 1) whick is still blank on the table. The player who has no more cellto mark when its turn comes up loses the game. Prove that A has a strategy forwinning.

Silviu Boga, Iasi

G247. Let A = 1, 2, 3, . . . , n, n ≥ 6, and X,Y be two disjoint subsets of the setA, X ∪ Y = A, each of them consisting of at least three elements. Prove that fourelements x, y ∈ X, x = y and a, b ∈ Y, a = b exist such that x− y = a− b.

Gheorghe Iurea, Iasi

171

Page 88: format .pdf, 3.6 MB

G248. If a ∈ N∗ show that the number 5a (a 2 + 1) is not a perfect square.Gheorghe Iurea, Iasi

G249. Solve in natural numbers the equation 85m − n 4 = 4.Cristinel Mortici, Targoviste

G250. Prove that a3+ b3 ≥ 2√ab(a−2 b)(b−2 a), any would be the positive real

numbers a and b.Gabriel Popa, Iasi

G251. If a, b, c are positive real numbers such that ab + bc + ca = 3, show thata2(b+ c) + b2(c+ a) + c2(a+ b) ≥ 6.

Monica Golea, eleva, Craiova

G252. Let n ∈ N, n ≥ 2 and consider the positive real numbers x1, x2, . . . , xn.If S = x1 + x2 + . . .+ xn, prove that

max

x1 +

1

S − x1, x2 +

1

S − x2, . . . , xn +

1

S − xn

≥ 2√

n− 1.

Ani Draghici si Mariana Marculescu, Craiova

G253. Let M be an arbitrary point on the side AB of the square ABCD. The

angle bisector of ÖMDC intersects the side BC at point N . Show that BM +BN <AM + CN.

Cecilia Deaconescu, Pitesti

G254. The diagonals of the trapezium ABCD cut each other at point O. Theline through O that is parallel to the base AB intersect the side BC at P . The pointQ is situated in the opposite half-plane to the one determine by line AD and pointB, and the lines QB and QC intersect AD at R, respectively at S. Prove that thelines PQ, BS and CR are concurrent.

Claudiu-Stefan Popa, Iasi

G255. Let AB and AC be the tangents from point A to a circle C (B and C

being the points of contact) and let R be the plane region closed by the small arc ÷BCof circle C and the line segments AB and AC. Prove that MN ≤ max(AB,AC),any would be the points M and N in R.

Marian Tetiva, Barlad

B. Highschool Level

L246. Let us consider the triangle ABC with m( bA) ≥ 90, which is inscribed in

the circle C. The points D and D′ are taken on the side BC such that ÕABC ≡ ÕCAD

and ÕACB ≡ ÖBAD′. The tangent circle to the lines AD, BD and to the circle C isalso tangent to the line segment BD at M . The circle tangent to the lines AD′, CD′

and to circle C is tangent to the line segment CD′ at N . Show thatMN

BC≤

√2− 1.

Neculai Roman, Mircesti (Iasi)

L247. Three points are consideered on the sides BC, CA and AB of the triangleABC, respectively denoted A1, B1 and C1, such that AB + BA1 = AC + CA1,

172

Page 89: format .pdf, 3.6 MB

AB + AB1 = BC + CB1 and AC + AC1 = BC + BC1. If A2, B2 and C2 are thecontact points of the circle inscribed in triangle ABC with the sides BC, CA andrespectively AB, show that A1 B

21 +B1C

21 + C1A

21 ≥ A2 B

22 +B2 C

22 + C2 A

22.

Marius Olteanu, Rm. Valcea

L248. Prove that the following inequality holds in any triangle :

3(ra + rb + rc)

2p2≥ 1

ra + rb+

1

rb + rc+

1

rc + ra≥ 6

5R+ 2r.

Andi Gabriel Brojbeanu, elev, Targoviste

L249. Let ABCD be a bicentric quadrilateral, and let e and f denote the lengths

of its diagonals. Prove thatpÈ

2√2Rr

≥ e+ f√ef

.

Vasile Jiglau, Arad

L250. Establish which numbers n among 1, 2, . . . , 9 satisfy the equation tgπ

n−

tg2π

n+ tg

n=

√3n.

Ionel Tudor, Calugareni

L251. Let n ∈ N, n ≥ 2 and consider the nonnegative numbers x1, x2, . . . , xn

with the property that x21 + x2

2 + . . .+ x2n = 3n2. Prove that (x1 + x2 + . . .+ xn)

3 ≥9n (x1x2 + x1x3 + . . .+ xn−1xn).

Lucian Tutescu si Ionut Ivanescu, Craiova

L252. Let n ∈ N, n ≥ 5 and the real numbers a1 < a2 < . . . < an. All the sumsai + aj , i = j are calculated, giving distinct results. Prove that t ≥ 2n− 3 and thatt = 2n− 3 if and only if a1, a2, . . . , an is an arithmetic progression.

Titu Zvonaru, Comanesti

L253. Let a, b, c be positive real numbers with a ≤ c and x, y, z ∈ [a, c] such

that x+ y+ z = a+ b+ c and1

x+

1

y+

1

z=

1

a+

1

b+

1

c. Show that the numbers x, y

and z respectively coincide, in a certain order, with a, b and c.Marian Tetiva, Barlad

L254. Find the real numbers x, y, z in [1, 3] such that x2 + y2 + z2 = 14, andx3 + y3 + z3 = 36.

Marian Tetiva, Barlad

L255. The real numbers a < c < b and the sequence (xn)n≥ 1 are considered.Any convergent subsequence of sequence (xn) has limit a or limit b. Let us denoteAn = k ∈ N | k ≤ n and xk ≤ c and Bn = k ∈ N | k ≤ n and xk > c. If

there exists the limit L = limn→∞

cardAn

cardBnand it is finite and nonzero, show that the

sequence yn =x1 + xn + . . .+ xn

nconverges and find its limit (as a function of a, b

and L). Study the cases L = 0 and L = +∞ as well.Cristinel Mortici, Targoviste

173

Page 90: format .pdf, 3.6 MB

Pagina rezolvitorilor

CAMPULUNG MUSCEL

Colegiul National ,,Dinicu Golescu”. Clasa a X-a (prof. PETRISOR Con-stantin). NECULA Emanuel: IX(137,139), X(136-140), XI(136-138), G(239,243),L.243.

GRAJDURI (IASI)

Scoala Gimnaziala ,,Valea Satului”. Clasa a IV-a (prof.ınv.primar URMA Ana-Lacramioara). CASANDROI Gabriela: P(257,258,260,262,264); PETIEANU Ionela-Vasilica: P(257,258,260,262,264); RUSU Alexandra: P(257,258,260,262,264); STOI-CA Mariana-Ionela: P(257,258,260,262,264).

IASI

Scoala nr. 3 ,,Al. Vlahuta”. Clasa a VI-a (prof. MARIN Mirela). ENEACodrut: P(266,267), VI(158,161,163); POPOVICI Teodor-Andrei: P(266-268), VI(158,160-163). Clasa a VII-a (prof. MARIN Mirela). CIOCOIU Alexandra: VI(158,163),VII(160-162); CONDURACHE Alexandra: VI(158,163), VII(160-162); COSTEA Ale-xandru: VI(158,163), VII(160-162); MARINMarius: VI(158,161), VII(160-162); MAN-ZAT Rebecca: VI(158,163), VII(160-162); TROFIN Ana: VI(158,163), VII(160-162);VLAD Ioana: VI(158,163), VII(160-162).

Scoala Gimnaziala nr. 26 ,,G. Cosbuc”. Clasa a IV-a (prof.ınv.primar RACUMaria). CIOPEICA Sebastian-Andrei: P(255,261,263,265,266); GROSU Victor-Ale-ssandru: P(255,256,260-262,265); LUCHIAN Maria-Clara: P(255-258,260-262,265);MANOLE Alexandra-Georgiana: P(255-262,265); PASNICU Cosmin-Constantin:P(255-257,259-261,265); POPESCU Andrei-Eduard: P(255,259-261,265); RAILEANUAna-Maria: P(255-262,265); RAILEANU Razvan-Constantin: P(255-262,265); VA-SILE Raluca-Andreea: P(255-257,259-262).

Scoala nr. 33 ,,M. Kogalniceanu”. Clasa a III-a (ınv. SIRBU Lenuta). CIO-COIU Alexandru Boris: P(261-266).

Liceul Economic ,,Virgil Madgearu”. Clasa a IX-a (prof. OLENIUC Claudia).JOLDESCU Petronela Lavinia: VII(159,160), VIII(158,159,162); LUNGU AdelinaVeronica: VII(159,160), VIII(158,159,162); PLOP Cosmin Alexandru: VII(159,160),VIII(158,159,162). Clasa a X-a (prof. OLENIUC Claudia). GEORGESCU Razvan:VIII(158,159,162), X.138, XI.136; GHERGHEL Petruta: VIII(158,159,162), X.138,XI.136; MARIN Ileana: VIII(158,159,162), X.138, XI.136; PLACINTA Catalina-Alexandra: VIII(158,159,162), X.138, XI.136; SCRAB Bianca-Maria: VIII(158,159,162), X.138, XI.136. Clasa a XI-a (prof. OLENIUC Claudia). SIRGHI Nico-leta: X(138,140), XI.136, XII(136,138); SOFRONEA Daniela: X(138,140), XI.136,XII(136,138).

Colegiul National ,,Emil Racovita”. Clasa a V-a (prof. BUDEANU Catalin).OLENIUC Iulian: P(266,267), V.159, VI(158,160).

Colegiul National Iasi. Clasa a VI-a (prof. LAZAR Cristian). POPA IoanaMaria: VI(158-164), VII(158-163). Clasa a VIII-a (prof. POPA Gabriel). DOMIN-TE Stefan: VII(158-164), VIII(158-164).

174

Page 91: format .pdf, 3.6 MB

Colegiul National ,,M. Eminescu”. Clasa a V-a (prof. BLENDEA Gheorghe).ALISTAR Stefan-Daniel: P(265-268), V.159; VASILIU Razvan Andrei: P(265-268),V.159; ZAMCANU Andrei-Rares: P(265-268), V.159.

Liceul Teoretic ,,Miron Costin”. Clasa a IV-a. LUCHIAN Denisa Alexandra:P(260,263-268), V.161.

ROSIORI (BACAU)Scoala Gimnaziala nr. 1. Clasa a VIII-a (prof. CICEU Nela). ONOFREI

Adina: VI(158,163), VII(158-163), VIII(159,161,162).

TARGOVISTEScoala ,,Vasile Carlova”. Clasa I (ınv. ION Daniela). BACIU Maria Briana:

P(255-257,259,260).

TRUSESTI (BOTOSANI)Liceul Teoretic ,,Demostene Botez”. Clasa a XII-a (prof. CULIDIUC Catalin).

PAUCA Ovidiu: X.138, XI.136, XII(136-139), G.240, L.240.

TIGANASI (IASI)Scoala Gimnaziala cu clasele I-VIII ,,M. Kogalniceanu”. Clasa a II-a (prof.ınv.

primar VOICU Ana). GANEANU Andreea: P(255-258,260); PIU Eduard-Narcis:P(255-258,260); PIU Iuliana: P(255-258,260); RUSU Alexandru-Gabriel: P(255-258,260); SANDU Ruben: P(255-258,260). Clasa a III-a (ınv. SAMSON Daniel-Mihai).DUCA Daria-Catalina-Stela: P(263-267). Clasa a IV-a (prof.ınv.primar BADITAAurica). DUCA Anamaria: P(261,263-267); DUCA Ema-Stefania: P(261,263-267);DUCA Roxana-Georgiana: P(261,263-267); GHIOANCA Alexandra-Ionela: P(261,263-267); SANDU Marta: P(261,263-267); STEFANACHE Anamaria: P(261,263-267); TICAN David-Petru: P(261,263-267). Clasa a V-a (prof. IACOB Aida An-dreea). DUCA Adriana: P(265-267), V(159,160); DUCA Denis-Alexandru: P(265-267), V(159,160); GANEANU Maria-Teodora: P(265-267), V(159,160); ROMANIUCGeorgiana: P(265-267), V(159,160). Clasa a VI-a (prof. IACOB Aida Andreea).CAZADOI Ioana Cristina: P.265, V(159,163), VI.164, VII.158.

Elevi rezolvitori premiati

Scoala nr. 3 ,,Al. Vlahuta”, Iasi

MARIN Marius (cl. a VII-a): 2/2012(5pb), 1/2013(5pb), 2/2013(5pb).

Scoala nr. 26 ,,G. Cosbuc”, Iasi

VASILE Raluca Andreea (cl. a IV-a): 2/2012(5pb), 1/2013(7pb), 2/2013(7pb).

Scoala nr. 33 ,,M. Kogalniceanu”

CIOCOIU Alexandru Boris (cl. a III-a): 2/2012(9pb), 1/2013(6pb), 2/2013(6pb).

Scoala Gimnaziala ,,Valea Satului”, Grajduri (Iasi)

175

Page 92: format .pdf, 3.6 MB

RUSU Alexandra (cl. a IV-a): 2/2012(5pb), 1/2013(5pb), 2/2013(5pb).STOICA Mariana Ionela (cl. a IV-a): 2/2012(5pb); 1/2013(5pb), 2/2013(5pb).

Scoala Gimnaziala cu clasele I-VIII ,,M. Kogalniceanu”, Tiganasi (Iasi)

GHIOANCA Alexanda-Ionela (cl. a IV-a): 2/2012(11pb); 1/2013(7pb), 2/2013(6pb).STEFANACHE Anamaria (cl. a IV-a): 2/2012(11pb), 1/2013(7pb), 2/2013(6pb).TICAN David-Petru (cl. a IV-a): 2/2012(11pb), 1/2013(7pb), 2/2013(6pb).DUCA Anamaria (cl. a IV-a): 1/2012(6pb), 1/2013(7pb), 2/2013(6pb).

Liceul Teoretic ,,Miron Costin”, Iasi

LUCHIAN Denisa Alexandra (cl. a IV-a): 2/2012(7pb), 1/2013(10pb), 2/2013(8pb).

Liceul Economic ,,Virgil Madgearu”, Iasi

SIRGHI Nicoleta (cl. a XI-a): 1/2012 (5pb), 2/2012(5pb), 2/2013(5pb).SOFRONEA Daniela (cl. a XI-a): 1/2012 (5pb), 2/2012(5pb), 2/2013(5pb).

Colegiul National ,,Emil Racovita”, IasiOLENIUC Iulian (cl. a V-a): 2/2012(7pb), 1/2013(6pb), 2/2013(5pb).

IMPORTANT

• In scopul unei legaturi rapide cu redactia revistei, pot fi utilizate urmatoareleadrese e-mail: t [email protected] si [email protected] . Peaceasta cale colaboratorii pot purta cu redactia un dialog privitor la ma-terialele trimise acesteia, procurarea numerelor revistei etc. Sugeram cola-boratorilor care trimit probleme originale pentru publicare sa le numerotezesi sa-si retina o copie xerox a lor pentru a putea purta cu usurinta o discutieprin e-mail asupra acceptarii/neacceptarii acestora de catre redactia revistei.

• La problemele de tip L se primesc solutii de la orice iubitor de matematicielementare (indiferent de preocupare profesionala sau varsta). Fiecare dintresolutiile acestor probleme - ce sunt publicate ın revista dupa jumatate dean - va fi urmata de numele tuturor celor care au rezolvat-o.

• Adresam cu insistenta rugamintea ca materialele trimise revisteisa nu fie (sa nu fi fost) trimise si altor publicatii.

• Rugam ca materialele tehnoredactate sa fie trimise pe adresa redactieiınsotite de fisierele lor (de preferinta ın LATEX).

• Pentru a facilita comunicarea redactiei cu colaboratorii ei, autorii materi-alelor sunt rugati sa indice adresa e-mail.

176

Page 93: format .pdf, 3.6 MB

Revista semestrială RECREAŢII MATEMATICE este editată de ASOCIAŢIA “RECREAŢII MATEMATICE”. Apare la datele de 1 martie şi 1 septembrie şi se adresează elevilor, profesorilor, studenţilor şi tuturor celor pasionaţi de matematica elementară.

În atenţia tuturor colaboratorilor Materialele trimise redacţiei spre publicare (note şi articole, chestiuni de metodică,

probleme propuse etc.) trebuie prezentate îngrijit, clar şi concis; ele trebuie să prezinte interes pentru un cerc cât mai larg de cititori. Se recomandă ca textele să nu depăşească patru pagini. Evident, ele trebuie să fie originale şi să nu fi apărut sau să fi fost trimise spre publicare altor reviste. Rugăm ca materialele tehnoredactate să fie însoţite de fişierele lor.

Problemele destinate rubricilor: Probleme propuse şi Probleme pentru pregătirea concursurilor vor fi redactate pe foi separate cu enunţ şi demonstra-ţie/rezolvare (câte una pe fiecare foaie) şi vor fi însoţite de numele autorului, şcoala şi localitatea unde lucrează/învaţă.

Redacţia va decide asupra oportunităţii publicării materialelor primite. În atenţia elevilor Numele elevilor ce vor trimite redacţiei soluţii corecte la problemele din rubricile

de Probleme propuse şi Probleme pentru pregatirea concursurilor vor fi menţionate în Pagina rezolvitorilor. Elevii menţionaţi de trei ori vor primi o diplomă şi un premiu în cărţi. Elevii rezolvitori vor ţine seama de regulile:

1. Pot trimite soluţii la minimum cinci probleme propuse în numărul prezent şi cel anterior al revistei (pe o foaie va fi redactată o singură problemă).

2. Elevii din clasele VI-XII au dreptul să trimită soluţii la problemele propuse pentru clasa lor, pentru orice clasă mai mare, din două clase mai mici şi imediat anterioare. Cei din clasa a V-a pot trimite soluţii la problemele propuse pentru clasele a IV-a, a V-a şi orice clasă mai mare, iar elevii claselor I-IV pot trimite soluţii la problemele propuse pentru oricare din clasele primare şi orice clasă mai mare. Orice elev poate trimite soluţii la problemele de concurs (tip G şi L).

3. Vor fi menţionate următoarele date personale: numele şi prenumele, clasa, şcoala şi localitatea, precum şi numele profesorului cu care învaţă.

4. Plicul cu probleme rezolvate se va trimite prin poştă (sau va fi adus direct) la adresa Redacţiei:

Prof. dr. Temistocle Bîrsan Str. Aurora, nr. 3, sc. D, ap. 6, 700 474, Iaşi Jud. IAŞI E-mail: [email protected]

Page 94: format .pdf, 3.6 MB

CUPRINS Profesorul CONSTANTIN CORDUNEANU la aniversarea a 85 de ani (T. Bîrsan) ............. 89

ARTICOLE ŞI NOTE M. TĂRNĂUCEANU – Grupuri finite cu proprietatea (P) .......................................................... 92 I. PĂTRAŞCU – Cercurile mixtliniare adjuncte înscrise asociate unui triunghi .......................... 96 D.M. BĂTINEŢU-GIURGIU, N. STANCIU – Cîteva aplicaţii ale inegalităţii Ionescu- Weitzenböck ..................... 100 M. BENCZE – Câteva generalizări şi rafinări ale inegalităţii x2 +y2 +z2 ≥ xy+yz+zx ................. 103

NOTA ELEVULUI A.G. BROJBEANU – Caracterizarea unor proprietăţi de perpendicularitate în care sunt implicate punctele O, I, H, G, O9 ................... 106

CORESPONDENŢE A. REISNER – La cissoïde, podaire de la parabole ………………………................................. 110 F. SMARANDACHE – Another proof of the Pătraşcu’s theorem ……………………….……. 114

CHESTIUNI METODICE M. MONEA, S. MONEA – Comentarii pe marginea unor probleme .......................................... 116 D. POPESCU – Asupra unei clase de ecuaţii/ inecuaţii ............................................................... 119 F. STĂNESCU – Rezolvarea unor ecuaţii şi inecuaţii integrale .................................................. 122

CUM CONCEPEM ... CUM REZOLVĂM M. TETIVA – Merge şi aşa! ....................................................................................................... 126

ŞCOLI ŞI DASCĂLI

A. NEICU – Colegiul Naţional „Ştefan cel Mare” Hârlău ........................................................ 129

CONCURSURI ŞI EXAMENE Concursul de matematică "Florica T. Câmpan", ed. a XIII-a, 2013 ..................................... 132

PROBLEME ŞI SOLUŢII Soluţiile problemelor propuse în nr. 1/2013 ............................................................................. 136 Soluţiile problemelor pentru pregătirea concursurilor propuse în nr. 1/2013 ........................... 151 Probleme propuse ......................................................................................................................... 163 Probleme pentru pregătirea concursurilor ................................................................................... 169 Training problems for mathematical contests ............................................................................ 171 Pagina rezolvitorilor .................................................................................................................... 174 Elevi rezolvitori premiaţi ............................................................................................................ 175 ISSN 1582 – 1765 8 lei